fizikai szemle 68/7–8 – 2018....

80
fizikai szemle 2018/7–8

Upload: others

Post on 20-Oct-2019

0 views

Category:

Documents


0 download

TRANSCRIPT

Page 1: Fizikai Szemle 68/7–8 – 2018. július–augusztusgoliat.eik.bme.hu/~f.simon/publications/Media/FizSzem-20180708.pdf · okokból standard modellnek hívunk, a világot anyagi és

fizikai szemle

2018/7–8

Page 2: Fizikai Szemle 68/7–8 – 2018. július–augusztusgoliat.eik.bme.hu/~f.simon/publications/Media/FizSzem-20180708.pdf · okokból standard modellnek hívunk, a világot anyagi és

5 mm

a) b)

c) d)

e) f)

g) h)

i) j)

k) l)

m) n)

o) p)

1. ábra. Bábállapotbeli hûtés hatása Ikarusz boglárka lepkék imágói-nak szárnyaira. Hûtetlen hím egyed szárnyainak felszíne (a, c) és fo-nákja (e, g), 62 napig hûtött hím szárnyainak felszíne (b, d) és fonákja(f, h), hûtetlen nõstény szárnyainak felszíne (i, k) és fonákja (m, o), 40napig hûtött nõstény szárnyainak felszíne (j, l) és fonákja (n, p). (LásdPiszter Gábor és szerzõtársai írását a 225. oldaltól.)

5. ábra. Ikarusz boglárka hím és nõstény egyedeinek szárnyairólkészült optikai mikroszkópos felvételek. (a) Hûtetlen hím szárnyfelszí-ne, (b) 22 napig hûtött hím szárnyfelszíne, (c) hûtetlen nõstény szárny-felszíne, (d) 20 napig hûtött nõstény szárnyfelszíne. (Lásd Piszter Gá-bor és szerzõtársai írását a 225. oldaltól.)

200 m�

a)

b)

c)

d)

Page 3: Fizikai Szemle 68/7–8 – 2018. július–augusztusgoliat.eik.bme.hu/~f.simon/publications/Media/FizSzem-20180708.pdf · okokból standard modellnek hívunk, a világot anyagi és

DUPLASZÁMRemélem, hogy a szokás szerint augusztus végén megjelenô nyári dupla-számunkban – amiben néhány terület több cikkel is szerepel – mindenOlvasónk sok, számára érdekes írást talál.

Részecskefizikával két cikk foglalkozik: Horváth Dezsôé a CERN NagyHadronütköztetôjénél kapott érdekes, de más kísérletek által sem megerôsí-tett, sem megcáfolt eredményekkel ismertet meg, amelyek akár „új fizika”születését is jelezhetnék. Ugyanakkor a szerzô felhívja a figyelmet, hogymindig óvatosan kell kezelnünk a feltûnô új jelenségek észlelését: fontos azeredmény megfelelô matematikai-fizikai megbízhatósága és a függetlenmérés általi megerôsítése. A „Fizika tanítása” rovatunkban Oláh Éva Máriaés Fülöp Csilla cikke azt vizsgálja, hogy milyen lehetôségeink vannak, hogya 14–18 éves diákjainkat megismertessük az érdeklôdés középpontjábanálló részecskefizikával.

Ugyancsak két írásunk foglalkozik az élôvilág és a fizika találkozásával:Piszter Gábor és munkatársai cikke a lepkék szárnyának nanoarchitektúrá-ját mutatja be korszerû vizsgálati módszerekkel végzett méréseik alapján;a tanítási cikkek között pedig Rajkovits Zsuzsanna a víziállatok életjelensé-geinek egész sorát tárgyalja a hidrosztatika témaköréhez kapcsolódóan.Piszter Gáborék írása ugyanakkor az anyagtudomány nanoszerkezeteketvizsgáló vonulatához tartozik, és ugyancsak a korszerû anyagtudományszámítógépes módszereinek alkalmazására mutat példákat Pusztai Tamásés munkatársai cikke.

Az egyre fejlettebb mérési módszerek jelentôségét a csillagászat és aszt-rofizika területérôl érkezett két cikk is igazolja. Szûcs Tamás a csillagokbanlejátszódó magreakciók leírásában fontos, igen alacsony hatáskeresztmet-szetek pontosabb laboratóriumi meghatározásának – a detektorok aktívárnyékolásán alapuló – lehetôségét mutatja be. Kóspál Ágnes és munkatár-sainak írása a csillagkeletkezési folyamatok részleteinek vizsgálatát segítô újóriásberendezésekkel ismertet meg minket, egyebek között az 1 milliárdeurós költséggel Chilében megépített ALMA teleszkóppal.

A nukleáris ipar és technológia iránti tanulói érdeklôdés fokozása különö-sen fontos jelenleg, amikor elkezdôdött a két új paksi blokk létesítésénekelôkészítése. Az erre irányuló két nagy hagyománnyal rendelkezô rendez-vénysorozatról, az Országos Szilárd Leó Nukleáris Tanulmányi Versenyrôl,illetve a Nukleáris Szaktáborról egy-egy cikkünk számol be.

A nyár ad aktualitást egy különleges témával foglalkozó írásnak, amelybenMolnár János Albert az általa konstruált siófoki kétszálas napórát mutatja be.

E számunkban – régi törekvésünknek megfelelôen – a fizika tanításánakminden szintjével foglalkozunk. Oláh Éva Mária és Fülöp Csilla már emlí-tett cikke mellett Simon Ferenc írása az egyetemi oktatás egy témájátdolgozza fel. Juhász András és Tél Tamás pedig az ELTE Fizika Tanításadoktori programjának – amely gyakorló fizikatanárok számára teszi elér-hetôvé a doktori fokozat megszerzését – 10 éves tapasztalatait és ered-ményeit foglalja össze.

Nagy örömünkre szolgál, ha egy korábban megjelent cikkünkhöz érkezikkiegészítés vagy magyarázat, mint amelyet Kürti Jenô küldött Kiss Miklósrelativitáselmélet oktatásáról szóló, áprilisi számunkban közölt írásához.

Lendvai Jánosfôszerkesztô

Page 4: Fizikai Szemle 68/7–8 – 2018. július–augusztusgoliat.eik.bme.hu/~f.simon/publications/Media/FizSzem-20180708.pdf · okokból standard modellnek hívunk, a világot anyagi és

Fizikai SzemleMAGYAR FIZIKAI FOLYÓIRAT

megjelenését támogatják: A FIZIKA BARÁTAI

Az Eötvös Loránd Fizikai Társulat havontamegjelenõ folyóirata.

Támogatók: a Magyar TudományosAkadémia Fizikai Tudományok Osztálya,az Emberi Erõforrások Minisztériuma,

a Magyar Biofizikai Társaság,a Magyar Nukleáris Társaság

és a Magyar Fizikushallgatók Egyesülete

Fõszerkesztõ:Lendvai János

Szerkesztõbizottság:Bencze Gyula, Biró László Péter,

Czitrovszky Aladár, Füstöss László,Gyürky György, Hebling János,Horváth Dezsõ, Horváth Gábor,

Iglói Ferenc, Kiss Ádám, Koppa Pál,Ormos Pál, Papp Katalin, Simon Ferenc,

Simon Péter, Sükösd Csaba,Szabados László, Szabó Gábor,

Takács Gábor, Trócsányi Zoltán,Ujvári Sándor

Mûszaki szerkesztõ:Kármán Tamás

A folyóirat e-mailcíme:[email protected]

A lapba szánt írásokat erre a címre kérjük.

A beküldött tudományos, ismeretterjesztõ ésfizikatanítási cikkek a Szerkesztõbizottság,illetve az általa felkért, a témában elismert

szakértõ jóváhagyó véleménye utánjelenhetnek meg.

A folyóirat honlapja:http://www.fizikaiszemle.hu

A címlapon:Egy polikristályos csírából kialakuló

különleges növekedési forma, amelyet azMTA Wigner FK SZFI munkatársai állítottak

elõ fázismezõmodell segítségével.Hasonló virágszerû szferolitok

a természetben is megfigyelhetõk.

TARTALOM

Lendvai János: Duplaszám 217Horváth Dezsõ: Új felfedezések a CERN Nagy Hadronütköztetõjénél:

furcsa részecskék 219Piszter Gábor, Kertész Krisztián, Horváth Zsolt Endre, Biró László

Péter, Bálint Zsolt: Ikarusz boglárka lepkék szerkezeti és pigmenteredetû színeinek stresszállósága 225

Szücs Tamás: Alacsony hátterû magfizikai mérések, avagy a nukleárisasztrofizika kihívásai 230

Kóspál Ágnes, Ábrahám Péter, Varga József: A csillagkeletkezésvizsgálata új interferometrikus mûszerekkel 235

Pusztai Tamás, Rátkai László, Gránásy László: Anyagtudományszámítógéppel – 2. rész 241

Molnár János Albert: Kétszálas napóra – egy ritkaság Siófokon 245Juhász András, Tél Tamás: Tíz éves az ELTE Fizika Doktori Iskola

„Fizika Tanítása Programja” gyakorló fizikatanárok számára 251

A FIZIKA TANÍTÁSARajkovits Zsuzsanna: Fizika az élõ természetben 256Kürti Jenõ: Relativitáselméletrõl középiskolában – másként, kiegészítés 263Sükösd Csaba: XXI. Országos Szilárd Leó Nukleáris Tanulmányi

Verseny – 1. rész 267Mester András: A tizedik Nukleáris Szaktábor 272Rátz Tanár Úr életmûdíj, 2017 – Mester András tanár úrral Radnóti

Katalin beszélget 275Simon Ferenc: Nagyfrekvenciás jelek kábelbeni terjedésének fizikai

alapjai – 1. rész 278Oláh Éva Mária, Fülöp Csilla: A csapból is részecskefizika folyik? 283

KÖNYVESPOLCDaniel Whiteson, Jorge Cham: Halványlila gõzünk sincs (Szalai Tamás ) 288

HÍREK – ESEMÉNYEKÖtven évvel ezelõtt hunyt el Gyulai Zoltán, a hazai kísérleti

szilárdtest-fizika úttörõje 289Sólyom Jenõ: Emlékbeszéd Gyulai Zoltán mellszobránál 289Hartmann Ervin: Tanítványok tanítványai 291Búcsú: Bonifert Domonkosné Bottyán Katalin 291Humboldt-díjban részesült Legeza Örs 292

J. Lendvai: Double-issueD. Horváth: New inventions at the LHC of CERN: strange particlesG. Piszter, K. Kertész, Zs. E. Horváth, L. P. Biró, Zs. Bálint: Stress resistance

of structural and pigmentary colors of polyommatus icarus butterfliesT. Szücs: Low background nuclear measurements or the challenges of nuclear

astrophysicsÁ. Kóspál, P. Ábrahám, J. Varga: Investigation of star formation by new

interferometric instrumentsT. Pusztai, L. Rátkai, L. Gránásy: Computational materials science – Part 2J. A. Molnár: Double strand sun dial in SiófokJuhász András, Tél Tamás: 10 years of “Teaching of Physics” Program of

ELTE’s PhD School in Physics

TEACHING PHYSICSZs. Rajkovits: Physics in living natureKürti Jenõ: Comments on a paper of Miklós KissCs. Sükösd: 21st Szilárd Leo National Nuclear Study Competition – Part 1A. Mester: 10th Nuclear Specialized CampRátz Life Achievement Award of 2017 – A. Mester, teacher interviewed by K. RadnótiF. Simon: Propagation of high frequency signals in wires – Part 1É. M. Oláh, Cs. Fülöp: Particle physics all around

BOOKS, EVENTS

A Mathematikai és Természettudományi Értesítõt az Akadémia 1882-ben indította

A Mathematikai és Physikai Lapokat Eötvös Loránd 1891-ben alapította

Fizikai SzemleMAGYAR FIZIKAI FOLYÓIRAT

LXVIII. ÉVFOLYAM, 7–8. (763–764.) SZÁM 2018. JÚLIUS–AUGUSZTUS

Page 5: Fizikai Szemle 68/7–8 – 2018. július–augusztusgoliat.eik.bme.hu/~f.simon/publications/Media/FizSzem-20180708.pdf · okokból standard modellnek hívunk, a világot anyagi és

ÚJ FELFEDEZÉSEK A CERN NAGY

1. ábra. A svájci–francia határon (keresztekkel jelölve) fekvõ CERN és környéke a Szuper-proton-szinkrotron (SPS) és az LHC gyûrûivel, valamint háttérben az Alpokkal. Jelentõs magyar csoport aCMS, ALICE, NA61 és TOTEM kísérleteknél dolgozik, de most az LHCb-rõl lesz inkább szó.

2. ábra. A standard modell részecskéi: fermionok (kvarkok és lep-tonok) és bozonok, a kölcsönhatások közvetítõi.

ne

e

nm

m

nt

t W

Z

g

gu

d

c

s

t

b

H

Higgs-bozonkvar

kok

lep

ton

ok

közvetítõ

k

HADRONÜTKÖZTETÕJÉNÉL: FURCSA RÉSZECSKÉK

Horváth Dezsõ Széchenyi-díjas kísérleti ré-szecskefizikus. 1970-ben végzett az ELTE-n,vizsgálatait Dubnában és Leningrádbankezdte, a kanadai TRIUMF-ban, az amerikaiBNL-ben, a svájci Paul-Scherrer Intézetben,az olasz INFN-ben, majd a CERN-ben foly-tatta. Budapest–Debrecen kutatócsoporto-kat szervezett CERN-kísérletekre. 2006 ótakoordinálja a magyar fizikatanárok részecs-kefizikai oktatását a CERN-ben. Emeritusprofesszor, magántanárként részecskefizikátoktat a Debreceni Egyetemen.

Horváth DezsoMTA Wigner Fizikai Kutatóközpont, Budapest

és MTA Atommagkutató Intézet, Debrecen

Új fizika jelei az LHC-nál?

A részecskefizika mindig azérdeklõdés élvonalában van,és jelentõsebb eredményeiértgyakorlatilag kijár a Nobel-díj.Ezért szinte évente felsejlik va-lamilyen új felfedezés benne,és nem okvetlenül mérési hibakövetkeztében, mint a fénynélgyorsabb neutrínók esetében.Amint korábban megírtuk, aCERN Nagy Hadronütköztetõ-jénél (1. ábra ) az ATLAS és aCMS kísérlet 2015-ben az ad-digi adatoknak ellentmondanilátszó új jelenséget látott ki-alakulni: Higgs-bozon-szerûrészecske jelét 750 GeV/c 2-estömegnek megfelelõ energiá-nál, amely azonban nem egye-zett a Higgs-bozon tulajdonsá-gaival. Az új jel a két kísérlet2016-ban gyûjtött adataiban fokozatosan elenyészett,beleolvadt a háttérbe. Ugyanakkor viszont az anti-anyag vizsgálatára szakosodott LHCb együttmûködésegész sor olyan új részecske megfigyelését jelentettebe, amelyeket több korábbi kísérlet már látni vélt,más kísérletek pedig megcáfoltak. A továbbiakbannéhány furcsa és eddig más kísérlet által sem meg-erõsített, sem pedig megcáfolt részecskefizikai megfi-gyelést írunk le.

A mindenható standard modell

A részecskefizika jelenlegi elmélete, amelyet történetiokokból standard modellnek hívunk, a világot anyagiés kölcsönhatást közvetítõ részecskék segítségévelírja le, az elõbbieket Enrico Fermi olasz fizikus nevé-

bõl fermionoknak, az utóbbiakat Satyendra Bose in-diai fizikus neve után bozonoknak hívjuk. A fizikajelenlegi állása szerint a 2. ábra valamennyi alapvetõelemi részecskét tartalmaz, mindegyiket meg is figyel-tük, utoljára, 2012-ben a Higgs-bozont. A fermionokháromfélék lehetnek: hat kvarkból állnak össze amagerõkben részt vevõ összetett részecskék, a hadro-nok, amelyek közül a legismertebbek az atommagotalkotó neutron és proton. Az ugyancsak hat leptonfele töltött részecske, közöttük a legkönnyebb az ato-mok elektronja, és mindegyikhez tartozik egy-egyelektromosan semleges neutrínó. A bozonok közül aγ-foton közvetíti az elektromágneses kölcsönhatást, a

HORVÁTH DEZSO: ÚJ FELFEDEZÉSEK A CERN NAGY HADRONÜTKÖZTETOJÉNÉL: FURCSA RÉSZECSKÉK 219

Page 6: Fizikai Szemle 68/7–8 – 2018. július–augusztusgoliat.eik.bme.hu/~f.simon/publications/Media/FizSzem-20180708.pdf · okokból standard modellnek hívunk, a világot anyagi és

8 g-gluon a magerõket és a W- és Z-bozon pedig az

3. ábra. A σ bizonytalanságú normális valószínûségeloszlás meg-bízhatósági fokozatai.

0,4

0,3

0,2

0,1

0

34,1% 34,1%

13,6% 13,6%2,1% 2,1%

0,1%0,1%

–3 –2 – m s sssss 2 3

atommagok bomlását vezérlõ gyenge kölcsönhatást.A Higgs-bozon szerepe sajátos: õ maga nem közvetítkölcsönhatást, de szerepe van az elemi részecskéktömegének létrejöttében.

A standard modellt eddig valamennyi mérési ada-tunk igazolni látszik, a segítségével végzett számítá-sok eredményét az adatok igazolják. Ugyanakkor jónéhány megfigyelésrõl, mint például a neutrínók tö-mege, a Világegyetem sötét anyaga, vagy az antianyageltûnése az Õsrobbanás után nem ad számot. Vanokunk rá, hogy egyre pontosabb részecskefizikai kí-sérletekkel tapogassuk, keressük a standard modellérvényességi határait. A Higgs-bozon felfedezése ótajelenleg ez az LHC, a CERN Nagy Hadronütköztetõjé-nek egyik fõ feladata.

Felfedezés és kizárás

A kísérleti eredményekhez matematikai bizonytalan-ságot és megbízhatóságot rendelünk, ezek jellemzikazok minõségét.

Felfedezés

A bizonytalanság sokféle forrásból tevõdik össze: azészlelt események számából ered a statisztikus, a jelmögötti zaj vagy háttér, valamint a mérõberendezésés az adatelemzéshez felhasznált információ a sziszte-matikus bizonytalanság forrása. Mindezeket némilegfélrevezetõ kifejezéssel mérési hibának is hívják,habár egyáltalán nem hiba következménye. A külön-bözõ bizonytalansági források járulékát matematikai-lag összegezve (ez egyébként igencsak bonyolulteljárás) kapjuk mérési eredményünk teljes bizonyta-lanságát, amelyet hagyományosan a szigma (σ ) görögbetûvel jelölünk. Megállapodás szerint a részecskefi-zikában akkor fedezünk fel valami újat, ha (1) az leg-alább a teljes kísérleti bizonytalanság ötszörösével,5σ -val emelkedik ki a mögötte található háttérbõl, azismert folyamatok tengerébõl, azaz megbízhatósága abizonytalanság legalább ötszöröse; (2) azt másik füg-getlen kísérlet megerõsíti és (3) az eredményt máskísérlet nem cáfolja meg. Az eddigi tapasztalat szerint,amikor az elsõ két feltétel teljesült, azt általában atöbbi kísérlet is megerõsítette.

Kizárás

A megbízhatóság matematikai fogalmára elsõsorbanakkor van szükségünk, ha nem kapunk a keresett újjelenségre utaló, meggyõzõ jelet, és azt akarjuk tud-ni, mit jelent az eredmény a feltételezett modell ér-vényességére vonatkozóan. A gyorsítós részecskefi-zikában – megállapodás szerint – azt a jelenségetzárjuk ki, amelyre a kizárás megbízhatósága (idegenszóval konfidenciája ) 95%-os, vagyis annak valószí-nûsége, hogy a jel mégis létezik, csak nem vettükészre, kevesebb, mint 5% (3. ábra ). A statisztika tu-

dománya azt mondja, hogy ha például egyszerûszámlálásnál nem észleltünk semmit, azaz 0 ese-ményt láttunk, akkor 95%-os megbízhatóság mellettcsak azt állíthatjuk, hogy az észlelt eseményekszáma 3-nál kevesebb volt.

Nagyon bonyolult feladat a szisztematikus bizony-talanság megállapítása, hiszen a különbözõ eltérésekforrásai összefüggenek, az egyik változása a többit ismegváltoztathatja, és igyekeznünk kell mindezt a leg-pontosabban figyelembe venni. Ugyanakkor a fizikusis gyarló emberi lény: vigyáznunk kell, nehogy felfe-dezési vágyunk befolyásolja adataink elemzését. En-nek elkerülésére két módszert is alkalmazunk:

• Az adatelemzést szimulációk segítségével opti-malizáljuk, publikáljuk, és jóvá hagyatjuk az egészegyüttmûködéssel, mielõtt alkalmaznánk. Tekintettelarra, hogy az adatokat sok csoport elemzi és végered-ményképpen a legjobb elemzés kerül nyilvánosságra,az elemzési módszerek a versengõ csoportok igenalapos kritikájának vannak kitéve.

• Az adatgyûjtés közben, ha már valamilyen újjelenséget sejtünk, az oda várható adatokat kizárjukaz elõzetes elemezésbõl, és egy elõre meghatározottpillanatban, rendszerint nagyobb konferencia elõttszabadítjuk fel, valamennyi hasonló elemzésen dolgo-zó csoportnak egyidejûleg, nehogy egymást befolyá-solják. Ezt a módszert vak analízisnek hívjuk és azorvostudományból kölcsönöztük.

Összefoglalva tehát: a megfigyelés nálunk 5σ több-let, a kizárás pedig 95%-os hiány. A (részecske)fiziká-nak azonban vannak területei, ahol a gyorsítós méré-seknél kevesebb az észlelés. A neutrínófizikában és azasztrofizikában 90%-os kizárást is meg szoktak adni.

Elfogadott felfedezés: a Higgs-bozon

Az 5σ többlettel kapcsolatos pozitív tapasztalat elle-nére azonban a részecskefizikában akkor fogadunk elegy felfedezést, amikor azt egy másik kísérlet hasonlómegbízhatósággal megerõsíti. A Higgs-bozon megfi-gyelését akkor merte a CERN hivatalosan bejelenteni,amikor mind az ATLAS, mind pedig a CMS kísérlet 5σeseménytöbblettel látott ugyanolyan tömegû és aHiggs-bozonhoz hasonló tulajdonságú részecskét (4.ábra ). Ugyanakkor a néhai LEP elektron-pozitron üt-köztetõ négy kísérlete közül az ALEPH csaknem meg-gyõzõen látott Higgs-bozont 10 GeV-vel kisebb tö-

220 FIZIKAI SZEMLE 2018 / 7–8

Page 7: Fizikai Szemle 68/7–8 – 2018. július–augusztusgoliat.eik.bme.hu/~f.simon/publications/Media/FizSzem-20180708.pdf · okokból standard modellnek hívunk, a világot anyagi és

megnél, mint ahol késõbb felfedezték, de a másik

4. ábra. A sok egyéb részecske mellett keletkezõ Higgs-bozon bomlása két nagy energiájúfotonra (pirossal jelölve) a CMS-detektorban.

5. ábra. Balra a hexakvark (dibarion), középen a mezon+barion molekula és jobbra a pentakvark.

három (DELPHI, L3 és OPAL) ugyanott semmi többle-tet nem észlelt; emiatt komoly viták voltak a közöscikk megfogalmazásáról. A LEP-nél több hasonló esetis volt, amikor az egyik kísérlet észlelt valami újat, dea többi nem látta: ez a statisztika gonosz tréfája a kí-sérletezõkkel.

Hasonló eset volt az is, ami a frissen észlelt Higgs-bozon tanulmányozása alatt történt 2012-ben. Az ada-tok negyede már meggyõzõen mutatta az új részecs-két, de mindkét kísérlet esetén valamivel nagyobb va-lószínûséggel jelentkezett, mint az elméleti jóslat. Az-után az ATLAS adataiban kezdetben nõtt a jel, a CMSadataiban viszont csökkent, állandóan a kísérleti bi-zonytalanság határán tartva a különbséget. Az év végé-re minden adat szépen belesimult a standard modellelszámított értékekbe. A Higgs-bozonos adatelemzésfejlõdése is érdekes volt: kezdetben a kísérletek elem-zõi a statisztikus bizonytalanságot igyekeztek csökken-teni, emiatt viszont az adatmennyiség növelésével aszisztematikus bizonytalanság nagyobb lett a statiszti-kusénál. 2013-ban mindkét kísérlet új elemzési mód-szert készített, amellyel a szisztematikus bizonytalan-ságot a statisztikus alá sikerült csökkenteni, a különb-ség az ATLAS esetében volt igazán feltûnõ. Vegyük

észre, hogy egy kísérlet adatgyûjtésétnem érdemes folytatni, amikor a sta-tisztikus bizonytalanság már a szisz-tematikus alatt van.

Összetett kvarkállapotok

A standard modell feltételezi, hogy akvarkok közötti erõs kölcsönhatástöltése három állapotú, és azt a szín-látással való kiváló analógia alapjánszíntöltésnek hívjuk. A természetcsak színtelen állapot létezését en-gedi, ezért csak a kétféle legegysze-rûbb színtelen kötött kvarkállapotlétezik: a három kvarkból álló bario-nok ([qqq], mint az [udd] neutronés az [uud] proton), illetve a kvark +antikvark kötött állapotok, a mezo-nok. Három antikvark természetesen

antibariont alkot, olyan például az [ ] antiproton.u u dElvben semmi sem gátolja háromnál több kvarkból állóhadronok kialakulását, amíg a színtelenség követelmé-nye kielégül. Ilyenek lehetnek a tetrakvark vagy di-mezon, [ ], a pentakvark [ ] és a hexa-q q q q q q q q qkvark vagy dibarion [qqqqqq] vagy [ ],q q q q q qbennük a q akármelyik kvarkot jelölheti, vegyesen.Ezek az állapotok lehetnek két hadron molekulasze-rûen kötött állapotai, vagy kvarkanyagszerû erõsenkötött állapotok. Még egzotikusabb a sok gluon kötöttállapota, a gluonlabda. Mindegyik kielégítheti a szín-telenség követelményét.

Dibarionok vagy hexakvarkok

Egy hexakvark (5. ábra, balra) biztosan létezik, adeuteron, a proton és neutron kötött állapota, és azvalóban molekulaszerû, hiszen a neutron és a protonkülön-külön van benne, viszonylag gyenge kötésben.Elméletileg megjósolták a H dibarion létezését, amely[uds] [uds] vagy [udsuds] alakú, és azt többen felfe-dezték, majd létezését még többen megcáfolták. A 80-as évek végén egy konferencián tanúja voltam, hogy adibarionokkal foglalkozó ülést a konferencia elnökea következõ mondattal jellemezte: „A dibarionok ér-

HORVÁTH DEZSO: ÚJ FELFEDEZÉSEK A CERN NAGY HADRONÜTKÖZTETOJÉNÉL: FURCSA RÉSZECSKÉK 221

Page 8: Fizikai Szemle 68/7–8 – 2018. július–augusztusgoliat.eik.bme.hu/~f.simon/publications/Media/FizSzem-20180708.pdf · okokból standard modellnek hívunk, a világot anyagi és

dekes tulajdonsága, hogy a mérések

6. ábra. Az LHCb észlelõrendszer szerelése a CERN Nagy Hadronütköztetõjénél. A kísérle-tet az antianyag hiányának tisztázására és ritka jelenségek észlelésére építették, fõlegb-kvarkot tartalmazó hadronállapotok tanulmányozásával. A képen a detektor óriási dipó-lusmágnese látszik, azt ma már eltakarja az észlelõrendszer.

7. ábra. Balra a Pc pentakvark feltételezett szerkezete, jobbra a Pc pentakvarkkeletkezése Λb

0 barion bomlásakor.

c

c–

ud

u b

u

d

u

c–

c

u

d

s

u–

{}

}Lb0 Pc

+

K–

W–

pontosításával hajlamosak eltûnni.”A dibarionokat azóta is érdeklõdésövezi: az InSpire publikációs adatbá-zis 1980 elõtt 50, 1980 és 1990 kö-zött 350, 1991 és 2000 között 260,2001 és 2010 között 120, 2011 ótapedig 120 dibarionokkal foglalkozópublikációt tart nyilván.

Pentakvarkok

Az elhíresült Θ+ = [ ] penta-u u d d skvark, az elsõ ötkvarkos állapot (5.ábra, középen és 5. ábra, jobbra)létezését szovjet elméleti fizikusokvárták (jósolták meg elméleti számí-tás alapján, ahogy mondani szoktuk)1540 MeV/c 2 tömeg környékén. Ein-stein E = mc 2 formulája alapján,amely összefüggésbe hozza az ener-giát és a tömeget, a részecskék tö-megét energiaegységekben, általá-ban millió elektronvoltban (1 eV energiát nyer egyelektron 1 V feszültség hatására), MeV-ben mérjük ésa részecske jele után írjuk. Ennek megfelelõen ez apentakvark a Θ+(1540) nevet kapta. A javaslat nagypublicitást kapott, nyolc kísérlet korábbi adatok elem-zése alapján azonnal felfedezni is vélte (a részecskefi-zika eseményeket tárol, amelyekhez késõbb visszalehet térni és újabb szempontokból elemezni), majdmég többen megcáfolták. Sokan utánaszámoltak, pél-dául Csikor Ferenc és társai (Eötvös Egyetem) számí-tásai 2006-ban a Θ+(1540) létezését kétségbe vonták.Ez a pentakvark le is került a napirendrõl.

Annál meglepõbb volt, amikor az antianyag kutatá-sára alakult LHCb együttmûködés (6. ábra ) 2015-benközölte, hogy két pentakvarkállapotot (7. ábra, balra)fedeztek fel a b-kvarkot tartalmazó Λb

0 barion bomlás-termékeiben (7. ábra, jobbra). Az LHC nagy energiájúproton-proton ütközéseiben keletkezõ b-kvarkok aprotonokból kiszabaduló u és d kvarkokkal Λb

0 =[bu d] bariont keltenek, amely nagyon sokféleképpenbomlik. A bomlástermékekbõl azonosították a J/Ψ =[ ] mezon (két Nobel-díjas felfedezõje, Ting és Rich-c cter különbözõ betûvel jelölte, ezért maradt rajta a ket-tõs szimbólum) igen jellegzetes, hegyes csúcsot adóJ/Ψ → μ+μ− müonpárra bomlását, és ennek segítségévela J/Ψ + proton energiaspektrumában megjelenõ ren-

geteg ismert Λb0 → K− + J/Ψ + p bomlás mellett, 4380 és

4450 MeV energiánál két új tömegcsúcsot figyeltekmeg. A 8. ábrán látható mért energiaspektrumban aPc pentakvark 4450 MeV energiánál keskeny, 4380MeV-nél széles csúcsot ad a standard modell általszámított, már ismert bomlások járulékai mellé. Mind-két megfigyelés megbízhatósága igen nagy, a Pc(4380)konfidenciája 9σ, a Pc(4450)-é 12σ.

Az LHCb együttmûködés a fenti megfigyelést egyolyan cikkben közölte, amelynek – a nagyenergiáskutatásokban szokásos módon – rengeteg szerzõjevan; az abc-sorrendben felírt 725 szerzõ listája Aaijnevével kezdõdik – mint például a PRL 115, 072001 is,amelybõl jó pár ábra származik – (az ATLAS-cikkekelsõ szerzõje Aad, a CMS-cikkek szerzõi az országokszerinti abc-ben szerepelnek, jelenleg az örmények-kel [Armenia] kezdve, amíg például Albánia vagy Ar-gentína nem csatlakozik).

Tetrakvarkok

2016 nyarán közölték az LHCb kísérlet másik érdekes,tetrakvarkokra vonatkozó megfigyelését. Tetrakvarkegyszerûen két mezon enyhén kötött állapota is lehet,tehát a nagy energiájú protonütközések óriási szét-szórt részecskemennyiségében szinte lehetetlen meg-

figyelni, hacsak nem tartalmaz több nehézkvarkot. Mivel a proton u és d kvarkot tar-talmaz, olyan állapotokat célszerû keres-nünk, amelyek kizárólag náluk nehezebbkvarkokból állnak. Egy ilyen állapotot mára Chicago melletti Fermilab Tevatron üt-köztetõjénél sikerült kimutatni. Ezt is a J/Ψ→ μ+μ− bomlással ismerték fel (tolvajnyel-ven azt mondjuk, címkézték ), de ezúttal aB+ = [ ] mezon bomlásakor J/Ψ = [ ],b u c cK+ = [ ] és Φ = [ ] mezonokra. A mérts u s s

222 FIZIKAI SZEMLE 2018 / 7–8

Page 9: Fizikai Szemle 68/7–8 – 2018. július–augusztusgoliat.eik.bme.hu/~f.simon/publications/Media/FizSzem-20180708.pdf · okokból standard modellnek hívunk, a világot anyagi és

energiaspektrum a J/Ψ + Φ rendszer tömegére vonat-

8. ábra. Pentakvarkok megjelenése a Λb0 barion bomlásában 4380 és 4450 MeV/c 2

tömegnél. A két pentakvark satírozott lila és kék csúcsa illesztett, a többi folyamatjáruléka szimuláció eredménye. A folyamatok összege jól lefedi a mért energia-spektrumot.

esem

énys

zám

/(15

MeV

)800

700

600

500

400

300

200

100

0

adatillesztésháttérP (4450)P (4380)

(1405)(1520)(1600)(1670)(1690)(1800)(1810)(1820)(1830)(1890)(2100)(2110)

c

c

L

L

L

L

L

L

L

L

L

L

L

L

4000 4200 4400 4600 4800 5000mJ/Yp (MeV)

9. ábra. A J/Ψ+Φ rendszer tömegére mért energiaspektrum az LHCb kísérlet adataival.Fölül a tetrakvarkállapotok feltételezése nélkül, alul azok szimulációba illesztésével.Jobb oldalt lent az adatelemzéskor figyelembe vett, ismert folyamatok láthatók, a fel-tételezett tetrakvark-állapotok járuléka satírozva.

120

100

80

60

40

20

0

jelö

lt/(1

0M

eV)

4100 4200 4300 4400 4500 4600 4700 4800mJ/ +Y F (MeV)

120

100

80

60

40

20

0

jelö

lt/(1

0M

eV)

4100 4200 4300 4400 4500 4600 4700 4800mJ/ +Y F (MeV)

adatillesztésháttér1 NRK(1 )K (1 )K(2 )+K (2 )K*(1 )K*(2 )K(0 )1 (4140)1 (4274)0 (4500)0 (4700)0 NR

+K

+

+

– –

+

+

+

+

+

+J/ +

F

Y F

X

X

X

X

kozott, és a két Tevatron-kísérlet 4140 MeV-nél látott5σ feletti konfidenciával [ccss] tetrakvarkcsúcsot. AzLHCb kísérlet ezt a megfigyelést igazolta, sõt elké-pesztõ mennyiségû adatukban még há-rom hasonló állapotot találtak.

Az LHCb kísérlet tehát 2016-ban sike-resen megfigyelt több kötött tetrakvark-állapotot. Amint azt a 9. ábra illusztrálja,a mért energiaspektrum nem igazán írha-tó le a standard modell ismert folyama-taival, de négy tetrakvark-állapot beil-lesztése a szimulációba az elméleti görbeés a mért adatok illeszkedését jelentõsenmegjavítja. A négy tetrakvarkcsúcs 4140,4274, 4500 és 4700 MeV tömegenergiánáltalálható 5σ feletti megbízhatósággal.

Gluonlabdák

Tudvalevõ, a protonok és neutronoktömegének mintegy fele gluonok formá-jában van jelen, a három alapvetõ kvarktömege (vegyértékkvarknak is hívják)néhány százalékkal járul csak hozzá, atöbbi tiszta energia. Az erõs kölcsönha-tás elmélete, a kvantum-színdinamikamegengedne tisztán gluonokból állórészecskét, azaz kötött állapotot, eztgluonlabdának (angolul glueball) hívják.A mezonok részletes tanulmányozásalehetõséget nyújt ezek megfigyelésére,mivel a kvarkok egyesülésére vonatkozószabályok csak bizonyos kvantumszá-mokat engednek meg a kvark + anti-kvark állapotokra. Anomális kvantum-számok megfigyelése esetén mezonokkötött állapotára vagy gluonlabdára isgyanakodhatunk. Elméleti fizikusok meg

vannak gyõzõdve róla, hogy az enyhénrejtélyes és emiatt alaposan tanulmányo-zott f(1710) részecske valójában tisztagluonlabda és nem csak egyszerûen me-zon gluoncsomóval vagy mezon-mezonkötött állapot.

Megmagyarázatlan és nem igazoltjelenségekCDF-esemény

A részecskefizikai kísérletek hoztak né-hány igazán elképesztõ és máig sem nemigazolt, sem nem megmagyarázott jelen-séget. Számomra a legérdekesebb az azesemény volt, amelyet a Tevatron CDFkísérlete észlelt 20 évvel ezelõtt: egy 1960GeV energiával ütközõ proton és antipro-ton teljes tartalma eltûnt és oldalra csakegy elektron, egy pozitron és két foton

repült ki viszonylag gömbszimmetrikusan, nagy ener-giával. Jó kérdés, hova lett a 3-3 kvark és antikvarkmeg a rengeteg gluon, tehát a sok színes részecske: afizika jelenlegi tudása szerint ez az esemény egysze-

HORVÁTH DEZSO: ÚJ FELFEDEZÉSEK A CERN NAGY HADRONÜTKÖZTETOJÉNÉL: FURCSA RÉSZECSKÉK 223

Page 10: Fizikai Szemle 68/7–8 – 2018. július–augusztusgoliat.eik.bme.hu/~f.simon/publications/Media/FizSzem-20180708.pdf · okokból standard modellnek hívunk, a világot anyagi és

rûen nem volt lehetséges, illetve tudományosabban

Az ILC sematikus terve. A nyalábok találkozási pontjában két detek-tort terveznek, amelyeket az ütközési pont körül ki-be csúsztatássallehet cserélni. A rendszer teljes hosszát 31 km-re tervezik.

teljes hossz 31 km(egyes részek

nem méretarányosak)

tárológyûrûk

LINAC

LINAC

e -forrás–

e -forrás+

e -csomagkompresszor

+e -csomagkompresszor

ütközési pont,detektorok

fogalmazva, nagyon-nagyon valószínûtlen volt. Elmé-leti cikkek valóságos özöne próbálta megmagyarázni,és az észlelést soha nem sikerült megismételni. ACDF-kísérletnek volt egy másik furcsa mérése is:2011-ben nagy izgalmat váltott ki egy új részecskeészlelése, amely W bozonra és két hadronzáporra(azaz kvarkpárra) bomlott. Az észlelés ellent mondotta standard modellnek, a megbízhatósága 3,2σ volt,amely a késõbbi adatok felhasználásával 4,8σ-ra nõtt.Ugyanakkor viszont a Tevatron másik fõ kísérlete, aDZero közölte, hogy nincs ott semmi, ami eltemetteaz egészet.

Negyedik neutrínó

A Los Alamosi LSND kísérlet 2001-ben elektron-anti-neutrínók megjelenését észlelte olyan folyamatban,ahol azok nem keletkezhettek. Közepes energiájú pro-tonokat lõttek réztömbbe, ahol azok pozitív és negatívπ-mezonokat, pionokat keltettek. A negatív pionokelnyelõdtek az atommagokban, a pozitívak pedig to-vábbrepülve elbomlottak müonra és müon-neutrínóraπ+ → μ+ + νμ. A gyors müon is elbomlott pozitronra,elektron-neutrínóra és müon-antineutrínóra, a végálla-potban tehát háromféle neutrínó jelenhetett csak meg,elektron-antineutrínók nem. A megjelenésüket meglehet magyarázni egy negyedik neutrínóíz létezésével,de az meg ellentmond az eddigi kísérleti tapasztalat-nak. Ráadásul az a negyedik neutrínó steril kell hogylegyen, nem lehet párja a töltött leptonok között, hi-szen az a standard modell kísérleti ellenõrzésénélrégen megjelent volna. Az észlelés megbízhatósága3,3σ volt, tehát jóval a bûvös 5σ alatt maradt. Új kísér-letet szerveztek MiniBooNE néven a jelenség ellenõr-zésére, amelyet hasonló feltételek mellett, de egynagyságrenddel nagyobb energián és neutrínóröpteté-si távolságon végeztek a Fermilabban. Az eredményigen érdekesen alakult: 2007-ben azt jelentették, hogy

nem látják a jelenséget, 2009-ben pedig azt, hogy talánmégis, 2012-ben pedig már észlelték 3,8σ többlettel,tehát még mindig a kritikus 5σ alatt. Már szervezik akövetkezõ kísérletet a dolog ellenõrzésére.

A 750 GeV-es LHC-részecske

Errõl korábban cikkeztünk, hiszen óriási izgalmatváltott ki, amikor az ATLAS és a CMS olyan Higgs-bozon jellegû részecskét észlelt az LHC-nál, amelyellentmondani látszott a standard modellnek. 2015-ben mindkét kísérlet észlelte jelét 3,4σ megbízható-sággal, de a 2016-os adatokban már nem, tehát vélet-len statisztikus ingadozás volt.

Tanulság

A részecskefizikától felfedezéseket várunk, különö-sen, mivel van jó pár rejtély, amelyet elmélete, a stan-dard modell nem képes megmagyarázni (például aneutrínóíz-rezgés, vagy a sötét anyag léte és az anti-anyag hiánya a Világegyetemben), ha a kísérletilegmért adatokat remekül le is írja. Azonban nagyon óva-tosan kell kezelnünk minden új jelenség észlelését:fontos az eredmény megfelelõ matematikai-fizikaimegbízhatósága és a másik, független mérés megerõ-sítése. Több példát is hoztunk, amelybõl kiderül, mi-lyen fontos, hogy több kísérlet mûködjék párhuzamo-san; legutóbb a gravitációs hullámok felfedezése ismegmutatta, mennyire lényeges volt, hogy az ameri-kai LIGO kísérlet két egyforma berendezést mûködte-tett egymástól 3000 km-re. A tervezés alatt álló követ-kezõ óriási részecskegyorsító, az ILC lineáris elektron-pozitron ütköztetõ nyalábjai csak egyetlen pontbanfognak találkozni, ezért a malomjáték csiki-csukimódszerének megfelelõen – hogy meglegyen az ered-mények megfelelõ ellenõrzése – két mérõberendezéstfognak felváltva ki-be tologatni az ütközési pontba.

ILC, a jövõ esetleges tiszta ütköztetõjeA nagyenergiás részecskefizika kétféle gyorsítóberendezésseldolgozik. A protonütköztetés felfedezési potenciálja két okbólóriási. Egyrészt a proton (és párja, az antiproton) stabil részecs-ke, nagy mennyiségben tárolható, és nagy tömege miatt kis su-gárzási veszteség mellett óriási energiára gyorsítható. Másrésztösszetett rendszer, ahol az alkatrészek a legkülönbözõ energiá-val ütközhetnek egymással, igen széles energiaeloszlást tapo-gatva le. Ugyanakkor, ahogy valamennyi LHC-kísérlet észlelésé-nél látjuk, az érdekes folyamat más folyamatok óriási hátterénül, tehát igen nehéz azonosítani. Ezért az új felfedezéseket cél-szerû az új jelenség keletkezési energiájára beállított elekton-pozitron ütköztetõnél részleteiben tanulmányozni. A LEP (Nagyelektron-pozitron) ütköztetõ a maximáls elérhetõ energájúelektronokat és pozitronokat ütköztetett, és – bár a Higgs-bo-zont nem szolgáltatta – valóságos kincsesbányája lett a standardmodell ellenõrzésének. A körkörös gyorsítók elektronenergiájáta szinkrotronsugárzás korlátozza, a 125 GeV/c 2 tömegû Higgs-bozon részletes tanulmányozásához tehát már egymással szem-ben épített, lineáris gorsítót kell építeni.

Ilyen terv a Nemzetközi lineáris ütköztetõ (International Li-near Collider, ILC), amelyet már viszonylag régen megterveztek,a helyszínére is több jelentkezõ van, csak a pénz gyûlik nehezenössze. Az ábrán látható a két egymással szembe kapcsolt lineárisrészecskegyorsító és az ütközési pontnál a két ki-be tologathatóészlelõrendszer. Így az egyik által esetlegesen felfedezett új je-lenség késõbb, a másik kísérlettel megerõsíthetõ vagy cáfolható.

224 FIZIKAI SZEMLE 2018 / 7–8

Page 11: Fizikai Szemle 68/7–8 – 2018. július–augusztusgoliat.eik.bme.hu/~f.simon/publications/Media/FizSzem-20180708.pdf · okokból standard modellnek hívunk, a világot anyagi és

IKARUSZ BOGLÁRKA LEPKÉK SZERKEZETI ÉS PIGMENTEREDETÛ SZÍNEINEK STRESSZÁLLÓSÁGA

A munka az OTKA K 111741 és K 115724 pályázatok, valamint azEmberi Erõforrások Minisztériuma ÚNKP-17-3-IV kódszámú ÚjNemzeti Kiválóság Programjának támogatásával készült.www.nanotechnology.hu/magyarul.html

Piszter Gábor okleveles fizikus, 2013-tól azMTA EK Mûszaki Fizikai és Anyagtudomá-nyi Intézet tudományos segédmunkatársa.Fõ kutatási tevékenysége a biológiai erede-tû fotonikus nanoarchitektúrák optikai ésszerkezeti vizsgálata, valamint ezek optikaielvû gõzérzékelõ szenzorként történõ al-kalmazása. Jelenleg az Új Nemzeti Kivá-lóság Program ösztöndíjasa.

Kertész Krisztián fizikus, az MTA EK Mû-szaki Fizikai és Anyagtudományi Intézettudományos munkatársa. 2010-ben szerzettdoktori fokozatot a Budapesti Mûszaki ésGazdaságtudományi Egyetem TTK FizikaiTudományok Doktori Iskolájában. Kezdet-ben fõként szén nanoszerkezetek kutatá-sán dolgozott, majd a biológiai és bioinspi-rált fotonikus nanoszerkezetek vizsgálatakerült érdeklõdése középpontjába, amely-ben felhasznál mikroszkópos módszereketés optikai spektrometriát.

Horváth Zsolt Endre okleveles fizikus, azMTA doktora, az MTA EK MFA tudomá-nyos tanácsadója. Fõ kutatási területe kü-lönféle nanoszerkezetek, nanostrukturáltanyagok és vékonyrétegek elektronmik-roszkópos vizsgálata, valamint vékonyréte-gek és más kristályos anyagok röntgendiff-rakciós, röntgen-reflektometriás szerkezet-vizsgálata.

Biró László Péter az MTA levelezõ tagja,Széchenyi-díjas, az MTA EK MFA kutató-professzora Kolozsváron született, egyetemitanulmányait is ott végezte. PhD fokozatot aBME-n szerzett, a Szegedi Tudományegye-tem címzetes egyetemi tanára. Meghatározószerepet játszott a nanométeres skálájúanyagtudomány hazai meghonosításában,fõ kutatási területei a pásztázószondás mik-roszkópia, a szén alapú nanoszerkezetek és2D anyagok, valamint a biológiai eredetûfotonikus nanoarchitektúrák.

Bálint Zsolt a biológiai tudományok kandi-dátusa (1997), 1983 óta a Magyar Termé-szettudományi Múzeum munkatársa, jelen-leg fõmuzeológusi beosztásban a Lepke-gyûjtemény vezetõje. Fõ érdeklõdési terü-lete a Lángszinérfélék (Lycaenidae ) sokfé-lesége, különös tekintettel taxonómiájukraés életmódjukra. Biró Lászlóval közel kétévtizede kutatja a lepkék optikájával kap-csolatos jelenségeket.

Piszter Gábor, Kertész Krisztián, Horváth Zsolt Endre, Biró László PéterMTA EK Muszaki Fizikai és Anyagtudományi Intézet

Bálint ZsoltMagyar Természettudományi Múzeum

A rovarok egyik leglátványosabb csoportja a lepkékrendje, ami évezredek óta folyamatosan az embericsodálat tárgya. Változatos színeik és mintázatukazonban nem öncélúak, hanem jól azonosítható bio-lógiai szerepük van. A lepkék szárnyának felszíni(felsõ) oldalán megjelenõ élénk színek jellemzõen aszexuális kommunikációt szolgálják, elõsegítve a vi-zuális alapon történõ párválasztást. Ettõl eltérõen, aszárnyak fonák (alsó) oldalán található bonyolult min-tázatoknak a rejtõzködésben van elengedhetetlenszerepük, így a lepkék összecsukott szárnyakkal szin-te tökéletesen képesek beolvadni a környezetükbe[1]. A két szárnyoldal eltérõ biológiai funkciójáhozkülönbözõ optikai tulajdonságokra van szükség, mi-

vel a felszínnek a figyelemfelkeltést, a fonáknak pedigaz álcázást kell megvalósítania. Ehhez a pikkelyek-nek, azaz a szárnymembránt tetõcserépszerûen borítókitinlapocskáknak kell a két oldal szerepének megfe-lelõ színképzést megvalósítaniuk. A jelenség különö-sen szépen megfigyelhetõ a Magyarországon is honosBoglárka-rokonú lepkék esetében, ahol a hímekszárnyfelszíne fémesen csillogó, élénk kék színárnya-latokban pompázik, míg a fonákoldal szürkés-barnaalapszínû, bonyolult foltmintázattal kiegészítve [2–4].Ráadásul, a két oldal színei eltérõ színképzési folya-matoknak köszönhetik a különbözõ optikai tulajdon-ságaikat. A Boglárka-rokonúakban, a fonák pikkelyei-ben általában egy széles spektrális tartományban el-nyelõ pigment, a melanin található a kitinben, amelybarna árnyalatok elõállítására képes. A szárnyfelszínélénk kék, illetve kékes árnyalatú színei ezzel szem-ben festékanyagok és fotonikus nanoszerkezetek (-ar-chitektúrák) összjátékából állnak elõ [5–7].

A szárnypikkelyek nanoarchitektúrája egy olyankompozit anyag, ami kitinbõl és levegõvel töltöttüregek periodikus, háromdimenziós rendszerébõlépül fel. Az üregek mérete és tipikus távolsága a lát-ható fény hullámhossztartományába esik, azaz aligpárszáz nanométerenként követik egymást a kitinbeágyazva. Ennek következtében a kitin-levegõ fotoni-kus nanoszerkezet úgy lép kölcsönhatásba a ráesõfénnyel, hogy a szerkezetben terjedni nem képes

PISZTER G., KERTÉSZ K., HORVÁTH ZS. E., BIRÓ L. P., BÁLINT ZS.: IKARUSZ BOGLÁRKA LEPKÉK SZÍNEINEK STRESSZÁLLÓSÁGA 225

Page 12: Fizikai Szemle 68/7–8 – 2018. július–augusztusgoliat.eik.bme.hu/~f.simon/publications/Media/FizSzem-20180708.pdf · okokból standard modellnek hívunk, a világot anyagi és

bizonyos összetevõit veri csak vissza. Ez a jelenséghasonló ahhoz, ahogyan a félvezetõ anyagokbanegyes elektronenergiákon tiltott sáv képzõdik. Azilyen jellegû szerkezeteket fotonikus kristályoknaknevezzük [1]. A továbbhaladó komponensek elnye-lõdnek a nanoarchitektúrában jelen lévõ melaninmolekuláiban, így a két folyamat összjátékának ered-ménye, hogy szemünkhöz csak a fotonikus nano-szerkezet által visszavert összetevõ jut el, amit szín-ként érzékelünk. Mivel a szerkezet méretei határoz-zák meg a terjedni képtelen fény hullámhosszát, azígy keletkezõ színt szerkezeti színnek nevezik. En-nek legegyszerûbb esete a fizikában jól ismert vé-konyréteg-interferencia. A különleges színképzéseredménye az is, hogy a fotonikus kristályokat tartal-mazó szárnypikkelyek gyakran fémesen csillognak ésiránytól függõen verik vissza a fényt.

A Boglárka-rokonúak pontmintázata fajonkéntegyedi (fajspecifikus), így az entomológusok, azaz arovarokkal foglalkozó tudósok a fonákmintázat alap-ján azonosítják a lepkék faját. Korábbi vizsgálatainksorán megmutattuk, hogy a Magyarországon honosBoglárka-rokonúak hímjei esetében a szárnyfelszínkék árnyalata is fajspecifikus, azaz a hímek faja meg-határozható pusztán a kék szín optikai vizsgálatával[3, 4]. Ennek hátterében az állhat, hogy a lepkék szín-érzékelése képes ezen árnyalatok megkülönbözteté-sére, így az evolúció évmilliói alatt a szín és a vizuálisérzékelés együttes fejlõdésével egy olyan szexuáliskommunikációs csatorna jött létre, amelyet fajtársaikazonosítására tudnak felhasználni. Ahhoz, hogy aszín alapján történõ párválasztás hatékonyan megva-lósulhasson, a szerkezeti színek nagyfokú reprodu-kálhatósága szükséges. Vizsgálataink alapján arra ju-tottunk [8], hogy a Lángszinérfélék családjának Euró-pában legelterjedtebb tagja, az Ikarusz boglárka (la-tin nevén Polyommatus icarus ) lepkefajban csak egynagyon szûk spektrális tartományba (±10 nm) esõszínek fordulnak elõ. Az ettõl eltérõ színû hímeket anõstények vélhetõen nem ismerik fel fajtársként, ígynem lesznek sikeresek a párkeresésben, emiatt gén-jeik sem öröklõdnek tovább.

Mivel a szárnyfelszín kékje nagy távolságból is ha-tékonyan mûködõ kommunikációs csatorna, szembena szárnyfonák bonyolult mintázatával, azt feltételez-zük, hogy fontos biológiai funkciója miatt a szerkezetieredetû kék színek sokkal stabilabbak a külsõ hatá-sokkal szemben, mint a pigment eredetû fonákmintá-zatéi. Ennek vizsgálatára külsõ környezeti stressznektettük ki a lepkéket a fejlõdésük során: bábállapotúpéldányokat különbözõ, 10 és 62 nap közti ideig hû-tõszekrényben, 5 °C-on tároltunk. A petékbõl kikelõhernyók növekedés és többszöri vedlés után bebábo-zódnak. A bábban megtörténik a teljes átalakulás,ennek ideje jellemzõen 7-10 nap. Ekkor formálódnaka szárnyak, az azokat borító szárnypikkelyek, vala-mint a teljes pigmentációs folyamat is ekkor zajlik le.A bábok hûtésével le tudtuk lassítani ezt a fejlõdésifolyamatot, ezzel beavatkozva a fotonikus nanoarchi-tektúrák és a pigment eredetû színek kialakulásába.

Azt tapasztaltuk, hogy hûtés hatására rajzolatbelirendellenességek (aberrációk) alakulhatnak ki a ter-mészetes körülmények között, hûtés nélkül neveltpéldányokkal szemben [9]. Ezen aberrációk mértéke,mint látni fogjuk, függ a hûtési idõtõl.

Kísérleti módszerek

Hûtési kísérleteinkhez saját nevelésû Ikarusz boglárkaegyedeket használtunk. A fajra jellemzõ az ivari kétala-kúság, azaz a hímek élénk kék szárnyfelszínûek, anõstények viszont az esetek túlnyomó többségébenbarnák. A kísérletünkben szülõ generációként alkalma-zott imágókat (kifejlett egyedeket) a csillebérci Ener-giatudományi Kutatóközponthoz (régi nevén KFKI te-lephelyhez) közeli tisztások egyikén fogtuk be. A kékhímeket és barna nõstényeket ragadozóktól védve, há-lóval fedett tápnövényen petéztettük. A kikelt hernyókis itt nevelkedtek, amíg a tápnövény elegendõ volt. Eztkövetõen begyûjtöttük a hernyókat, és egyesével Petri-csészékbe helyezve egészen a bábozódásukig folytat-tuk táplálásukat. Az egyedek, a tipikusan hajnali órák-ban lejátszódó bábozódásuk után, a hûtõszekrénybekerültek, ahol 10 és 62 nap közötti meghatározottideig +5 °C-on tartottuk õket. A hûtési idõ leteltével abábok a keltetõ edénybe kerültek, ahonnan a kikelésután az imágókat egyesével begyûjtöttük. Az imágókszárnyainak színét integrálógömbös optikai spektro-metriával vizsgáltuk. E módszerrel a szárnyról vissza-vert összes fényt detektálni tudjuk, így a szerkezetiszínek közti kicsiny eltérések is hatékonyan feltárha-tók és összehasonlíthatók. Az egyedeken nemcsak aszerkezeti színt, hanem a fonákoldal mintázatát ésannak elváltozásait is megvizsgáltuk. A rajzolat egyeselemeinek megváltozásához (aberrációjához) pontér-tékeket rendeltünk, amelyeket összevetettünk a hûtésiidõvel. Továbbá, néhány kiválasztott példány szárny-pikkelyeirõl optikai mikroszkópos, valamint pásztázóés transzmissziós elektronmikroszkópos felvételeket iskészítettünk. Ezek segítségével tártuk fel a fotonikusnanoszerkezet tulajdonságait.

A fonákoldal pigment alapú mintázata

Az Ikarusz boglárka szárnyainak fonákoldali mintázataa hímek és a nõstények esetében megegyezik. Ez azösszetett, pigmentszínekbõl álló mintázat a Boglárka-rokonúak nemzetségében tipikusan fehér gyûrûvelhatárolt fekete pöttyöket és narancsszínû foltokat tar-talmaz. Az 1. ábrán (lásd az elsõ, belsõ borítón) né-hány, a kísérlet során kikelt lepke szárnya látható. Ahûtetlen egyedeken megfigyelhetõ (1.e, g, m, o. áb-rák), hogy a fekete pöttyök a szárnyak teljes felületénmegtalálhatók; ezeket a példányokat tekintettük össze-hasonlítási alapnak. A fonákoldali mintázat elemei ahûtési idõ növelésével megváltozhatnak: elõfordul afoltok méretének csökkenése, valamint azok elhalvá-nyulása és eltûnése is. Ezeket a változásokat a továb-

226 FIZIKAI SZEMLE 2018 / 7–8

Page 13: Fizikai Szemle 68/7–8 – 2018. július–augusztusgoliat.eik.bme.hu/~f.simon/publications/Media/FizSzem-20180708.pdf · okokból standard modellnek hívunk, a világot anyagi és

biakban fonákoldali rendellenességeknek (aberrációk-

2. ábra. Ikarusz boglárka lepkék fonákoldali szárnyrajzolatának át-lagos rendellenességei (aberrációi) a hûtési idõ függvényében. Lát-ható, hogy a két mennyiség között egyenes arányosság van, azaz ahûtési idõ növelésével egyre több elváltozást tapasztaltunk a fonák-oldali mintázatban. s.e.: standard hiba, a.u.: tetszõleges egység.

16

14

12

10

8

6

4

2

0

aber

ráci

ó(a

.u.)

átlagos aberráció s.e.illesztett egyenes

hûtési idõ (nap)0 10 20 30 40 50 60

3. ábra. Különbözõ ideig hûtött Ikarusz boglárka hímek szárnyfelszínén mértvisszaverési spektrumok a kék maximumra normálva. A kinagyított részen a csú-csok maximumai láthatók.

no

rmál

t ref

lekt

ivitá

s no

rmál

t ref

lekt

ivitá

sn

orm

ált r

efle

ktiv

itás

1,0

0,9

0,8

0,7

0,6

0,5

0,4

hullámhossz (nm)

hullámhossz (nm)

900800700600500400300

vad

10 nap

31 nap

62 nap

370 380 390 400

1000

999

998

4. ábra. Ikarusz boglárka lepkeszárnyak felszíni visszaverési ma-ximumának hullámhossza a hûtési idõ függvényében. Látható, hogya 10 napos hûtés nem okozott változást a szerkezeti színben, azon-ban a hosszabb hûtési idõk esetén jelentõsen megnõtt a szórás, és akékpikkelyes nõstények is megjelentek.

vad hímhímnõstény

430

420

410

400

390

380

370

csú

csh

ullá

mh

oss

z(n

m)

0 nap

10 nap

20 nap

22 nap

31 nap

36 nap

40 nap

53 nap

62 nap

hûtési idõ (nap)

nak) nevezzük. A leghosszabb ideig, 62 napig hûtötthím és a 40 napig hûtött nõstény aberrációi az 1.f, h,illetve az 1.n, p ábrákon láthatók. Megfigyelhetõ,hogy a hímnél szinte teljesen eltûnt a fajra jellemzõmintázat, valamint a nõstényben is jelentõsen lecsök-kent a pöttyök mérete és száma is. Mind a 49 vizsgáltpéldány esetében elvégeztük a hûtéskor létrejövõaberrációk számszerûsítését úgy, hogy a mintázat ele-meinek változásához számértékeket rendeltünk: avizsgált folt kismértékû aberrációjához egy pontot, mígjelentõs változásához, illetve eltûnéséhez két pontotrendeltünk. E pontokat egyedenként összegeztük,majd az azonos ideig hûtött lepkékre átlagoltuk. Azígy kapott átlagos aberrációkat a hûtési idõ függvényé-ben ábrázoltuk (2. ábra ). Látható, hogy a szárnyak

fonákoldalán elõforduló rendellenességek átlaga egye-nes arányosságban van a hûtési idõvel, azaz minélhosszabb ideig hûtöttük a bábokat, annál nagyobb vál-tozás jelent meg a kikelt lepkék fonákmintázatában.

A szerkezeti szín változásai

A szárnyak felszínének szerkezeti színét integrálógöm-bös spektrometriával vizsgáltuk. A hûtetlen, valamint a10, 31 és 62 napig hûtött hímek átlagolt visszaverésispektrumait hasonlítottuk össze egymással (3. ábra ).A 10 napos hûtés nem volt hatással a szerkezeti színre,a visszaverési spektrum egybevág a hûtetlen egyede-ken mérttel. Viszont a 31 és 62 napig hûtött hímekben

különbözõ irányú eltérés tapasztalható acsúcs maximumának pozíciójában: a 31napos hûtés az ultraibolya-irányba tolta avisszaverési maximumot, míg a 62 napighûtött lepkékben nagyobb hullámhosszakfelé tolódott a csúcs. A jelenség elemzésé-hez minden vizsgált egyed esetében ábrá-zoltuk a csúcsok maximumának hullám-hosszát a hûtési idõ függvényében (4. áb-ra ). Az elsõ nyolc minta mindegyike álta-lunk felnevelt hûtetlen egyed volt, amelyektermészetes változatossága megfelel a ko-rábban vizsgált, természetben nevelkedettpéldányokénak [8]. Így a hûtés hatásáralétrejövõ spektrális változásokat ezzel acsoporttal hasonlítottuk össze. A 10 napigtartó hûtés nem okozott jelentõs hullám-hosszbeli eltolódást a szerkezeti színben,ahogy ez a 3. ábrán is látható. Ez össz-hangban van a fonákoldali mintázat meg-változásával is, mivel ilyen idejû hûtésnélcsak kismértékû rendellenességeket tapasz-taltunk. Azonban hosszabb ideig tartó hû-

PISZTER G., KERTÉSZ K., HORVÁTH ZS. E., BIRÓ L. P., BÁLINT ZS.: IKARUSZ BOGLÁRKA LEPKÉK SZÍNEINEK STRESSZÁLLÓSÁGA 227

Page 14: Fizikai Szemle 68/7–8 – 2018. július–augusztusgoliat.eik.bme.hu/~f.simon/publications/Media/FizSzem-20180708.pdf · okokból standard modellnek hívunk, a világot anyagi és

téskor a szerkezeti szín változása nem a fonákoldalon

6. ábra. A szárnypikkelyek fotonikus nanoszerkezeteinek pásztázó elektronmikroszkópos felvételei: Ikarusz boglárka (a) hûtetlen hím, (b)22 napig hûtött hím, (c) 20 napig hûtött nõstény.

500 nm

a) b) c)

7. ábra. A szárnypikkelyek fotonikus nanoszerkezeteinek keresztmetszeti transzmissziós elektronmikroszkópos felvételei: Ikarusz boglárka(a) hûtetlen hím, (b) 22 napig hûtött hím, (c) 20 napig hûtött nõstény. A metszetek a pikkelyek hosszanti gerinceire (6. ábra ) merõlegesenkészültek.

a) b) c)1 m�

látott lineáris módon változó aberráció–hûtésidõ függ-vényhez hasonló kapcsolatot mutatott. A mért értékekszórása a többszörösére emelkedett, függetlenül a hû-tési idõtõl, valamint az eredetileg barna nõstényekenkisebb-nagyobb mértékû kék pikkelyes tartományokalakultak ki (1.j és 1.l ábra ). Vagyis a hûtés hatására anõstények szárnypikkelyeiben is megjelent a fotonikusnanoarchitektúra. Ez a megnövekedett színbeli változa-tosság, valamint a kékes nõstények megjelenése aztmutathatja, hogy a bábok tartós hûtése formájábankeltett külsõ stressz hatására a példányok egyedi gene-tikai változatossága jelenhetett meg.

Pikkelyek optikai és elektronmikroszkópiája

Az Ikarusz boglárkák szárnyainak megváltozásait op-tikai mikroszkóppal is megvizsgáltuk. A Boglárka-rokonúak nemzetségében jellemzõen két-két pikkely-réteg fedi a szárnymembrán mindkét oldalát, amelyekközül a felszín fedõpikkelyei rendelkeznek szerkezetiszínekkel, az alappikkelyeket melanin színezi sötét-barnára. A természetes körülmények között nevelthímeknél megfigyelhetõ a rendezett kék fedõpikke-lyek rendszere, amelyek tetõcserépszerûen fekszenekegymáson (5.a ábra az elsõ, belsõ borítón). A barnaalappikkelyek alig néhány helyen lógnak ki a fedõ-pikkelyek közül, jellemzõen ott, ahol ez utóbbiakbólhiányzik egy-két darab. Ha összehasonlítjuk a 22napig hûtött hím (5.b ábra ) pikkelyeinek rendszeréta hûtetlenével, akkor határozott különbségeket ta-pasztalunk: a hûtés hatására szinte teljesen eltûntek a

rendezett pikkelysorok és -rétegek, helyüket pedigkék és barna pikkelyek rendezetlen halmaza vette át.A nõstényeknél ilyen jellegû, a rendezetlenség irányá-ba mutató folyamatot nem figyeltünk meg, viszontvalamennyi hûtött példányon feltûnõ a kék pikkelyekmegjelenése (5.d ábra ), ami nem jellemzõ a vadonbefogott barna nõstényekre (5.c ábra ).

Pásztázó elektronmikroszkópba helyezve a hûtet-len és a hûtött egyedek szárnyait, feltárhatók a fotoni-kus nanoszerkezetek részletei. A hûtetlen hím eseté-ben (6.a ábra ) megfigyelhetõ a kék színt keltõ, szi-vacsszerû fotonikus nanoszerkezet [2], ami a pikkelyteljes térfogatát kitölti. A felülnézeti képeken a 22napig hûtött hím (6.b ábra ) és a 20 napig hûtött nõs-tény (6.c ábra ) fedõpikkelyeiben is hasonló szerke-zet fedezhetõ fel. Tehát a kék nõstényekben is a foto-nikus nanoszerkezet megjelenése az, ami a szárnykékségét okozza.

Transzmissziós elektronmikroszkóppal bepillantástnyertünk a pikkelyek belsõ szerkezetébe. Ehhez 70nanométer vastag metszeteket készítettünk a fentihárom egyed szárnyaiból, és az így nyert szeleteketelektronsugárral átvilágítva képet alkottunk a fedõ-pikkelyek keresztmetszetérõl. A metszeteket a 6. áb-rán látható, a pikkelyek teljes hosszán végigfutó ge-rincekre merõlegesen készítettük. A hûtetlen (7.aábra ) és a hûtött (7.b ábra ) hímek fotonikus nano-szerkezetei között nem látható számottevõ különb-ség. A nõstényen (7.c ábra ) megjelenõ kék pikkelyekszerkezete viszont eltér a hímekétõl: kevesebb rétegûés rendezetlenebb fotonikus nanoszerkezet tölti ki apikkely térfogatát, ennek megfelelõen a visszavertszerkezeti színük is eltérõ (4. ábra ).

228 FIZIKAI SZEMLE 2018 / 7–8

Page 15: Fizikai Szemle 68/7–8 – 2018. július–augusztusgoliat.eik.bme.hu/~f.simon/publications/Media/FizSzem-20180708.pdf · okokból standard modellnek hívunk, a világot anyagi és

Következtetések

Az Ikarusz boglárka bábok hosszú idejû hûtésénekhatására az imágók szárnyainak felszínén mind aszerkezeti szín, mind a pigment alapú fonákmintázatmegváltozott, azonban a különbözõ színképzési fo-lyamatok eltérõ mértékû és irányú változásokat mu-tattak. A fonákoldal festékanyagokon alapuló mintá-zatában a hûtési idõvel egyenesen arányos rendelle-nességek jöttek létre: a hûtési idõ növelésével a szár-nyak egyre nagyobb részén tapasztaltunk megválto-zó vagy eltûnõ jellegzetességeket. Ezzel szemben afelszín szerkezeti színbeli változásainak karaktere,azaz a hûtési idõ hatására megnövekedett színbeliváltozatossága azt mutatta, hogy a kék szexuális jel-zõszín esetében kismértékû és egyedi spektrális vál-tozások jelennek meg a bábok tartós hûtésével. Ezkapcsolatban lehet a lepkék rejtett genetikai variá-cióival, amelyeket a környezeti stressz (tartós hûtés)aktivált a vizsgált egyedekben. Az irodalomban be-számolnak hasonló, hõsokk okozta megváltozásról,továbbá a 20, illetve 40 napig hûtött nõstények kékszínének megjelenése (1.j és 1.l ábra ) is ezt a feltéte-lezést támasztja alá.

A Kárpát-medencében, illetve az elterjedési terüle-tük középsõ részén az Ikarusz boglárka nõstényei-nek szárnyfelszínén a barna szín dominál. Viszont afaj elterjedési területének szélein a nõstényekre is akék szín jellemzõ, amit a fedõpikkelyeikben kialaku-ló, a hímekéhez hasonló, szivacsszerû fotonikus na-noszerkezet állít elõ. Ennek egy lehetséges magyará-zata, hogy az elterjedési terület szélén a populációsûrûsége alacsony, így a megszokott párkeresési stra-tégia nehezebben valósítható meg. Viszont, ha a nõs-tények is „felveszik” a fajra jellemzõ, fajspecifikusszerkezeti színt, akkor a feltûnõ, messzirõl is követ-hetõ szexuális jelzõszín használatával genetikai állo-mányuk továbbörökítésének esélyét jelentõsen meg-növelik.

Összegezve, az Ikarusz boglárka bábállapotbantörténõ, ellenõrzött hûtésével kétféle elváltozást hoz-tunk létre: (i) A pigmentek által színezett fonákoldalimintázat rendellenessége, ami egyenesen arányos ahûtési idõtartammal, és kisebb egyedi eltérésektõl el-tekintve minden példánynál nagyon hasonló jellegûmegváltozásokat tapasztaltunk. (ii) A szárnyak felszí-nén lévõ szerkezeti színek megváltozása, amelyugyan kismértékben függött a hûtési idõtõl, de azegyedi variációk megjelenése sokkal hangsúlyosabbvolt. Valószínûsíthetõ, hogy a bábok tartós hûtésévela faj rejtett genetikai variációit aktiváltuk, amelyek atermészetben csak szélsõséges körülmények között,például az elterjedési területük szélein jelennek meg.Érdemes kiemelni, hogy a szexuális jelzõszín stabilitá-sának – a párválasztásban játszott szerepe miatt – na-gyobb a jelentõsége a leszármazottak genetikai állo-mányának meghatározásában, mint a rejtõzködésbenszerepet játszó pigment alapú mintázatnak.

Irodalom1. Márk G. I., Bálint Zs., Kertész K., Vértesy Z., Biró L. P.: A bioló-

giai eredetû fotonikus kristályok csodái. Fizikai Szemle 57/4(2007) 116–121.

2. Bálint Zs., Biró L. P.: A lepkék színeváltozása. Természet Világa135/7 (2004) 310–313.

3. Kertész K., Piszter G., Vértesy Z., Biró L. P., Bálint Zs.: Színekharmóniája: a boglárkalepkék szerkezeti kék színének fajfelis-merési szerepe – I. rész. Fizikai Szemle 63/7–8 (2013) 231–234.

4. Kertész K., Piszter G., Vértesy Z., Biró L. P., Bálint Zs.: Színekharmóniája: a boglárkalepkék szerkezeti kék színének fajfelis-merési szerepe – II. rész. Fizikai Szemle 63/9 (2013) 293–298.

5. Rajkovits Zs., Illy J.: Az élõ természet színei. Fizikai Szemle 51/3(2001) 76–79.

6. Rajkovits Zs.: Szerkezeti színek az élõvilágban. Fizikai Szemle57/4 (2007) 121–126.

7. Bálint Zs., Biró L. P.: A lepkeszárny kémiai és fizikai színei. Ter-mészet Világa 140/11 (2009) 486–489.

8. Piszter G., Kertész K., Bálint Z., Biró L. P.: Variability of thestructural coloration in two butterfly species with different pre-zygotic mating strategies. PLoS ONE 11 (2016) e0165857.

9. Kertész K., Piszter G., Horváth Z. E., Bálint Z., Biró L. P.: Changesin structural and pigmentary colours in response to cold stress inPolyommatus icarus butterflies. Scientific Reports 7 (2017) 1118.

Most Társulatunknak lenne szüksége

egyletmentõ ötletekre!

http://forum.elft.hu

Ezek az ötletek nem vesznek el,

ha a

linken, az ELFT stratégiai vitafórumán adjuk elõ.

Jobb egy mentõötlet mint öt mentõ egylet

– írta Karinthy Frigyes az egyletistápolás margójára.

PISZTER G., KERTÉSZ K., HORVÁTH ZS. E., BIRÓ L. P., BÁLINT ZS.: IKARUSZ BOGLÁRKA LEPKÉK SZÍNEINEK STRESSZÁLLÓSÁGA 229

Page 16: Fizikai Szemle 68/7–8 – 2018. július–augusztusgoliat.eik.bme.hu/~f.simon/publications/Media/FizSzem-20180708.pdf · okokból standard modellnek hívunk, a világot anyagi és

ALACSONY HÁTTERÛ MAGFIZIKAI MÉRÉSEK, AVAGY

1. ábra. A magpotenciál és a Coulomb-potenciál összegeként azatommag határán egy potenciálgát képzõdik. Ez nehezíti a fúziólétrejöttét (fölül). A csillagokban található részecskék sebességel-oszlásából és a hatáskeresztmetszet szorzatából kialakuló Gamow-ablak, ami az asztrofizikailag jelentõs energiatartomány (alul).

po

ten

ciál

isen

ergi

a

atommag

nagyenergiás bejövõ részecske

kisenergiás bejövõ részecske

sugárre

latív

való

szín

ûsé

g

Maxwell–Boltzmann-eloszlás

alagúteffektus valószínûsége

Gamow-ablak

k TB E0 energia

A NUKLEÁRIS ASZTROFIZIKA KIHÍVÁSAI

Szücs Tamás 2008-ban szerzett fizikus ésfizikatanári diplomát az ELTE-n. 2012-ben aDebreceni Egyetemen védte meg PhD disz-szertációját alacsony hatáskeresztmetszetekmérési módszereibõl. Két évet töltött kuta-tóként a drezdai HZDR kutatóintézetben,ott egy új föld alatti gyorsítólaboratóriumkialakításában vállalt meghatározó szerepet,és ahol idén vezetésével megkezdõdnek azelsõ kísérletek az új föld alatti gyorsítólabo-ratóriumban. 2016–2018-ig MTA posztdok-tori ösztöndíjas az MTA Atomkiban.

Szücs TamásMTA Atommagkutató Intézet és

Helmhotz-Zentrum Dresden-Rossendorf, Németország

Világegyetemünk anyagának legnagyobb (98,5%-nyi)része még mindig az õsrobbanáskori nukleoszintézissorán keletkezett hidrogén és hélium. A legtöbbegyéb elem magreakciók útján, a csillagok különbözõfejlõdési szakaszaiban jött létre [1]. A könnyebb ele-mek fúziója egészen a vascsoportig képes felépíteni akémiai elemeket, miközben a reakciókban energiaszabadul fel [2]. Ezen energia kisugárzása tartjaegyensúlyban a csillagokat a gravitációval szemben. Avasnál nehezebb elemek létrehozásához már energia-befektetés szükséges. A csillagok statikus fázisában iselõfordulhatnak ilyen endoterm reakciók, például azs-folyamat, de a csillagélet végi robbanásban is szá-mottevõ nehezebb elem keletkezik az r- és p-folya-matokban [3–5]. A periódusos rendszer egészen aFöldön megtalálható legnehezebb természetes ele-mig, az uránig népesül be. A nehéz elemek hasításá-val kinyerhetjük az azokat létrehozó robbanás ener-giájának egy részét.

A nukleáris asztrofizika egyik fõ feladata az eleme-ket felépítõ magreakciók pontos megismerése. Ezekfontosak mind a csillagok életének és a Világegyetemfejlõdésének megértéséhez, mind a világunkat alkotóelemek létrejöttének feltérképezéséhez.

Magreakciók a csillagokban

A csillagokban lejátszódó magreakciók magfizikaiértelemben meglepõen alacsony hõmérsékleten zajla-nak. A hõmozgást végzõ részecskék átlagos energiájaa néhány tízmilliótól néhány milliárd kelvin hõmér-sékleteken 1–100 keV.1 A magfizikai energiaskálát a

1 1 eV az az energia, amire egy elemi töltés szert tesz 1 V gyorsítófeszültség hatására (1,6 10−19 J).

részecskék tipikus kötési energiája jelöli ki, ami ahõmozgásnál nagyságrendekkel magasabb 1–10 MeV-es energiatartomány.

Töltött részecskék közötti reakció esetén az elekt-rosztatikus taszítás is nagy szerephez jut, mivel a fu-zionáló atommagok mindegyike pozitív töltéssel bír.A Coulomb-taszításnál erõsebb magerõk csak rövid

hatótávolságúak, a fúzió létrejöttéhez az atommagok-nak kellõ közelségbe kell kerülniük. Az úgynevezettmagpotenciálon kívül csak a taszítás érvényesül. Afúzió létrejöttéhez a magoknak át kell jutniuk a vonzóés taszító potenciálból kialakuló Coulomb-gáton (1.ábra, fölül).

A gát magassága egyenesen arányos a résztvevõmagok töltésével, két proton esetén értéke körülbe-lül 600 keV. Visszatérve a csillagokban uralkodó hõ-mérsékletekre, látható, hogy az asztrofizikailag jelen-tõs magreakciók mélyen a Coulomb-gát alatt zajla-

230 FIZIKAI SZEMLE 2018 / 7–8

Page 17: Fizikai Szemle 68/7–8 – 2018. július–augusztusgoliat.eik.bme.hu/~f.simon/publications/Media/FizSzem-20180708.pdf · okokból standard modellnek hívunk, a világot anyagi és

nak, a kvantummechanikai alagúteffektus nélkül lét-re sem jönnének, tehát a Nap sugárzása az alagútef-fektus egyik közvetett bizonyítéka. A reakciók ha-táskeresztmetszete2 az alagúteffektus valószínûségé-

2 A hatáskeresztmetszetet szokásosan σ-val jelöljük. A reakciólezajlási valószínûségével arányos, felület dimenziójú mennyiség,általánosan használt mértékegysége a barn, ami 10−24 cm2-relegyenlõ.

vel arányos, így a csökkenõ energiával exponenciá-lisan csökken.

Az asztrofizikában fontos mennyiség a reakcióse-besség vagy reakcióráta, ami az egységnyi idõ alattlejátszódó magreakciók száma. Ez a mennyiség ará-nyos a sebességre átlagolt hatáskeresztmetszettel. Acsillagokban a részecskék sebességeloszlása Maxvell–Boltzmann-eloszlást követ, amit az exponenciálisancsökkenõ hatáskeresztmetszettel kell átlagolni. A ket-tõ szorzata egy jól meghatározott energiatartománytjelöl ki, az úgynevezett Gamow-ablakot (1. ábra,alul). Ez az az energiatartomány, ahol a hatáskereszt-metszet ismerete szükséges a csillagbeli reakciórátaszámolásához. A Napunkban például a két protonfúziója esetében a Gamow-ablak középpontja körül-belül 5,9 keV, ami két nagyságrenddel van a Cou-lomb-gát alatt.

A csillagok statikus fázisaiban ez az arány hason-lóan alakul. A Gamow-ablak energiája hiába növek-szik a magasabb hõmérséklettel és tömegszámmal, aCoulomb-gát a reakciópartnerek rendszámának növe-kedésével szintén nõ. Mindez nagyon alacsony hatás-keresztmetszeteket jelent nano- és pikobarn nagyság-rendben, míg a tipikus magfizikai hatáskeresztmetsze-tek millibarn és barn között vannak. A nukleáris aszt-rofizika nagy kihívása éppen ezen nagyon kis hatás-keresztmetszetek meghatározása, a nagyon apró jelin-tenzitások mérése.

Magreakciók a laboratóriumban

E cikkben elsõsorban a kisebb tömegû magok fúziósreakcióinak vizsgálatával foglalkozom, proton-, illetvealfa-befogási reakciókról lesz szó. A magreakcióklétrehozásához részecskegyorsítókat használunk. Alaboratóriumbeli reakcióráta a céltárgy atommagjai-nak száma szorozva a bombázó részecskék áramávalés a hatáskeresztmetszettel. Tipikus kísérleti körülmé-nyek között ez maximum pár reakció óránként, desok estben ennél is jóval kevesebb. Nukleáris asztrofi-zikai kutatásokban – az alacsony hatáskeresztmetsze-tek miatt – nem a gyorsító maximális energiája szabhatárt a kísérleteknek, hiszen már sokszor a néhányMeV reakcióenergia is a Gamow-ablak felett van. Sok-kal fontosabb a stabil, nagy áramú nyaláb és a stabilcéltárgy. A gyorsítótechnika fejlõdésével manapságakár néhány milliamperes proton- vagy alfa-nyaláb iselõállítható, a következõ kihívás olyan céltárgyakelõállítása, amelyek ezt a nyalábintenzitást károsodásnélkül bírják ki. Napjainkban – a céltárgy tönkretétele

nélkül – maximálisan néhány 100 μA-es nyalábokattudunk használni. A másik maximalizálandó paramé-ter a céltárgymagok száma. Az ionnyaláb fékezõdik acéltárgyban, annak mélyén már más energiával lépreakcióba, mint a felszínen. Mivel a hatáskeresztmet-szet az általunk használt energiákon exponenciálisancsökken, a mélyebb rétegekben már nem történikszámottevõ mennyiségû reakció. Így az úgynevezetteffektív céltárgymagszám vastagabb céltárgy eseténsem változik.

Miután a nyalábáramot és az effektív céltárgyma-gok számát maximalizáltuk, megbecsülhetjük, mi az alegkisebb hatáskeresztmetszet, amit mérhetünk. Ha areakció hatáskeresztmetszete a Gamow-ablakbanmérhetetlen, akkor extrapolációt kell alkalmaznunk.Az extrapoláció annál pontosabb, minél inkább meg-közelítjük a releváns energiatartományt, tehát a legki-sebb, még mérhetõ jelintenzitásokat kell elérnünk.

A hatáskeresztmetszet mérése

A magreakciók létrejöttét legtöbbször a keletkezettreakciótermék által kibocsátott részecske vagygamma-sugárzás detektálásával határozzuk meg. Írá-somban csak gamma-detektálással foglalkozom nagy-tisztaságú germánium (HPGe) detektorokkal. Ezenbelül is az úgynevezett promptgamma-módszerrel,ahol a reakcióban keletkezett, többnyire magasangerjesztett végmag legerjesztõdése közben kibocsátottgamma-sugárzást detektáljuk.

Egy adott átmenetbõl származó gamma-sugárzásjól meghatározott energiájú, így az egy csúcsot alkot aspektrumban. E csúcs területe arányos a kibocsátottgamma-sugarak számával, így a reakció-hatáskereszt-metszettel is.

A gamma-sugárzás kibocsátása statisztikus folya-mat, így a csúcsterület beütésszáma Poisson-eloszlástkövet, bizonytalansága az értékének négyzetgyöke.Ebbõl következik, hogy alacsony beütésszámbólcsak nagy bizonytalansággal következtethetünk ahatáskeresztmetszetre. A mai precíziós asztrofizikaimérések mellett, egy-egy csillagfejlõdési fázis pontosismeretéhez a magfizikai reakcióráták nagy pontos-ságú ismerete szükséges, amihez egy-egy hatáske-resztmetszetet legfeljebb 10%-os teljes bizonytalan-sággal kell ismerni. Minden kísérlet szisztematikushibákkal is terhelt, így többnyire néhány százalékosstatisztikus bizonytalanságot kell elérnünk. A példakedvéért, ha 3%-os bizonytalansággal szeretnénkmérni 1 nanobarn hatáskeresztmetszetet, tipikus kí-sérleti körülmények között óránként várhatunk egybeütést.3 A szükséges 1111 beütéshez 1,5 hónap nya-

3 σ = 1−33 cm2; effektív céltárgymagszám: 1018 1/cm2; bombázóré-szecske-fluxus: 1017/h; 1% detektálási hatásfok.

lábidõre van szükség. Ez alatt az idõ alatt mind adetektoroknak, mind a gyorsítónak stabilan kell mû-ködnie, és a céltárgyunknak is ki kell bírnia az ion-bombázást. A helyzet még rosszabb, ha a detekto-

SZÜCS TAMÁS: ALACSONY HÁTTERU MAGFIZIKAI MÉRÉSEK, AVAGY A NUKLEÁRIS ASZTROFIZIKA KIHÍVÁSAI 231

Page 18: Fizikai Szemle 68/7–8 – 2018. július–augusztusgoliat.eik.bme.hu/~f.simon/publications/Media/FizSzem-20180708.pdf · okokból standard modellnek hívunk, a világot anyagi és

runkban nem csak a kívánt reakcióból származnak

2. ábra. Egy nagytisztaságú germániumdetektorban látható tipikusjel, adott energiájú gamma-sugárzás esetén.

teljesenergia-csúcs

kétszereskiszökési

csúcs

egyszereskiszökési

csúcs

511 keV511 keV

Compton-tartomány

beü

téss

zám

(lo

gari

tmik

us

skál

a)

energia

3. ábra. Különbözõ árnyékolások hatása különbözõ HPGe detektorokban észlelt laboratóriumi háttérre. A szürke spektrum mindig azárnyékolás nélküli hátteret mutatja. Balra 10 cm ólomárnyékolás, középen aktív árnyékolás, jobbra mély föld alatti elhelyezés és 25 cmólomárnyékolás hatása a laboratóriumi háttérre.

0 2 4 6 8E (MeV)

20 4 6 8 10 12 14E (MeV)

20 4 6 8 10 12 14E (MeV)

10

10

10

10

1

10

10

10

10

10

10

10

4

3

2

–1

–2

–3

–4

–5

–6

–7

beü

téss

zám

(keV

h)

–1–1

beütések, hanem a spektrumcsúcs alatt háttérbeüté-sek is találhatók. Ilyenkor a csúcsterület bizonytalan-sága a háttér beütésszámától is függ, megnövelveezzel a kívánt statisztikus bizonytalanság eléréséhezszükséges idõt. Ha a jel mellett feleannyi háttérbe-ütés is van, akkor a mérési idõ körülbelül duplájáranövekszik.

A csúcs alatti háttér általában elkerülhetetlen, ezrészben egyszerûen a detektorok fizikájának követ-kezménye. Ha csak egyféle energiájú gamma-sugár-zás érkezik, akkor sem csak egy csúcsot látunk aspektrumban, hanem kisebb energiájú beütéseket is(2. ábra ). Ez a Compton-tartomány, amelynek beüté-sei akkor keletkeznek, ha a sugárzás kiszóródik adetektor aktív térfogatából, és nem adja le a teljesenergiáját. Ha már két gamma-sugarunk van, akkortipikusan az egyik a másik által keltett Compton-tarto-mányon ül, így alatta háttérbeütések vannak. Ezenkívül az úgynevezett kiszökési csúcsok is gondotokozhatnak, ha energiájuk pont megegyezik a vizs-gálni kívánt gamma-sugárzás energiájával.

A HPGe detektorok elõnye a nagyon jó energiafel-oldás. A gamma-átmenetek energiája pontosan meg-határozható, illetve a különbözõ gamma-sugárzásokjól elkülöníthetõk. Ez sokszor fontos, mert a reakció-termék általában több egymást követõ gamma-átme-nettel gerjesztõdik le, így számos, különbözõ energiá-jú gamma-sugárzást kell detektálnunk. Hátránya aviszonylag alacsony detektálási hatásfok, ami tipikuskísérleti elrendezésben 0,1–1%.

A detektorokban észlelt háttér

A detektor nemcsak az általunk keltett magreakció-ban keletkezett gamma-sugárzást észleli, hanem atermészetben, a környezetünkben elõforduló radio-aktív bomlások gamma-sugarait is. Ezek a mindenholjelen lévõ sugárzások nehezítik méréseinket, merthátteret okoznak a detektorban. A problémát általá-

ban nem a laboratóriumi háttér csúcsai adják – hiszenazok tipikusan elválaszthatók a reakció gamma-csú-csaitól –, hanem a sok összeadódó Compton-tarto-mány, amin a mérendõ jelek ülnek. Ha túl nagy a hát-tér, a jel mérhetetlen. A környezeti radioaktivitás ma-ximálisan 3 MeV-ig ad jeleket a detektorban, de ennélmagasabb energiákon is észlel hátteret a detektor (3.ábra ). Magasabb energiákon már nincsenek csúcsok,csak az energiával alig változó, közel állandó beütés-szám, amit a kozmikus háttérsugárzás okoz. A felsõlégkörbe belépõ kozmikus töltött részecskék többekközött müonokat keltenek, amelyek elérik a földfel-színt. Ezek a töltött részecskék a detektoron keresz-tülhaladva keltik a nagyenergiás jeleket egészen akár40-50 MeV-ig.

A kísérletek szempontjából lényeges háttértarto-mány 15 MeV-ig tart. A releváns reakciókban általá-ban maximum ekkora energiájú gamma-sugárzást kel-tünk és detektálunk.

Különbözõ eredetük miatt különbözõképpen kellkezelnünk a két jól elválasztható háttértartományt. Ecikkben kisenergiás háttérrõl 3 MeV alatt, nagyener-giás háttérrõl 3 MeV feletti jelek esetén beszélek.

232 FIZIKAI SZEMLE 2018 / 7–8

Page 19: Fizikai Szemle 68/7–8 – 2018. július–augusztusgoliat.eik.bme.hu/~f.simon/publications/Media/FizSzem-20180708.pdf · okokból standard modellnek hívunk, a világot anyagi és

A detektor feloldása, azaz a gamma-csúcsok félér-

4. ábra. A kozmikus sugárzás különbözõ komponenseinek mély-ségfüggése szikla esetén. A vízszintes vonallal jelzett neutronkom-ponens a laboratórium falait alkotó anyagokból származik, emiattszintje különbözõ helyszíneken más és más. Itt egy tipikus értéketábrázolok, annak jelzésére, milyen mélységtõl nem csökken továbba neutronháttér.

inte

nzi

tás

(cm

s)

–2–1

10

10

10

10

10

10

10

–1

–2

–3

–4

–5

–6

–7

10–110–2 1031 10 102

mélység (m)

müon

kozmikussugárzás neutronja

müonok keltetteneutron

hasadási és ( ,n) neutrona

tékszélessége az energia négyzetgyökével arányosannõ. Ha meg akarjuk becsülni a csúcs alatti háttérbe-ütések számát, akkor a 3. ábra számait a csúcsoktalpszélességével kell megszoroznunk. Így árnyékolásnélkül a nagyenergiás háttér 1-2 beütés óránként, míga kisenergiás háttérben 10-1000 beütést várhatunkugyanennyi idõ alatt.

Ezeket a háttereket mindenképpen csökkentenünkkell, ha a nukleáris asztrofizika nagyon kis jelintenzi-tásait akarjuk mérni.

A laboratóriumi háttér csökkentésénekmódszerei

A háttér csökkentésének legegyszerûbb és széles kör-ben alkalmazott módszere, a passzív árnyékolás (3.ábrán balra). Ilyenkor a detektort tipikusan ólomár-nyékolással vesszük körbe. A detektorba jutó 3 MeValatti gamma-sugarak mennyiségét 10 cm vastagólomfal körülbelül századára csökkenti. A nagyener-giás müonok viszont gond nélkül keresztülhaladnakezen árnyékoláson, így a 3 MeV feletti hátteret az aligcsökkenti. Ráadásul a müonok fékezési sugárzást ésmásodlagos részecskéket keltenek a passzív árnyéko-lásban, ami a detektorba jutva a laboratóriumi hátteretnöveli. Ezen másodlagos sugárzás miatt a földfelszí-nen nincs értelme 10-15 cm-nél vastagabb ólomárnyé-kolásnak, a vastagabb árnyékolás nem csökkenti to-vább a hátteret.

A müonok ellen hatékonyabb védekezés az úgyne-vezett aktív árnyékolás (3. ábrán középen). Ekkor akísérletet végzõ detektort egy másik is kíséri. A má-sodlagos detektor körbeveheti az elsõt, esetleg egy-szerûen alatta vagy felette helyezkedik el. Ilyenkor akeresztülhaladó müon mindkét detektorban jelet kelt.A másodlagos detektor jeleit vétóként felhasználva, azelsõdleges detektor müonjelei kiszûrhetõk. Egy gyû-

rûs elrendezés tipikusan egy-két nagyságrenddelcsökkenti a nagyenergiás hátteret, és másodlagos ef-fektusként a Compton-tartományt is.

A kettõ kombinálása nehézkes, ezért tipikusanvagy az egyiket vagy a másikat alkalmazzuk, attól füg-gõen, hogy milyen gamma-energia-tartományban vár-juk jeleinket.

A nagyenergiás müonok nagy áthatoló képessé-gûek, emiatt az általunk felépített passzív árnyékolásalig csökkenti intenzitásukat. Viszont nem feltétlenülnekünk kell árnyékolást építenünk, hanem megvizs-gálhatjuk, mi történik, ha a kísérleteinket a föld alattvégezzük így nyerve árnyékolást.

A 4. ábrán a detektorok nagyenergiás hátteréhezhozzájáruló események intenzitásának mélységfüggé-se látható. Ami szembetûnõ, hogy még 300 méter mé-lyen is a müonok alkotják a legfõbb háttérkompo-nenst, igaz intenzitásuk már ötezred részre csökkent.Fontos észben tartanunk, hogy a kísérletekhez gyorsí-tó szükséges, így nem elég csak a detektort föld alávinnünk, hanem a gyorsítót is oda kell telepítenünk.Természetesen létezik gyorsító által keltett, úgyneve-zett nyalábindukált háttér is, de annak kezelési tech-nikáira helyhiány miatt ez az írás nem tér ki.

Ha még mélyebbre megyünk, a csökkenés nemáll meg, és 1400 m mélyre már csak a felszíni müon-fluxus milliomod részé jut le. Ez az a mélység, ahovaa világon elõször nukleáris asztrofizikai célra gyorsí-tót telepítettek (Gran Sasso Nemzeti Laboratórium,Olaszország). Itt a LUNA (Laboratory for Under-ground Nuclear Astrophysics) kollaboráció immár25 éve az élen jár a nagyon apró jelintenzitások mé-résében.

Az elhanyagolhatóvá váló kozmikus sugárzás direkthatásának kiküszöbölése mellett a másodlagos sugár-zás keltése is lecsökken, ezáltal a detektort vastagabbpasszív árnyékolással lehet körülvenni, így a 3 MeValatti háttértartomány is jobban csökkenthetõ.

A 3. ábrán jobbra nagyon jól látszik, hogy a 3 MeValatti háttértartományban három nagyságrend a csök-kenés, míg a nagyenergiás háttér akár tízezred részéreis csökken, sõt 10 MeV felett háttérmentes tartománylátható. Ezen ultra alacsony háttérnek köszönhetõen aLUNA kollaboráció számos magreakció hatáskereszt-metszetét mérte meg a reakciók Gamow-ablakában.

Többek között a 3He(α,γ)7Be reakció vált mérhetõ-vé az õsrobbanáskori nukleoszintézis energiáin, ami-vel kizárhatóvá vált a primordiális lítiumproblémamagfizikai megoldása. A 93 keV-os reakcióenergián a0,2 nb hatáskeresztmetszetet körülbelül egy hónapnyalábidõ alatt mérték meg.

Késõbb, a 14N(p,γ)15O magreakció hatáskereszt-metszetének minden eddiginél alacsonyabb energiá-kon való mérésével, a korábbi extrapolációk ponto-sodtak, és a csillagokbeli becsült reakcióráta a koráb-biak felének adódott. A CNO ciklus ezen leglassúbbreakciója határozza meg a ciklus hosszát, így a nehe-zebb csillagok hidrogénégetõ fázisának életidejét. Aráta csökkenésébõl egyenesen következik a gömbhal-mazok életkorának növekedése.

SZÜCS TAMÁS: ALACSONY HÁTTERU MAGFIZIKAI MÉRÉSEK, AVAGY A NUKLEÁRIS ASZTROFIZIKA KIHÍVÁSAI 233

Page 20: Fizikai Szemle 68/7–8 – 2018. július–augusztusgoliat.eik.bme.hu/~f.simon/publications/Media/FizSzem-20180708.pdf · okokból standard modellnek hívunk, a világot anyagi és

A LUNA kollaboráció ezeken kívül a hidrogénfúzió

5. ábra. Földfelszínen, kis mélységben és mélyen a föld alatt ugyanazon detektor a laboratóriumi háttere aktív árnyékolással és anélkül.45 m mélyen aktív árnyékolással elért nagyenergiás háttér csak kevéssel több, mint mélyen a föld alatt.

földfelszínen

földfelszínen, aktív árnyékolással

45 m mélyen

45 m mélyen, aktív árnyékolással

1400 m mélyen

1400 m mélyen, aktív árnyékolással

0 1 2 3 4 5 6 7 8 9 10 11 12 13 14 15Eg (MeV)

beü

téss

zám

(keV

h)

–1–1

10

10

10

1

10

10

10

10

10

3

2

–1

–2

–3

–4

–5

számos magreakciójának paraméterét mérte meg ko-rábban elképzelhetetlenül alacsony energiákon, sokatpontosítva a különbözõ csillagfázisok megértésén.Legutóbbi összefoglaló közleménynek lásd [6].

Újabb lehetõségek

A LUNA együttmûködés nagy sikerének köszönhetõenszámos újabb helyszín merült fel föld alatti gyorsítóktelepítéséhez. Ennek létrejöttét a tudományos közös-ség több okból is sürgeti. Egyrészt a LUNA jelenlegi400 kV terminálfeszültségû gyorsítója nem tud elegen-dõ energiájú nyalábot szolgáltatni a héliumfúzió reak-cióinak megfelelõ energiatartományban való vizsgála-tához, amelyekhez pár MV-os gyorsítás szükséges.Másrészt – az egy-egy kísérletre fordítandó 2-3 évesmérési idõszak miatt –, ha párhuzamosan több vizsgá-latot tudnánk végezni, lecsökkenthetnénk az egy-egyfontos reakció pontosításához szükséges idõt.

A világban számos mély föld alatti helyszín van,viszont ezek nagy többsége nehezen megközelíthetõ,emiatt egy gyorsítólaboratórium kialakítása nehézkes.Ezért merült fel, hogy ha mélyen a föld alá nem me-hetünk, akkor aktívan árnyékolt detektorral próbál-junk megfelelõ hátteret elérni, hiszen – mint azt ko-rábban láthattuk – aktív árnyékolással a földfelszínenmár egy-két nagyságrendet csökkenteni tudtunk anagyenergiás háttérbõl.

Elõször az Atomki aktív árnyékolással rendelkezõdetektorát vittük a mélyen föld alatti LUNA laborató-riumba. A mérések a várt eredményt hozták. Aholmár elhanyagolható a müonok mennyisége, az aktívárnyékolás nem csökkent tovább a nagyenergiás hát-téren [7].

Következõ állomásunk egy kis mélységû pince-rendszer, a drezdai Felsenkeller volt. A kilenc alagút-ból az egyikben már elhelyezkedik egy föld alatti ala-csony hátterû laboratórium, ahol kis aktivitású mintá-kat mérnek. Ide vittük el ugyanazt az aktívan árnyé-

kolt detektort, amivel korábban a LUNA-ban mértünk.A 45 m mély szikla önmagában 40-ed részére csök-kenti a müonhátteret, és közben elhanyagolhatóváválik a kozmikus sugárzás neutronkomponense (4.ábra ). Így árnyékolatlan detektorban, 3 MeV felettienergiatartományban a háttér kicsivel alacsonyabb,mint aktívan árnyékolva a földfelszínen. Az aktív ár-nyékolás a mérések alapján hasonló mértékben csök-kenti a hátteret, mint a földfelszínen. Végeredmény-ben az aktív árnyékolásnak köszönhetõen a háttérvi-szonyok már alig különböztek a mélyen föld alatttapasztalhatótól [8, 9] és 5. ábra.

Ez a felismerés megnyitotta az utat, hogy kis mély-ségû föld alatti helyszínek is bekerülhessenek a nuk-leáris asztrofizikai célra használható gyorsítók telep-helyeinek sorába.

Az elmúlt évek folyamán a Felsenkeller pince-rendszerében kialakításra került egy gyorsítólabora-tórium, amelybe 2017 nyarán egy 5 MV-os Pelletrongyorsítót telepítettünk [10]. A laboratórium kialakítá-sa végsõ szakaszába ért és 2018-tól üzemelni fog. Anagyobb gyorsítópotenciálnak köszönhetõen a hé-liumfúzió reakciói is vizsgálhatóvá válnak. Elsõkénta 3He(α,γ)7Be reakció hatáskeresztmetszetét tervez-zük mérni széles energiatartományban, ezt követõena nukleáris asztrofizika „szent Gráljának” nevezett12C(α,γ)16O reakció hatáskeresztmetszetének mérésekerülhet sorra.

Irodalom1. Fülöp Zs., Gyürky Gy.: Az elemek születése. Szemelvények a

nukleáris tudomány történetébõl. Akadémiai Kiadó, Budapest,2009.

2. Gyürky Gy., Fizikai Szemle 61 (2011) 37.3. Kiss G. Gy., Fizikai Szemle 67 (2017) 7.4. Gyürky Gy., Fizikai Szemle 60 (2010) 37.5. Kiss G. Gy., Fizikai Szemle 67 (2017) 163.6. Broggini C. és mtsai, Ann. Rev. Nucl. Part. Sci. 60 (2010) 53.7. Szücs T. és mtsai, Eur. Phys. Jour. A 44 (2010) 513.8. Szücs T. és mtsai, Eur. Phys. Jour. A 48 (2012) 8. http://dx.doi.

org/10.1140/epja/i2012-12008-7 (open-access)9. Szücs T., PhD Disszertáció (2012), http://hdl.handle.net/2437/

12919110. https://www.hzdr.de/felsenkeller

234 FIZIKAI SZEMLE 2018 / 7–8

Page 21: Fizikai Szemle 68/7–8 – 2018. július–augusztusgoliat.eik.bme.hu/~f.simon/publications/Media/FizSzem-20180708.pdf · okokból standard modellnek hívunk, a világot anyagi és

A CSILLAGKELETKEZÉS VIZSGÁLATA ÚJINTERFEROMETRIKUS MÛSZEREKKEL

A Magyar Tudományos Akadémia 189. közgyûlése kapcsán meg-rendezett A fizika fejlõdési irányai címû tudományos ülésenelhangzott elõadás szerkesztett és kibõvített változata.

Kóspál Ágnes asztrofizikus, hat évet töltötta Leideni Obszervatóriumban és az Euró-pai Ûrügynökségnél, jelenleg az MTA CSFKCsillagászati Intézetében – az Európai Ku-tatási Tanács 1,3 millió eurós támogatásá-val – vezeti kutatócsoportját. A legnagyobbföldi és ûrtávcsövekkel vizsgálja a Naphozhasonló csillagok és bolygórendszereik ke-letkezését. Több, mint félszáz cikkbenpublikált eredményei elismeréseként el-nyerte a L’Oréal–UNESCO NemzetközilegKiemelkedõ Tehetségek díját Párizsban.

Ábrahám Péter fizikus és csillagász, azMTA CSFK Csillagászati Intézet tudomá-nyos tanácsadója, 2010–15 között igazgató-ja. Kutatási területe a csillagok keletkezése,a fiatal csillagok körüli korongok szerkeze-te, a csillagok fényváltozásai. Megfigyelé-seihez rendszeresen használ nagy térbelifelbontású, gyakran infravörös hullámhosz-szakon mûködõ földi vagy ûrmûszereket.Magyarországi vezetõje annak az EU tech-nológiai programnak, amely a MATISSE fej-lesztésében is részt vállalt.

Varga József az ELTE-n szerezte doktorifokozatát 2015-ben, galaxisok nagyskálásszerkezetének elemzésével. Azóta az MTACSFK Csillagászati Intézetében dolgozikposztdoktori kutatóként, ahol fiatal csillag-körüli korongok szerkezetét vizsgálja aVLTI infravörös interferometrikus mûsze-reivel. Egy nemzetközi csapat tagjakéntrészt vesz a VLTI új mûszere, a MATISSEbeüzemelésében és tesztelésében.

Kóspál Ágnes, Ábrahám Péter, Varga JózsefMTA CSFK KTM CSI

A Nap típusú csillagok keletkezése

Galaxisunkban a csillagok közötti teret porból ésgázból álló diffúz anyag tölti ki. Ennek egy részesûrû csillagközi felhõket alkot, amelyekben a hideggáz molekuláris állapotú. A molekulafelhõkben agáznyomás és a tömegvonzás egyensúlya könnyenfelborulhat, akár a felhõk összeütközése, akár egyszupernóva-robbanás által keltett lökéshullám kö-vetkeztében. Ilyenkor a gáz termikus nyomása, amágneses mezõ és a turbulens mozgások nem tud-nak ellenállni a gravitációnak, és a felhõ egyes ré-szei összeomolhatnak.

Egy ilyen összehúzódó, de csillagot még nem tar-talmazó felhõmag kezdetben lassan forog, és mivel afelszabaduló gravitációs energiát teljes mértékbenkisugározza, hõmérséklete 10 K körüli értéken marad.

Amikor a felhõmag közepe elég sûrûvé válik, a meg-növekedett nyomás megállítja az összeomlást, és aközéppontban kialakul a protocsillag. A protocsilla-gok már ionizált hidrogént tartalmaznak, tömegüknéhány tized naptömeg, méretük néhány napsugár. Agravitációs összeomlás kezdete után körülbelül tíz-ezer évvel alakulnak ki.

A kezdeti felhõmag nullától különbözõ perdületemiatt az anyag nem eshet közvetlenül a protocsillagfelszínére, hanem egy lapult, korongszerû struktúrátalkot. Ezen csillagkörüli korongok mérete hasonló ami bolygórendszerünkéhez, tipikusan néhány százcsillagászati egység (CSE, az átlagos Nap–Föld-távol-ság, vagyis 150 millió km) alatti, és bennük az anyaglassan befelé spirálozik, majd végül a csillagra hullik.Ekkor a protocsillag és a korong még mélyen beágya-zódik a környezõ anyagburokba, amelynek a kiterje-dése akár több ezer CSE is lehet. A korongból a csil-lagra hulló anyag egy része bipoláris kifújás formájá-ban távozik a rendszerbõl. Ennek során az anyag aprotocsillag pólusaitól áramlik kifelé, és elkezdi elfúj-ni a burkot. Ez a gravitációs összeomlás kezdete utánkörülbelül százezer évvel történik meg.

Ha az elfogyó burok már nem képes pótolni a ko-rong anyagát, a csillag tömegbefogási rátája egyrecsökken. A korongot ekkor már fõleg a csillag megvi-lágítása fûti, és megindulhat benne a porszemcsékösszetapadása, növekedése. Végül, néhány millió évalatt a korong gáz- és portartalmának nagy része telje-sen szétoszlik, vagy nagyobb testekké, planetezimá-lokká és bolygókká áll össze. Mindezen fázisok konk-rét csillagászati felvételeken is láthatók (1. ábra ).

Az itt felvázolt folyamattal kapcsolatban még szá-mos nyitott kérdés van. Amíg az apró porszemcsék-bõl bolygók lesznek, méretük 13, tömegük 40 nagy-ságrenddel nõ. Közben – a mérettõl függõen – külön-bözõ fizikai folyamatok lesznek a meghatározók anövekedésben. Úgy gondoljuk, hogy a Földhöz ha-sonló kõzetbolygók a korong legbelsõ részén, míg azóriás gázbolygók valamivel távolabb, a víz fagyás-pontjának megfelelõ sugáron túl keletkeznek. Ennekellenére a megfigyelt exobolygórendszerekben gyak-ran látunk gázóriásokat a csillag közvetlen közelé-ben, ami a korongon belüli vándorlásra, bolygómig-rációra utal.

A nagybolygók hatással vannak a korong szerke-zetére és sûrûségeloszlására is, meghatározva, hogya maradék porból hol alakulhatnak ki kisebb égites-tek, aszteroidák, üstökösök. Kiemelten fontos kér-dés a korong ásványi és kémiai összetétele is: meny-nyi víz, mennyi szerves molekula van jelen már alegkorábbi fázisokban, és ezek hogyan kerülhetnek

KÓSPÁL ÁGNES, ÁBRAHÁM PÉTER, VARGA JÓZSEF: A CSILLAGKELETKEZÉS VIZSGÁLATA ÚJ INTERFEROMETRIKUS MUSZEREKKEL 235

Page 22: Fizikai Szemle 68/7–8 – 2018. július–augusztusgoliat.eik.bme.hu/~f.simon/publications/Media/FizSzem-20180708.pdf · okokból standard modellnek hívunk, a világot anyagi és

a bolygók építõanyagába? Az élet

1. ábra. A csillag- és bolygókeletkezés fõbb fázisai. Forrás: ESA/Herschel/PACS, SPIRE/N.Schneider, Ph. André, V. Könyves (CEA Saclay, France), Gould Belt survey Key Prog-ramme, NASA/JPL-Caltech/WISE Team, ESO/ALMA (ESO/NAOJ/NRAO)/H. Arce, B. Rei-purth, ALMA (ESO/NAOJ/NRAO), NRC-HIA, C. Marois, Keck Observatory.

beágyazott protocsillag

csillag körüli korongexobolygórendszer

b c

d

e

molekulafelho

felhomagok´kialakulásához legkedvezõbb felté-telek a korong mely részein jöhet-nek létre? A nagy számú megfigyeltexobolygórendszer meglepõen vál-tozatos. Lehet, hogy e sokszínûségeredete a korongok változatosságá-ban rejlik? Ezek a kérdések a 21.századi asztrofizika legfontosabbkérdései közé tartoznak, megvá-laszolásuk pedig a legmodernebb,legnagyobb teljesítményû mûszerekhasználatát igényli.

Nagy szögfelbontásúészleléstechnika

A Naphoz hasonló csillagok keletke-zésérõl itt felvázolt forgatókönyvnagy része olyan, a Naprendszerszomszédságában található csillag-keletkezési területek tanulmányozá-sából származik, mint az Orion, Tau-rus, Perseus, Cepheus, Aquila, Ser-pens, Ophiuchus, Scorpius, Lupusés a Chamaeleon. Ezek a területekegy körülbelül 3000 fényév átmérõ-jû, fiatal csillagokból és óriás mole-kulafelhõkbõl álló gyûrû, a Gould-öv részei. A hozzánk legközelebbicsillagkeletkezési területek 500 fény-évre találhatók. Ilyen távolságban egy tipikus csillag-körüli korong látszó mérete 1 ívmásodperc (1″, ekko-rának látszana egy golflabda 9 km-rõl, vagy ekkora atelihold átmérõjének 1800-ad része).

Ha tehát a korongokban bármiféle szerkezeti rész-letet szeretnénk megfigyelni, olyan képeket kell ké-szítenünk, amelyek felbontása tized ívmásodperc,vagy még annál is jobb. Ráadásul a korong hõmérsék-let-eloszlása sem egyenletes: belsõ része forróbb, ígya Wien-féle eltolódási törvénynek megfelelõen inkábba közeli és középinfravörös tartományban sugároz,míg a külsõ, hidegebb részt elsõsorban távoli infravö-rös és milliméteres hullámhosszakon lehet megfigyel-ni. A különbözõ hullámhosszon történõ megfigyelé-sek elengedhetetlenek ahhoz, hogy egy fiatal csillag-ról és környezetérõl teljes képet kapjunk.

A csillagászati mûszerek által még éppen felbont-ható részletek mérete ideális esetben (ha a légkö-ri turbulencia zavaró hatását elhanyagoljuk) fordí-tottan arányos a távcsõtükör vagy antenna D átmé-rõjével és egyenesen arányos az észlelt fény λ hul-lámhosszával, tehát a szögfelbontás θ = k λ /D, aholk = 70, ha θ -t fokban mérjük. Látható tartományban1,3 méteres távcsõtükör kell a 0,″1 felbontáshoz, azinfravörös tartományban – például 10 μm-en – már25 méteres tükörátmérõ szükséges. A milliméterestartományban több km-es, a rádiótartományban pe-dig több száz km átmérõjû rádióantennákra lenne

szükség. Ilyen óriástávcsöveket és óriásantennákattechnikailag lehetetlen megépíteni, a költségekrõlnem is beszélve.

Gazdaságos és megvalósítható óriástávcsövet tehátúgy lehet készíteni, ha nem a teljes távcsövet, hanemcsak egyes kisebb részeit építjük meg, és azokat ösz-szehangoltan használjuk. Ezen az ötleten alapul azinterferometrikus technika, amelynek keretében távoliteleszkópokat irányítanak ugyanarra az égi objektum-ra, és a távcsövek jeleit interferáltatják. Egy ilyenrendszer esetében a felbontást már nem az egyes táv-csövek átmérõje szabja meg, hanem a köztük lévõtávolság, azaz a bázisvonal (2. ábra ).

Nem igaz azonban, hogy két távcsövet akármek-kora távolságba helyezve tetszõlegesen jó szögfelbon-tású képeket készíthetünk. A technikából adódóanugyanis egy interferométer nem közvetlenül az égenlátható fényességeloszlást szolgáltatja, hanem annakFourier-transzformáltját (az úgynevezett komplexvizibilitást), és azt is csak az adott bázisvonal hosszá-nak (illetve az abból számolt térfrekvenciának) éspozíciószögének megfelelõ pontban. Ahhoz tehát,hogy egy interferometrikus mérésbõl meghatározhas-suk a csillagászati objektum képét, a Fourier-teret alehetõ legtöbb pontban kell mintavételezni, amihezviszont sok különbözõ hosszúságú és pozíciószögûbázisvonalra, azaz minél több összekapcsoltan mûkö-dõ teleszkópra van szükség.

236 FIZIKAI SZEMLE 2018 / 7–8

Page 23: Fizikai Szemle 68/7–8 – 2018. július–augusztusgoliat.eik.bme.hu/~f.simon/publications/Media/FizSzem-20180708.pdf · okokból standard modellnek hívunk, a világot anyagi és

Bár a rádiócsillagászok már az 1940-es évektõl kí-

2. ábra. Kétantennás interferométer, a mûszer térbeli felbontását ab bázisvonal hossza szabja meg. Forrás: Condon & Ransom (2016),https://www.cv.nrao.edu/course/astr534/Interferometers1.html

c

b=

cos

g

q

t

q

1 2

b

V V1 2

V V t2 = cos( )wV V t1 = cos[ ( – )]gw t

R V=( /2)cos( )2gwt

3. ábra. ALMA antennák a chilei Atacama-sivatagban. Forrás: Carlos Padilla (UI/NRAO).

sérleteztek az interferometrikus technikával, az elsõnagyon hosszú bázisvonalú sikeres interferometrikuscsillagászati mérést (Very Long Baseline Interferometry,VLBI) 1967-ben sikerült elvégezni egymástól több mint3000 km távolságban lévõ két észak-amerikai rádióte-leszkóppal. Az adatokat korábban atomórák pontosidõjeleivel együtt mágneses adathordozókra rögzítet-ték, és utólag állították elõ az interferenciát, ezt hívjukkorreláltatásnak. Napjainkban már számos rádióanten-nát széles sávú optikai adatkábelek kötnek össze, lehe-tõvé téve a valós idejû korrelálást. Ilyen például azeurópai VLBI hálózat, amelyet magyar kutatók, példáulaz MTA CSFK Csillagászati Intézetében Frey Sándor ésmunkatársai is rendszeresen használnak.

Az ALMA és a NOEMA

A fiatal csillagok körüli korongok termikus sugárzásá-nak megfigyelésére különösen alkalmas a 2011-benüzembe állított új rádióteleszkóp-rendszer, az AtacamaLarge Millimeter/submillimeter Array (ALMA, 3. ábra ),mely a 0,3–3 mm-es sugárzás észlelésére alkalmas. Etartományban a földi légkör áteresztése sávos: bizonyossávokban jó az áteresztés, más sávokban a légkör telje-sen átlátszatlan. Az elnyelést a levegõben található víz-pára okozza. Szerencsére a vízgõz skálamagassága csak2 km (ilyen magasan esik felére a vízgõz parciális nyo-mása). Az ALMA-t emiatt kellõen száraz és magas helyreépítették, a chilei Atacama-sivatagban az 5050 m-estengerszint feletti magasságú Chajnantor-fennsíkra.

Az ALMA ötvennégy 12 m és tizenkét 7 m átmérõjû,tökéletesen paraboloid felületû (az attól való eltéréskisebb mint 20 μm) antennából áll. Teljes fénygyûjtõfelülete 6600 m2. A 66 antennához 192 alap tartozik, ígyazokat – az éppen adott mérési projekthez szükségestérbeli felbontásnak megfelelõen – különbözõ konfi-gurációkba lehet rendezni. A legnagyobb elérhetõ bá-zisvonal 16 km, amellyel így akár 0,″006 (6 milliívmá-sodperc) felbontás is elérhetõ (ekkorának látszana egygolflabda 1400 km-rõl). Ilyen felbontással már a koron-gok apró részletei is tanulmányozhatók.

Az ALMA 1 milliárd eurós építési költségét és afolyamatos mûködtetéshez-fejlesztéshez szükségesköltségvetést globális összefogásban európai, észak-amerikai és kelet-ázsiai országok, valamint Chile fede-zi. Az antennák jelének korrelálását (interferáltatását)a világ egyik legnagyobb speciális szuperszámítógé-pe, az ALMA-korrelátor végzi. A korrelátorban többmint 134 millió processzor van, amelyek együttesen17 peta-FLOP számítási sebességre képesek, azazmásodpercenként 17 000 billió számítási mûveletetvégeznek. Erre szükség is van, hiszen az antennákrólmásodpercenként 1 GB feldolgozandó adat érkezik.

Az ALMA eddigi egyik legrészle-tesebb és legélesebb képét a 450fényév távolságban, a Bika csillag-képben található fiatal (1 millióéves) Nap-típusú csillag, a HL Taukorongjáról készítette. Az ALMA-képen korábban soha nem látottrészletek tárultak fel a csillagkörülikorongban: koncentrikus fényesgyûrûk, amelyeket sötét rések vá-lasztanak el egymástól. Elképzelhe-tõ, hogy ezt a szerkezetet a korong-ban keletkezett, a fiatal csillag körülkeringõ bolygók okozzák. Ilyenesetben ugyanis ezen testek – gravi-tációs hatásuknál fogva – ki tudjáktisztítani a pályájuk menti környeze-tet. A HL Tau gyûrûs szerkezetû ko-rongja tehát közvetett bizonyíték le-het arra, hogy már egy ilyen fiatalrendszerben is létrejöhetnek boly-gók (1. ábra, jobbra lent).

KÓSPÁL ÁGNES, ÁBRAHÁM PÉTER, VARGA JÓZSEF: A CSILLAGKELETKEZÉS VIZSGÁLATA ÚJ INTERFEROMETRIKUS MUSZEREKKEL 237

Page 24: Fizikai Szemle 68/7–8 – 2018. július–augusztusgoliat.eik.bme.hu/~f.simon/publications/Media/FizSzem-20180708.pdf · okokból standard modellnek hívunk, a világot anyagi és

Földrajzi helyzeténél fogva az

4. ábra. Vázlat a V346 Nor rendszer felépítésérõl az ALMA-adatok alapján. Forrás: MTACSFK CSI/Kereszturi Ákos.

behullókülsoburok

pszeudokorong

forgó, csillagkörüli korong

protocsillag

5. ábra. A VLT Interferométer által használt távcsövek és az azokat összekötõ fényutak.Forrás: ESO.

ALMA csak a déli égbolton és az égiegyenlítõ mintegy 30 fokos környeze-tében elhelyezkedõ objektumokattudja mérni. Az északi objektumokmérésére egy francia–spanyol–németegyüttmûködésben a Francia-Alpok-ban fenntartott milliméteres interfero-méter, a NOrthern Extended Millime-ter Array (NOEMA) ad lehetõséget. ANOEMA 2550 m tengerszint feletti ma-gasságban a Plateau de Bure fennsí-kon található és jelenleg kilenc 15 mátmérõjû antenna alkotja. A tervekszerint a következõ években továbbihárom antennával bõvítik majd arendszert, amely így 2100 m2 fény-gyûjtõ felületû lesz. A NOEMA anten-náit is különbözõ konfigurációkba lehet rendezni, a leg-nagyobb bázisvonal jelenleg 760 m, az elérhetõ legjobbtérbeli felbontás pedig 0,″2. A NOEMA a 0,8–4,3 mm kö-zötti hullámhosszú sugárzást tudja detektálni.

E nemzetközi milliméteres interferometrikus pro-jektekben több magyar kutató is részt vett: Kóspál Ág-nes és Juhász Attila a Leideni Obszervatórium ALMA-központjában dolgozott, Fehér Orsolya pedig 2018õszétõl az Institute de Radioastronomie Millimetrique(IRAM) grenoble-i központjában fog – egyebek közt aNOEMA projektben – dolgozni. Az ezirányú kutatáso-kat a Magyar Tudományos Akadémia (MTA) és azEurópai Kutatási Tanács (ERC) is támogatja. KóspálÁgnes 2014-ben nyerte el az MTA Lendület-ösztöndí-ját a Csillagkörüli korongok – csillag- és bolygókelet-kezés az ALMA-korszakban témájú projektjével, majd2017-tõl az ERC kutatási ösztöndíját a Strukturáltanyagbefogási korongok mint a bolygókeletkezés idõ-függõ kezdõfeltételei címû pályázatára.

Az egyik magyar vezetésû, ALMA-adatokon alapulóérdekes eredmény a közelmúltból a déli féltekén, aNorma csillagképben található V346Nor jelû fiatal csillaggal kapcsolatos.A V346 Nor egy 0,1 naptömegû,néhány százezer éves, jelenleg isnövekvõ protocsillag. Kóspál Ágnesés munkatársai az ALMA-val a szén-monoxid-molekula egyik milliméte-res színképvonalát vették célba,amelynek Doppler-eltolódását ki-használva feltérképezték a rendszer-beli gáz mozgását. Az adatok elem-zésébõl kapott kép szerint a proto-csillagot körülbelül 350 CSE sugáronbelül Kepler törvényeinek megfele-lõen keringõ korong veszi körül (agáz v sebessége a csillagtól mért rtávolság függvényében: v ~ r 0,5),míg a csillagtól távolabb az anyagsebessége ennél kisebb, a v ~ r relá-ciót követi. Ez utóbbi sebességelosz-lás jellemzõ a pszeudokorongokra,

olyan struktúrákra, amelyekben az anyag a keringésés behullás kombinációjával mozog (4. ábra ).

A mérésekbõl az is kiderült, hogy az anyagáramlásüteme a V346 Nor korongjára lényegesen nagyobb,mint amennyi anyagot a korong továbbít a protocsil-lagra. Ennek következményeként az anyag a korongbelsõ peremén felhalmozódik, egy idõ után instabilláválik, majd hirtelen a központi csillagra zúdul. Ilyen-kor a nagy mennyiségû, rendkívül gyorsan felszaba-duló energia hatására a korong felforrósodik, és ben-ne olyan lényeges kémiai és ásványtani változásokmehetnek végbe, amelyek késõbb befolyásolhatják,hogy a korongban hol és milyen összetételû bolygókalakulnak ki.

A VLT Interferométer

Ha a fiatal csillagok körüli korongok legbelsõ, legfor-róbb, a földpályának megfelelõ sugarú vagy még an-nál is közelebbi részeire vagyunk kíváncsiak, akkor a

238 FIZIKAI SZEMLE 2018 / 7–8

Page 25: Fizikai Szemle 68/7–8 – 2018. július–augusztusgoliat.eik.bme.hu/~f.simon/publications/Media/FizSzem-20180708.pdf · okokból standard modellnek hívunk, a világot anyagi és

milliméteres tartomány helyett infravörösben kell na-

6. ábra. A MATISSE mûszer a VLT interferometrikus laboratóriumában. Forrás: MATISSEProject/Y. Bresson.

gyon jó felbontású méréseket végeznünk. Erre szolgálaz Európai Déli Obszervatórium (European SouthernObservatory, ESO) Paranal Obszervatóriumában talál-ható Nagyon Nagy Távcsõ Interferométer (VLTI). AVLTI-t négy 8,2 m és négy 1,8 m tükörátmérõjû távcsõalkotja (5. ábra ). A nagyobb távcsövek fix pozíciójúkupolákban találhatók, a kisebbek 30 különbözõ po-zícióba helyezhetõk. Az interferenciához a távcsövekjelét egy bonyolult tükörrendszerrel föld alatti alagút-rendszerben egy optikai laborba vezetik, ahol külön-bözõ mûszerekkel kiválasztják a mérni kívánt hullám-hossztartományt, majd létrehozzák és detektorokkalrögzítik az interferenciajelet. A távcsövek kettesével,hármasával vagy négyesével használhatók az interfe-rometriára, így a rendszer egy 200 méter átmérõjûóriástávcsõként üzemel, és vele akár 0,″004 szögfel-bontás is elérhetõ.

A MIDI (MID-infrared Interferometric instrument) aközépinfravörösben (a 8–13 μm közötti tartomány-ban) vett fel interferometrikus színképeket. Ezek ahullámhosszak azért különösen érdekesek, mert ittlátható a csillagkörüli korongok szilárd anyagánaknagy részét alkotó szilikátszemcsék egy széles színké-pi alakzata. Az alakzat profiljából lehet a szemcsékméretére és pontos anyagi szerkezetére is következ-tetni. A MIDI-nek köszönhetõen tudjuk, hogy sokkorongban a néhány CSE-en belüli részen a szilikát-szemcsék nagyok és kristályosak, ezen kívül viszont –

a csillagközi porra is jellemzõ –szubmikronos (1 μm alatti méretû)amorf szilikátszemcsék találhatók.

A MIDI-t 2015-ben leszerelték, ésrá egy évre minden adata nyilvános-sá vált az ESO adatarchívumában.Ekkor az MTA CSFK CSI-ben dolgo-zó Varga József és munkatársai le-töltöttek és egységesen feldolgoztakminden Nap-típusú fiatal csillagrólkészült MIDI-mérést. A több mint 80csillag mindegyikéhez korongmo-dellt illesztettek. Így kiderült, hogymekkora a korong 10 μm-en sugár-zó régiója, és ez hogyan korrelálkülönbözõ paraméterekkel, példáula központi csillag luminozitásával. Alétrehozott adatbázis a MIDI értékesöröksége, és nagymértékben segít-heti majd a késõbbi mérések meg-tervezését is.

Az MTA CSFK CSI másik kutatója,Lei Chen és kollégái a közepes tö-megû, vagyis a Napnál kissé többanyagot tartalmazó csillagokat kutat-ják. A VLTI egy másik mûszerével, aPIONIER-ral készítettek közeli infra-vörös interferometrikus méréseket aHD 169142 jelû csillagról. Az ada-tokból – szintén modellillesztés se-gítségével – kimutatták, hogy a csil-

lagtól 0,07 CSE távolságra (ez a Merkúr pályasugará-nak mindössze ötöde) mérhetõ mennyiségû por talál-ható, amelynek hõmérséklete 1500 K.

Az elõbb ismertetett két projektben a kutatók vi-szonylag kevés különbözõ bázisvonalon mért vizibili-táspontot illesztettek, és ebbõl határozták meg a modellparamétereit. Ezekkel a mûszerekkel azonban nem le-het valódi képet készíteni. Ezért nagy újítás a MIDIutódjának tekinthetõ MATISSE (Multi AperTure mid-Infrared SpectroScopic Experiment) mûszer, a közép-infravörösben mûködõ elsõ olyan interferométer, ame-lyik négy távcsõ jelét kombinálja, és képalkotásra isalkalmas. 3,2–3,9, 4,5–5,0 és 8–13 μm-en fog mûködni,továbbá színképeket is felvesz majd (6. ábra ).

A MATISSE tervezése, majd építése nagyjából egyévtizede kezdõdött, és magyar kutatók és mérnökökis részt vettek benne. Nevükhöz például az abszolútnulla fok közelében mûködõ motorok által okozotthõterhelésre vonatkozó hõtani szimulációk fûzõdnek,de magyarok tervezték és készítették az egyik mû-szeregység kalibrációs kameráját is. Továbbá model-lezést végeztek a mûszer biztonságos mozgatásáhozszükséges szállítóelemekre és a szállítási mûveleteksorán várható mechanikai igénybevételre, valaminthozzájárultak a mûszer felhasználói szoftverénekmegalkotásához és fejlesztéséhez is (7. ábra ).

2017 novemberében a MATISSE épségben megér-kezett a Paranal Obszervatóriumba. Az interferenciát2018 januárjában elõször egy belsõ kalibrációs forrá-

KÓSPÁL ÁGNES, ÁBRAHÁM PÉTER, VARGA JÓZSEF: A CSILLAGKELETKEZÉS VIZSGÁLATA ÚJ INTERFEROMETRIKUS MUSZEREKKEL 239

Page 26: Fizikai Szemle 68/7–8 – 2018. július–augusztusgoliat.eik.bme.hu/~f.simon/publications/Media/FizSzem-20180708.pdf · okokból standard modellnek hívunk, a világot anyagi és

son, majd februárban egy égi objektumon, a Sziriuszon

7. ábra. Magyar hozzájárulás a MATISSE-hoz: hõtani modell (balra fent), az elkészült kalibrációs kamera (jobbra fent), szállítóelemek mo-delljei (balra lent) és felhasználói szoftver részlete (jobbra lent). Forrás: MTA CSFK CSI.

Szerkesztõség: 1092 Budapest, Ráday utca 18. földszint III., Eötvös Loránd Fizikai Társulat. Telefon/fax: (1) 201-8682

A Társulat Internet honlapja http://www.elft.hu, e-postacíme: [email protected]

Kiadja az Eötvös Loránd Fizikai Társulat, felelõs kiadó Groma István fõtitkár, felelõs szerkesztõ Lendvai János fõszerkesztõ.

Kéziratokat nem õrzünk meg és nem küldünk vissza. A szerzõknek tiszteletpéldányt küldünk.

Nyomdai elõkészítés: Kármán Stúdió, nyomdai munkálatok: OOK-PRESS Kft., felelõs vezetõ: Szathmáry Attila ügyvezetõ igazgató.

Terjeszti az Eötvös Loránd Fizikai Társulat, elõfizethetõ a Társulatnál vagy postautalványon a 10200830-32310274-00000000 számú egyszámlán.

Megjelenik havonta (nyáron duplaszámmal), egyes szám ára: 900.- Ft (duplaszámé 1800.- Ft) + postaköltség.

HU ISSN 0015–3257 HU ISSN 1588–0540(nyomtatott) és (online)

is tesztelték. A mûszer tesztelésében és beüzemelésé-ben szintén részt vesznek magyar kollégák. A tervekszerint a MATISSE-t majd 2019-tõl pályázati rendszer-ben kínálják használatra a csillagászközösségnek.

A magyar kutatók a projekthez való hozzájárulásu-kért cserébe garantált távcsõidõt kapnak, azaz egyideig õk használhatják a négy 8,2 méteres távcsövet aMATISSE-szal. A célpont az FU Ori lesz, egy olyanrendkívül érdekes fiatal csillag, amely 1936-ban várat-lanul százszorosára fényesedett, és az azóta eltelttöbb mint 80 évben fényereje alig csökkent. Úgy gon-doljuk, hogy ennek a V346 Nor kapcsán már említettkorongbeli instabilitás, és így a megnövekedett – akorongról a csillagra történõ – tömegátadás az oka.Reméljük, hogy az új interferometrikus mérésekkelközelebb juthatunk majd a korong titkainak megérté-séhez, és ahhoz is, hogy általában miként épülnek fela Naphoz hasonló csillagok.

Az interferometria a modern csillagászat egyik leg-ígéretesebb technológiája, amely a földpálya méret-skáláján képes képet alkotni a születõ bolygórendsze-rekrõl. A magyar csillagászat az elmúlt 15 évben mind

technikailag, mind tudományosan részt vett és vesz azilyen irányú európai projektekben. Mindezt úgy sike-rült elérni, hogy Magyarország még nem tagja azEurópai Déli Obszervatóriumnak. A remélt ESO-csat-lakozásunk az ehhez hasonló mûszertechnikai és tu-dományos projekteket sokkal gördülékenyebbé tehe-ti, és folyamatosan biztosítja hozzáférésünket az élvo-nalbeli eszközökhöz, közöttük a most épülõ 39 m-estükörátmérõjû ELT távcsõhöz.

Magyar nyelvû ajánlott irodalomFrey Sándor, Gabányi Krisztina Éva: Nagy szögfelbontású rádió-inter-

ferometria – ûreszközökkel. In: Konkoly Monográfiák 6, KelemenJános, Szabados László (szerk.): Ûrcsillagászat Magyarországon,2010, 62–68; http://www.konkoly.hu/Mitteilungen/m6.pdf

Kóspál Ágnes, Moór Attila: Hogyan látja az ALMA a fiatal csillagokkorongjait? In: Meteor Csillagászati Évkönyv 2016. Magyar Csil-lagászati Egyesület, Budapest, 2015.

Mosoni László: Amíg a 100 méteres távcsövek elkészülnek. In: Me-teor Csillagászati Évkönyv 2013. Magyar Csillagászati Egyesület,Budapest, 2012.

http://mta.hu/tudomany_hirei/ujabb-lepes-a-bolygok-keletkezesenek-megerteseben-magyar-kutatoknak-elsokent-sikerult-megmerniuk-egy-fiatal-csillag-koruli-anyagaramlas-merteket-107855

http://mta.hu/tudomany_hirei/uj-muszer-kesziti-az-eddigi-legreszletesebb-felveteleket-az-egitestekrol-108509

240 FIZIKAI SZEMLE 2018 / 7–8

Page 27: Fizikai Szemle 68/7–8 – 2018. július–augusztusgoliat.eik.bme.hu/~f.simon/publications/Media/FizSzem-20180708.pdf · okokból standard modellnek hívunk, a világot anyagi és

ANYAGTUDOMÁNY SZÁMÍTÓGÉPPEL – 2. rész

3. ábra. A kutatási feladatok felosztása Donald Stokes szerint. Csoportunkbanalapkutatást végzünk, amelynek néha van, néha pedig nincs közvetlen gyakorlatimotivációja (pirossal bekarikázott negyedek).

Az

alap

ism

eret

ein

kn

öve

lése

acé

l?

igen

nem

Egy gyakorlati probléma megoldása a cél?nem igen

tiszta alapkutatás

tiszta alkalmazott kutatás

Bohr Pasteur

Edison

felhasználás-motivált alapkutatás

Az MTA Fizikai Osztálya 2018. május 10-i tudományos ülésén azo-nos címmel elhangzott elõadás bõvített, írott változata.

Pusztai Tamás (45) az MTA doktora, azMTA Wigner Fizikai Kutatóközpont tudo-mányos tanácsadója. Kutatásait a számító-gépes anyagtudomány területén végzi, azonbelül is elsõsorban a megszilárdulás soránkialakuló növekedési formák fázismezõmo-dellel történõ leírásával foglalkozik.

Rátkai László PhD hallgató a Wigner FizikaiKutatóközpontban komplex megszilár-dulási jelenségek fázismezõ elméleti mo-dellezésével foglalkozik. Eddigi munkáibana helikális belsõ szerkezetû eutektikusdendritek kialakulását vizsgálta, valamint afázismezõ és Lattice–Boltzmann-egyenletekcsatolásával kidolgozta az olvadékáramlásjelenlétében történõ megszilárdulás olyanmodelljét, amely megengedi a kialakulószilárd szemcsék mozgását is.

Gránásy László fizikus az MTA doktora, azMTA Wigner Fizikai Kutatóközpont tudo-mányos tanácsadója, az Academia Euro-paea tagja. Diplomát és PhD-fokozatot azELTE-n szerzett. Több európai, amerikai ésjapán egyetem, kutatóintézet ösztöndíjasa,vendégkutatója, professzora. Fõ érdeklõ-dési területe a komplex megszilárdulásifolyamatok térelméleti modellezése atomiskálán mûködõ és lokálisan átlagolt(coarse grained) módszerekkel.

Pusztai Tamás, Rátkai László, Gránásy LászlóWigner Fizikai Kutatóközpont, SZFI Kísérleti Szilárdtestfizikai Osztály

Az folyóirat elõzõ és a mostani számábanmegjelent kétrészes cikk témája a számító-gépes anyagtudomány. Az elsõ részben ál-talános áttekintést adtam az anyagtudo-mány helyzetérõl, különösen az új anyagokminél gyorsabb kifejlesztésének igényekapcsán felmerülõ kihívásokról és kezde-ményezésekrõl, amelyben a számítógépesanyagtudománynak fontos szerepe van. Amostani részben pedig néhány olyan, cso-portunk által végzett munkát ismertetünk(mivel csoportmunkáról van szó, immártöbbes szám elsõ személyben), amelyek be-mutatják, hogy a számítógépekkel végzettszimulációk miként segíthetik az anyagtu-dományi folyamatok megértését és mikéntkapcsolódhatnak valós, gyakorlati problé-mák megoldásához.

A kutatómunka Stokes-féle felosztása

A kutatási feladatokat hagyományosan azalapkutatás és alkalmazott kutatás kategó-riákba szoktuk sorolni, ez azonban nem mindig adelég árnyalt képet az adott munka jellegérõl. Ezeket akategóriákat finomította tovább Donald E. Stokes [15],aki két különbözõ kritérium által meghatározott fel-osztást javasolt. Az egyik kritérium, hogy az adottkutatási feladat során fontos cél-e a folyamatok meg-

értése, a természet mûködését leíró alapvetõ ismere-teink bõvítése, a másik pedig, hogy a kutatási fel-adatnak célja-e egy konkrét gyakorlati probléma meg-oldása. A két kérdésre adott válasz alapján egy 2 ×2-es mátrixszal jól reprezentálható feloszlást nyerünk,amelynek mezõit az adott terület jellemzõ kutatójárólBohr (tiszta alapkutatás), Pasteur (felhasználásmoti-vált alapkutatás) és Edison (tiszta alkalmazott kutatás)negyedeknek nevezte (3. ábra ). A negyedik negyedüresen marad, hiszen ha sem a megértés, sem pedig afelhasználás nem cél, nem beszélhetünk kutatásról.

A Wigner FK Szilárdtestfizikai és Optikai Intézeté-ben mûködõ csoportunk fõ célja mindig a megértésés az alapvetõ ismereteinket gazdagító alapkutatás. Afenti csoportosítást azért mutattuk be, mert jól illuszt-rálja, hogy ezen kategórián belül is léteznek külön-bözõ jellegû feladatok. Az alábbiakban nagyon rövi-den bemutatott néhány munkánkat is ebben a felosz-tásban tárgyaljuk.

PUSZTAI TAMÁS, RÁTKAI LÁSZLÓ, GRÁNÁSY LÁSZLÓ: ANYAGTUDOMÁNY SZÁMÍTÓGÉPPEL – 2. RÉSZ 241

Page 28: Fizikai Szemle 68/7–8 – 2018. július–augusztusgoliat.eik.bme.hu/~f.simon/publications/Media/FizSzem-20180708.pdf · okokból standard modellnek hívunk, a világot anyagi és

Tiszta alapkutatási feladatok

4. ábra. Hõmérséklet-gradiensben növekvõ aszimmetrikus szferolitok mikroszkópos felvétele [22](balra) és számítógépes szimulációja (jobbra). A szimulációs ábrán a színek a kémiai összetételtmutatják.

100 m�

5. ábra. Helikális belsõ szerkezetû eutektikus dendritek egy egyszerû ternér rendszerben. A bal ol-dali ábrán a piros és sárga színnel jelölt szilárd fázisok egyszeres, míg a jobb oldali ábrán három-szoros spirális szerkezetet mutatnak.

6. ábra. A növekedési morfológia változása irányított megszilárdulás során. A 4 × 4 ábra a 4 különbözõ hõmérséklet-gradiens (balróljobbra nõ) és 4 különbözõ húzási sebesség (lentrõl felfelé nõ) mellett kialakuló mikroszerkezetet mutatja [27]. A színezés az orientációszerint történt. A vizsgált tartományban kis sebességek esetén az oszlopos, míg nagy sebességek esetén az ekviaxiális növekedési formajellemzõ.

(Bohr-negyed)Polikristályos növekedési formák modellezése

Ezt a témakört mindenképpen szeretnénk megem-líteni, hiszen legnagyobb sikereinket ebben értük el[16–18]. Említésnél többre azonban nem vállalko-zunk, mert egyrészt a témakör óriási, másrészt e té-

máról korábban a FizikaiSzemlében is részletesen ír-tunk [19–21]. Munkánk alapjaaz az orientációs mezõvelkibõvített fázismezõmodellvolt, amelyet csoportunkbanfejlesztettünk tovább úgy,hogy az orientációs mezõ fo-lyadékfázisra kiterjesztésévelalkalmassá vált olyan megszi-lárdulási folyamatok leírásárais, amelyek során az orientá-ciós rendezõdés nem, vagycsak részlegesen valósulmeg. Így a szokásos, orientá-ciósan rendezett, azaz egy-kristályszerkezetû szilárdszemcsék mellett olyan szem-csék modellezése is lehetõvévált, amelyek már növekedé-sük során polikristályos szer-kezetet alakítanak ki. Azelõbbire példa a dendritesmegszilárdulás, amelynek so-rán hópelyhekhez hasonló,azok szerkezeti sajátosságaitmutató szabályos formák fej-lõdnek, az utóbbira pedig a

szferolitok népes családja hozható példaként, ame-lyek – rendezetlen kristályszerkezetük ellenére vagyéppen azért – nagyobb méreteket elérve szabályos,kör-, illetve gömbszerû formát vesznek fel. A kísérle-tek és nekik megfelelõ szimulációk – amire a 4. ábramutat egy példát – kiváló kvalitatív egyezése aztmutatja, hogy az orientációs mezõn alapuló model-lünk jó általános alapot ad a szferolitok kialakulásá-nak leírására.

242 FIZIKAI SZEMLE 2018 / 7–8

Page 29: Fizikai Szemle 68/7–8 – 2018. július–augusztusgoliat.eik.bme.hu/~f.simon/publications/Media/FizSzem-20180708.pdf · okokból standard modellnek hívunk, a világot anyagi és

Helikális belsõ szerkezetû

3000

2500

2000

1500

1000

500

00 500 1000 1500 2000 2500 3000

0,44 0,45 0,46 0,47 0,48

koncentráció

3000

2500

2000

1500

1000

500

0

3000

2500

2000

1500

1000

500

0

eutektikus dendritek

Ezt a munkát francia kollégáink lenyûgözõ kísérletifelvételei motiválták [23]. Egy átlátszó háromkompo-nensû ötvözet megszilárdulásának mikroszkóposvizsgálata során helikális belsõ szerkezetû dendritesnövekedési formákat figyeltek meg. Bár ez a növeke-dési forma elég összetett, de kialakulásában szerepetjátszó folyamatok egyszerûek. Megmutattuk, hogyegy szimpla ternér, a folyadék és szilárd fázisokraideális, illetve regulárisoldat-modelleket feltételezõfázismezõmodellel ez a növekedési forma is jól repro-dukálható [24, 25]. A szimulációkban nem csak egy-szeres, hanem többszörös spirálokat is megfigyelhe-tünk (5. ábra ). A különbözõ számú spirálkarból állómegoldások akár ugyanolyan anyagi és egyéb fizikaiparaméterek mellett is létrejöhetnek. Azt, hogy a többlehetõség közül az adott rendszerben melyik megol-dás és melyik csavarodási irány valósul meg, a rend-szerben jelen levõ fluktuációk határozzák meg. A spi-rálkarok számának várható értéke a dendrit csúcsá-nak görbületét meghatározó paraméterek (dendrit-csúcs-sebesség, felületi szabadenergia, kinetikus an-izotrópia) függvénye [26].

7. ábra. Az oszloposról ekviaxiális növekedésre váltás (Columnarto Equiaxed Transition, CET) számítógépes szimulációja az ekvi-axiális részecskék mozgásának figyelembevételével. Az egymástkövetõ képek a szimuláció idõben egymást követõ pillanatképei, aszínezés az Al-Ti rendszer Al-tartalmát (mol%) mutatja. A titánbangazdag dendritek a gravitációs tér hatására lefelé mozdulnak. Ez amozgás áramlást kelt az olvadékban, ami visszahat a részecskékmozgására és növekedésére is.

Felhasználásmotivált alapkutatási feladatok(Pasteur-negyed)IMPRESS projekt

A technikai ötvözetek mechanikai tulajdonságaitalapvetõ módon határozza meg a megszilárdulás so-rán kialakuló mikroszerkezetük. Egy öntéssel elõállí-tott eszköz esetén az olvadék megszilárdulása a leg-hidegebb helyen, azaz az öntõforma falán indulmeg. A szilárd fázis növekedési formáját elsõsorbana ∇T hõmérséklet-gradiens és a megszilárdulási fronthûtési sebességtõl függõ v haladási sebessége hatá-rozza meg. A kialakuló mikroszerkezet eleinte azedény falán képzõdött szilárd magokból kiinduló,karácsonyfaszerû oszlopos dendritekbõl áll, amelykésõbb (∇T és v lokális értékeinek változása miatt) amegszilárdulási front elõtt képzõdõ kristálycsírákbólkiinduló ekviaxiális dendritekbõl álló szerkezetbevált át (6. ábra ). Ezen – angolul Columnar to Equi-axed Transition (CET) nevezett – folyamat megérté-se és kontrollálása tehát óriási gyakorlati jelentõség-gel is bír. Egy korábbi, mintegy 40 európai kutatóin-tézet részvételével létrejött IMPRESS nevû EU FP6-osprojektben ezen átalakulás modellezése volt a fel-adatunk. A programcélja egy olyan Al-Ti alapú repü-

PUSZTAI TAMÁS, RÁTKAI LÁSZLÓ, GRÁNÁSY LÁSZLÓ: ANYAGTUDOMÁNY SZÁMÍTÓGÉPPEL – 2. RÉSZ 243

Page 30: Fizikai Szemle 68/7–8 – 2018. július–augusztusgoliat.eik.bme.hu/~f.simon/publications/Media/FizSzem-20180708.pdf · okokból standard modellnek hívunk, a világot anyagi és

lõgép-hajtómû turbinalapát elõállítása volt, amely a

8. ábra. Néhány olyan eutektikus szerkezet, amelyek kialakulását jól és egy olyan, amelyet nem annyira jól (jobbra lent) sikerült model-lezni. A képek a kísérleti felvételeket és a nekik megfelelõ szimulációkat mutatják.

25 m�

1 m�

10 m�

0,5 mm

1 m�

szokásosan használt Ni-alapú szuperötvözetekhezképest jelentõsen kisebb sûrûségû, de legalábbolyan jó mechanikai tulajdonságokkal bír, még ameglehetõsen magas, 1000 K körüli üzemi hõmér-sékleten is.

GRADECET projekt

A CET tanulmányozására eddig felírt elméleti model-lek és numerikus szimulációk nem, vagy csak részle-gesen vették figyelembe az olvadékfázis áramlásá-nak hatását, amelyet a szilárd és a folyadék fázisokközti, valamint az olvadék kémiai összetételtõl füg-gõ sûrûségkülönbsége hajt a gravitációs tér jelenlé-tében. Az olvadékáramlás hatásának vizsgálata cél-szerûen az áramlást hajtó hatás, azaz a nehézségierõ hangolásával lehetséges. A gravitáció CET-regyakorolt hatásának tanulmányozása az Európai Ûr-ügynökség (European Space Agency, ESA) által fi-nanszírozott GRADECET projekt feladata, amelybena folyamat kísérleti és elméleti vizsgálata történt hi-pergravitációs (1–20g nehézségi gyorsulás az ESAnagy átmérõjû centrifugájában) és mikrogravitációs(10−5g nehézségi gyorsulás az ESA MAXUS rakétájá-ban) környezetben. E projektben is a modellezés volta feladatunk (7. ábra ). Az áramlási tér egyrészt amegszilárdulási front elõtt kialakuló kémiai össze-tételeloszlás módosításán keresztül befolyásolhatjajelentõsen a növekedési formát, másrészt az oszlo-posan növekvõ dendritek elõtt képzõdõ ekviaxiális

dendritek „szállításával” átrendezheti azok eloszlását,amitõl az oszlopos növekedés blokkolása más feltéte-lek mellett következhet be. Az elõbbi folyamat figye-lembe vételére már korábban is történtek próbálko-zások, de az utóbbi folyamat modellezése csak az el-múlt években történt meg. A stabilnak és hatékony-nak bizonyult megközelítésünk lényege, hogy min-den ekviaxiális részecskét egy külön mobil alrácsrahelyezünk. A részecskék növekedési egyenleteit ezenalrácsokon oldjuk meg, a részecskéket viszont az alrá-csokon keresztül, a rögzített globális rácson megol-dott Lattice–Boltzmann-egyenletek szerint meghatáro-zott áramlási térnek megfelelõen mozgatjuk.

ENSEMBLE projekt

Az ENSEMBLE nevû projekt célkitûzése metaanyagokelõállítása volt eutektikus önszervezõdés felhasználá-sával. Metaanyagoknak azokat a természetben általá-ban nem elõforduló anyagokat nevezzük, amelyekkülönleges tulajdonságaikat valamilyen ismétlõdõszerkezetüknek köszönhetik. Ilyen különleges tulaj-donság például a negatív törésmutató. Ahhoz, hogyezek a különleges tulajdonságok az optikai tarto-mányban lépjenek fel, az ismétlõdõ szerkezet perió-dusának összemérhetõnek kell lennie a fény hullám-hosszával. A 100–1000 nm tartományban periodikusszerkezetet kialakítható anyagok elõállításának egyikígéretes módja az eutektikus önszervezõdés felhasz-nálása. Az eutektikus megszilárdulás során az olva-dékfázisból egyszerre két szilárd fázis alakul ki. A

244 FIZIKAI SZEMLE 2018 / 7–8

Page 31: Fizikai Szemle 68/7–8 – 2018. július–augusztusgoliat.eik.bme.hu/~f.simon/publications/Media/FizSzem-20180708.pdf · okokból standard modellnek hívunk, a világot anyagi és

megszilárdulást kontrolláló két alapvetõ folyamat – akisebb hosszakat preferáló diffúzió és a kisebb görbü-leteket, azaz nagyobb távolságokat kedvelõ kapillari-tás – versengéseként a szerkezetben egy jellemzõméretskála alakul ki, az anyagi és külsõ kontrollpara-méterektõl függõ, úgynevezett Jackson–Hunt-hullám-hossz. A projekt keretében egy elméleti csoport általmeghatározott, adott optikai tulajdonságokat biztosítóeutektikus szerkezeteket valósítottunk meg, olyananyagokban, ahol a két szilárd fázis optikai tulajdon-ságai jelentõsen eltérõk. Mi számítógépes szimulá-ciókkal próbáltuk feltérképezni az adott szerkezet ki-alakításához szükséges paramétereket, míg egy kísér-leti csoport ténylegesen elõállította az adott szerkeze-tû anyagokat (8. ábra ) [28].

Irodalom15. Donald E. Stokes: Pasteur’s Quadrant – Basic Science and

Technological Innovation. Brookings Institution Press, 1997.16. László Gránásy, Tamás Pusztai, James A. Warren, Jack F. Doug-

las, Tamás Börzsönyi, Vincent Ferreiro: Growth of ’dizzy dend-rites’ in a random field of foreign particles. Nature Materials 2/2(2003) 92–96.

17. László Gránásy, Tamás Pusztai, Tamás Börzsönyi, James A. War-ren, Jack F Douglas: A general mechanism of polycrystallinegrowth. Nature Materials 3/9 (2004) 645–650.

18. László Gránásy, Tamás Pusztai, György Tegze, James A. Warren,Jack F. Douglas: Growth and form of spherulites. Physical Re-view E 72/1 (2005) 011605.

19. Gránásy László, Pusztai Tamás, Börzsönyi Tamás: A polikristá-lyos megszilárdulás térelméleti modellezése. Fizikai Szemle 55(2005) 203–211.

20. Pusztai Tamás, Bortel Gábor, Tóth Gyula, Gránásy, László:Komplex kristálymorfológiák modellezése három dimenzióban.Fizikai Szemle 56 (2006) 412–415.

21. Gránásy László: Számítógépes anyagtudomány: Tûkristályoktóla komplex polikristályos alakzatokig. Fizikai Szemle 67 (2017)403–406.

22. Andrzej Pawlak, Ewa Piorkowska: Crystallization of isotacticpolypropylene in a temperature gradient. Colloid and PolymerScience 279/10 (2001) 939–946.

23. Silvère Akamatsu, Mikaël Perrut, Sabine Bottin-Rousseau, Gab-riel Faivre: Spiral Two-Phase Dendrites. Physical Review Letters104/5 (2010) 056101.

24. Tamás Pusztai, László Rátkai, Attila Szállás, László Gránásy: Spi-raling eutectic dendrites. Physical Review E 87/3 (2013) 032401.

25. Szállás Attila, Rátkai László, Pusztai Tamás, Gránásy, László:Helikális mintázat eutektikus ötvözetekben. Fizikai Szemle 63(2013) 333–337.

26. László Rátkai, Attila Szállás, Tamás Pusztai, Tetsuo Mohri, LászlóGránásy: Ternary eutectic dendrites: Pattern formation and sca-ling properties. The Journal of Chemical Physics 142/15 (2015)154501.

27. László Gránásy, László Rátkai, Attila Szállás, Bálint Korbuly,Gyula I. Tóth, László Környei, Tamás Pusztai: Phase-Field Mo-deling of Polycrystalline Solidification: From Needle Crystals toSpherulites – A Review. Metallurgical and Materials Transac-tions A 45 (2014) 1694–1719.

28. László Rátkai, Gyula I. Tóth, László Környei, Tamás Pusztai,László Gránásy: Phase-field modeling of eutectic structures onthe nanoscale: the effect of anisotropy. Journal of MaterialsScience 52 (2017) 5544–5558.

KÉTSZÁLAS NAPÓRA– egy ritkaság Siófokon Siófok

Molnár János Albert a BME-n szerzett villa-mosmérnöki oklevelet, ismereteit késõbbmás területekkel bõvítette. 1965-tõl 2000.évi nyugdíjazásáig az olajiparban dolgo-zott. 1970-tõl oktat egyetemek elõadója-ként, az UNIDO szervezésében és a GáborDénes Fõiskolán. 1980-ban doktori dolgo-zatával társul az Országos Mérésügyi Hiva-tal Gázipari Hitelesítési Szabályzatainakelkészítéséhez. 450 oldalas könyve a nap-órák hagyományos és újdonság értékûszakismereteinek monográfiája.

Molnár János Albert

A Siófokra tervezett napóra jellegét eleve meghatároz-ta a hely sugallta alapötlet: az egyik árnyékvetõ szál avízszintes eresz és egy függõón a másik. A skála óra-vonalai az idõegyenleg nyolcasával úgy vannak egy-beszerkesztve, hogy azok látványos skálarajzot is al-kotnak. Így a helyi idõ szerint mûködõ kétszálas nap-óra alkalmas a zónaközép szerinti idõeltolódás és avalódi középidõ pontos mutatására. Azaz a napórakétszálas rendszere és egyedi kialakítása miatt is„napórás szakmai” érdekesség, európai ritkaság. Amost bemutatásra kerülõ számítási módszer némi ki-egészítéssel láncgörbe- vagy V alakú árnyékvetõ szál-hoz is használható.

Bevezetõ

Alighanem akkor kezdõdhetett az ember „homo sa-piens”-szé válása, amikor rájött arra, hogy van az idõés ennek mibenlétét kezdte firtatni. Hiszen az õsi,afrikai vagy európai barlangrajzok mozgásábrázolá-saiban, az egyiptomi piramisok, meg a stonehenge-iés egyéb csillagvizsgáló építmények hátterében az idõlétének megismerését, hatásainak firtatását fedezhet-jük fel, akárcsak a bibliai, babilóniai vagy a görögbölcsek filozófiai feljegyzéseiben. Az idõmérés ókorimódszerei és lehetõségei, meg a mindennapi élet ér-dekei az õsi napórákban találkozva öltöttek testet. Anapórák évezredeket átívelõ fejlõdése mindenkor atechnika, a tudomány és filozófia éppen elért csúcsaitmutatja, rögzíti. Így alakultak ki a különféle fajtájú ésstílusú árnyékvetõk és skálafelületek; a mûködésmó-dokat megalapozó csillagászati és matematikai isme-retek. A szoros filozófiai kapcsolat ma is jól láthatómaradványa a napórák gyakran népi bölcsességeketis közlõ feliratai és jelképes festményei.

Mára a napórák – a hagyományok ápolásán, eszté-tikai elemeik vagy szakmai érdekességeik bemutatá-sán túli – eredeti feladata, az idõ pontos mérése (és

MOLNÁR JÁNOS ALBERT: KÉTSZÁLAS NAPÓRA – EGY RITKASÁG SIÓFOKON 245

Page 32: Fizikai Szemle 68/7–8 – 2018. július–augusztusgoliat.eik.bme.hu/~f.simon/publications/Media/FizSzem-20180708.pdf · okokból standard modellnek hívunk, a világot anyagi és

nem a pontos idõ mérése!) gyakorlatilag elenyészett.

1. ábra. A napóra fényképe, a kép készítésének ideje – mint leol-vasható – nyári idõszámítás szerint május 1. 9 óra 40 perc.

Ám az újdonságokat keresõ kutatói vágy maradandó.Ennek egyik nem régmúltbeli eredménye volt HugoMichnik elzászi tanár dolgozata [7] 1923-ban az újfaj-ta, kétszálas asztali napórákról. Ezzel szinte egyidõsDrágffy Sándor érsekvadkerti földbirtokos 1925. évibejelentésére kiadott Magyar Királyi Szabadalmi leírásegy gyûrûs napóráról. 1923 óta számtalan tanulmány[l. irodalomjegyzéket] látott napvilágot eleinte a két-szálas fali napórákról, majd 1980-tól az általános hely-zetû kitérõ egyenesek vagy térgörbék, meg egy tet-szõleges helyzetû felületbõl álló „napóra” skálaszer-kesztésének matematikai csemegéirõl [1, 3]. Idõvel –ingyenes amatõr, valamint profi igények kielégítésérealkalmas fizetõs – számítógépes napóratervezõ cso-magok is megjelentek a piacon (Sonne, Shadows,Orologi Solari stb.). Ennek ellenére Európában meg-lehetõsen ritkán lehet kétszálas napórát látni. Magyar-országon nyilvántartott, mintegy 800 napóra közöttegyelõre egy kétszálas sincs. Ismereteim szerint hoz-zánk legközelebbi az olaszországi Udine város szom-szédságában lévõ 2500 lakosú Aiello del Friuli falu (ϕ= 45,87216°; λ = −13,36474°) több, mint 120 darabosnapóramúzeumában mûködik.

Egy nemrég épített siófoki társasház szerencsés mó-don olyanná lett, hogy falára nagyon kevés kiegészítés-sel érdekes, kétszálas napórát lehetett készíteni. A házépítészeti adottságai és a napóra kivitelezési lehetõsé-gei miatt csak függõleges és vízszintes árnyékvetõ szá-lak jöhettek számításba, mint szabadon választhatóalapparaméterek. Ugyanis adott az épület méternyi szé-les esõvédõ tetejének vízgyûjtõ csatornája, ami egyúttalegy kétszálas napóra vízszintes keresztszálaként is ki-válóan mûködik. A tetõ tartószerkezetének megfelelõpontjához rögzített és nehezékkel terhelt zsineg megpompás délköri árnyékvetõje lett az így kialakított, 1.ábrán látható napórának. Már csak a skála hiányzott,csak ezt kellett megtervezni és elkészíteni.

Alapadatok

A napórák skálájának tervezésére számtalan elv ésmódszer ismeretes. Ezek ugyanazokat az alapvetõföldrajzi, valamint technikai-fizikai adatokat igénylikés körülbelül azonos jellegû és tartalmú skálaadatokateredményeznek, de kényelmi és többletszolgáltatá-saikban jelentõsen eltérnek egymástól. Valamennyinapórás számítás alapvetõen a helyi idõ, a helyi kö-zépidõ és a zónaidõ közti kapcsolatok, no meg azidõpont és az idõtartam közti különbség szabatosismeretét, az ezekhez tartozó óraszög – idõszög meg-felelõ használatát igényli. Idõszakonként még a nyáriidõszámítás okozta egy órányi eltolódást is figyelem-be kell venni.

A napóra helyének földrajzi szélességi és hosszúsá-gi (ϕ, λ ) koordinátáját legegyszerûbben az internetestérképekrõl vagy kézi térképgépekrõl lehet leolvasni.Ennek eredménye a siófoki helyszínre: ϕ = 46,9046°;λ = −18,0514°.

A skála tervezéséhez még szükséges geometriaialapadat a fal elfordulása. Ezt is sokféle módon lehetmegmérni. Az adott esetben az volt a legegyszerûbb,hogy meghatároztam azt az idõpontot, amikor a Napa falat éppen merõleges irányból világítja. Ehhez améréshez jól lehetett használni a nyitott garázskapufüggõleges ajtótokjának az ellenõrzötten vízszintesaljzatbetonra vetett árnyékát és ellenõrzésképpen azideiglenes helyre rögzített függõónét is. A mérés zó-naidejébõl lehetett a Nap súrolási irányszögét, majdebbõl a fal elfordulását kiszámítani. Ehhez egy csilla-gászati adatokat számító és kellõen pontos számítógé-pes programot volt kényelmes használni. Több alka-lommal végzett megfigyelések átlaga kellõ pontosság-gal adja a fal irányát, azaz a kelet-nyugati iránytól valóelfordulás α szögét. Ez az adott helyen α = −39,5°.

Az idõ

Tudott, hogy a tengelye körül forgó Föld ellipszisalakú pályán kering a Nap körül. Kepler óta számsze-rûsíteni is tudjuk, hogy mennyivel lassabban haladnaptávolban, és napközelben mennyivel gyorsabban,mint az éves átlag. A Föld egyes helyein lévõ napóráka forgó Földrõl látszó Nap napi 24 órás periodikuspályájából adódó árnyékhelyzet alapján a mindenko-ri, valódi helyi idõ szerint mûködnek. Egy-egy adotthelyen a Nap valamely delelése és a rákövetkezõ de-

246 FIZIKAI SZEMLE 2018 / 7–8

Page 33: Fizikai Szemle 68/7–8 – 2018. július–augusztusgoliat.eik.bme.hu/~f.simon/publications/Media/FizSzem-20180708.pdf · okokból standard modellnek hívunk, a világot anyagi és

lelése közti idõtartam a Nap napi teljes körbefordulá-sához igazodóan szögmértékben számolva mindignapi 360°. Eszerint lehet szögfokokkal mérni, illetveidõszögben, óraszögben megadni a napi idõpontot.Viszont a napi delelések közti idõtartamok attól isfüggenek, hogy a Föld éves keringése során pályájamelyik részén van és éppen milyen a forgástengelyhelyzete a Naphoz képest. Hiszen az adott hely dele-léskor mindig a Nap felé fordul, miközben a Földpályamenti naponkénti elmozdulásainak követéseibeleszámítanak a Naphoz viszonyított tengely körüliforgásának idejébe. A tengely ferdesége is befolyásol-ja a napok deleléstõl delelésig tartó idejének hosszát.Így a Földrõl megfigyelt napi delelések közti idõtar-tam éves periódusidõvel folyamatosan változik, rá-adásul a tengelyferdeség miatt ehhez a változáshozmég egy további, de fél éves periódusú járulék is tár-sul. Így a valódi Nap szerinti nap hossza hol hosz-szabb, hol rövidebb a névleges napi 24 óránál.

Ezt az egyenetlenséget kiküszöbölendõ, a gyakorlatiélet igényei miatt meghatároztak egy kitalált (képzelt,közepes, kiegyenlített, fiktív) Napot a valódi helyett. Eza képzelt Nap olyan, hogy a földi egyenlítõ síkjában ésegyenletesen jár körbe az év során, mert az egyenlete-sen folyó idõt a Föld forgása szerint akarjuk mérni, azazaz egyenlítõn. Ez a kitalált Nap úgy mozog, hogy aTavaszponton való átmenetének idõpontja és helyemindig egybeesik a valódi Napéval. Azaz a valódi Napés a kitalált Nap szerinti év hossza másodpercre egyen-lõ, miközben a valódi Nap hol a kitalált elõtt jár, holmögötte. E kitalált Nap szerinti, egyenletesen múló idõaz egyes földrajzi helyekhez tartozó kiegyenlített helyiidõ, rövidebben tK helyi középidõ. A tK középidõ és a tHvalódi helyi idõ különbségét, a tK − tH = IE [idõperc]különbséget idõegyenlegnek, idõegyenlítésnek (dehelytelenül idõegyenletnek!), rajza nyomán nyolcasgörbének (netán analemmának) is nevezik. Ugyanis azidõegyenleg napi értékeit mutató függvény görbéjetöbb helyi szélsõértékkel bír, más koordináta-rendszer-beli képe meg elnyúlt 8-as alakú. Nyelvektõl és nemzetiszokásoktól függõen találkozhatunk még az EoT = −IE= tH− tK = [szögfok] vagy [idõperc] értelmezésekkel,meg az idõfajták esetén a polgári idõ, civil idõ, óraidõ,idõszög stb., illetve ezek latin, német, angol stb. válto-zatlan (fordítás nélküli) átvételével is.

Az országos idõjelzések (rádió, mobiltelefon) azónaközépnek kinevezett földrajzi helyek egyenlete-sen folyónak tekintett középideje szerinti tZ zónaidõ[óó:pp:ss] értékét közlik. Eszerint járnak a gépi órák,az ennek megfelelõ idõpontokat bármikor meg tudjukhatározni. De mi a helyi idõ értéke?

A napóra helyének földrajzi hosszúsága és a helyzónaközepe (nálunk Görlitz, λZ = −15°) közti különb-ségbõl számítható a

földrajzi hosszúságkülönbség. Itt a dλ szögtávolság, a

dλ = λ − λ Z = 18,0514 − 15,00 = 3,051 [szögfok] ≈

≈ 4 3,051 = 12,206 [idoperc]

napóra λ és a zónaközép λZ helye ugyancsak fokok-

ban mérve. Ebbõl a szögtávolságból az az idõkülönb-ség adódik, amennyivel több a napóra helyén a τHvalódi helyi idõ a zónaközép valódi helyi idejénél (haa napóra a zónaközéptõl keletre van). Ez a különbségegyébként megegyezik a napóra helyének és a zóna-középnek megfelelõ középidõk különbségével is.Emiatt szokás olyan napóraskálát is alkalmazni,amelyrõl ezzel a hosszúságkülönbséggel javított zóna-közép szerinti középidõt lehet leolvasni. Ilyenkorennyivel kisebb számok kerülnek a skálavonalra.Másként fogalmazva, ha a keleti λ ≥ 15°, akkor a ská-lavonalak az óramutató járásával ellentétes iránybakissé elfordulnak és picit az óravonalak szögkülönb-ségei is módosulnak. Ha a zónaközép középidejéhezhozzászámítjuk az idõszerû naptári napnak megfelelõIE [perc] idõegyenleg értékét, megkapjuk a zónaközéptH helyi idejét. Ha ehhez még hozzávesszük a földrajzihosszúságkülönbségnek megfelelõ dλ idõkülönbsé-get, a napóra helyén érvényes, az adott tZ zónaidõnekmegfelelõ valódi helyi idõt kapjuk. A Nap helyzetadatainak (m magassági szög, Az irányszög) megha-tározásához ezt a tH helyi középidõt kell még átszá-mítani a 4 perc/fok váltószámmal idõszögértékké. Avalódi helyi idõnek megfelelõ τH óraszög esetén azt isfigyelembe kell venni, hogy számítástechnikai kénye-lem miatt elõírt feltétel a déli 12,00 órának megfelelõτ (12) óraszög = 0,00°. Így

Az IE korrekciót is tartalmazó napórák skálavonalai

τ H = 15 (tZ − 12,00) IE − dλ [fok,ff].

már nem egyenesek, mint a helyi idõt mutatóké, ha-nem elnyújtott 8-asra hajazó, vagy „S” alakú görbékadódnak óravonalként.

A tervezés

A valódi Nap látszólagos napi pályaívének meghatá-rozásához – és így a skálaszámításokhoz – ismernikell a Nap δ deklinációértékét is. Az IE [°] és δ [rad]pontos számítására hosszú – meg még hosszabb kép-letek ismeretesek [8]. A napóra tervezéséhez használtkellõ pontosságú változatok:

valamint

IE = 0,0000075 0,001868 cosP − 0,032077 sinP −

− 0,014615 cos(2 P ) − 0,040849 sin(2 P ) [° ],

ahol

δ = 0,006918 − 0,399912 cosP 0,070257 sinP −

− 0,006758 cos(2 P ) 0,000907 sin(2 P ) −

− 0,002697 cos(3 P ) 0,00148 sin(3 P ) [rad],

P = 2 πDi − 1

365[rad], Di = 1, 2, …, 365,

itt Di a nap évbeli sorszámát jelöli, Di (január 1.) = 1.

MOLNÁR JÁNOS ALBERT: KÉTSZÁLAS NAPÓRA – EGY RITKASÁG SIÓFOKON 247

Page 34: Fizikai Szemle 68/7–8 – 2018. július–augusztusgoliat.eik.bme.hu/~f.simon/publications/Media/FizSzem-20180708.pdf · okokból standard modellnek hívunk, a világot anyagi és

A napi, illetve órai adatokat ezeknél

2. ábra. A térbeli elrendezés vázlatrajzai.

Nap

dél

észak

függõleges szál függõleges árnyék

árnyékvízszintes szál

y

T

E

O

Az

m

GV

GF

z

x

Px

Pyaj

P x,y( )

vízszintes szál

Nap

GF

a

Az – a x

GV

észak

kelet

falT

E

O

P

z

pontosabban megadó képletekre csak a faltájolásának méréséhez lehet szükség. Adeklináció és az idõegyenleg éves lefutását– tájékoztatásként – az interneten mellékelt(http://fizikaiszemle.hu/extra/1807molnar.xlsx) Excel munkalap ábrája mutatja.

A kétszálas napóra mûködésének vázla-tos megismeréséhez és a skála tervezésé-hez kiindulásul egy olyan derékszögûkoordináta-rendszert vegyünk fel, amely-nek függõleges, felfelé pozitív y és a víz-szintes, jobbra pozitív x tengelye (és így azorigója is) a fal síkjában van. Az x tengelymagassági helyzetét úgy kell megválaszta-ni, hogy az a fallal párhuzamos és a víz-szintes árnyékvetõ szállal azonos magas-ságban legyen. A függõleges szál helyzetekijelöli a skálafelület azon szélességét, amia szál árnyékfelfogó síktól (a faltól) valótávolságából és a Nap vízszintes irányúpályaívébõl határozható meg. A gyakorlat-ban a feladat éppen fordított volt: a falskálafelületként mûködõ szélességéhezkellett választani a szál faltól való távolsá-gát. A Nap emelkedési mozgásából és azeresz helyzetébõl adódó árnyékvándorlásmértéke szerencsére kellõ összhangbanvolt a falrész felhasználható méreteivel. Afelvett koordináta-rendszer falra merõlegesz tengelye illeszkedik a függõleges szál-hoz, amelynek faltól való távolsága azadott esetben z = GF = 44,7 cm. Az eresz-csatorna árnyékvetõ éle a faltól z = GV =119,0 cm távolságra és a talajszint felett~266,0 cm magasságban van. A rendszergeometriai elrendezésének térbeli vázlatátés vízszintes vetületi nézetét a 2. ábra mu-tatja.

A számítások

A skála számítási munkájához célszerû Excel táblávaldolgozni. Nemcsak a könnyen használható, beépítetttrigonometriai és egyéb függvényei, hanem a jól ke-zelhetõ rajzolási lehetõségei miatt is. A használt csilla-gászati és ábrázoló geometriai összefüggések részlete-zéseit, meg egyebeket is az irodalomjegyzék szerintimûvek tartalmazzák.

Ismeretes, hogy a Nap vízszintes síkban értelmezettAz irányszögét a

képlettel határozhatjuk meg. A skálapontok sorozatos

tgAz = sinτsinϕ cosτ − cosϕ tgδ

számításaihoz érdemes a

tg⎛⎜⎝

⎞⎟⎠

Az2

= cosm cosϕ sinδ − sinϕ cosδ cosτcosδ sinτ

képlettel számolni, mert az inverz tangens függvénytöbbértelmûsége miatt a végeredményt tekintve ezzellesz kevesebb gondunk.

A Nap magassági szögét a

képlettel számoljuk [9].

sinm = sinϕ sinδ cosϕ cosδ cosτ

Az elõkészítés után lássuk a szálárnyékok P (x,y, t )metszéspontjának koordinátáit, azaz a skálapontokat.Az elvben végtelen hosszú, függõleges árnyékvetõ szálfalon képzõdõ árnyéka egész nap függõleges (és ön-magával párhuzamos) marad, de a Nap vízszintes moz-gásából adódóan balról-jobbra vándorol. Az árnyékvo-nal mindenkori vízszintes x koordinátáját az

összefüggés adja.

x = GF tg Az (τ ) − α

248 FIZIKAI SZEMLE 2018 / 7–8

Page 35: Fizikai Szemle 68/7–8 – 2018. július–augusztusgoliat.eik.bme.hu/~f.simon/publications/Media/FizSzem-20180708.pdf · okokból standard modellnek hívunk, a világot anyagi és

A vízszintes szál árnyékára az

és a −∞ < x < ∞ összefüggések igazak, amelyek a 2.

y =−GV tgm

cos(Az − α )< 0

ábrából származtathatók.Az óravonalak Z skálaszögeire ugyan közvetlenül

nincs szükség, de többféle módon is meghatározhat-juk az idõegyenleges korrekciók alapját képezõ skála-egyenesek szöghelyzetét. Az Excel miatti numerikusmódszer szerint a skálavonal két ismert pontjáhoztartozó koordinátákból a tgZ = Δx /Δy képlettel szá-molhatunk. Analitikusan a

képleteket használhatjuk, ahol τ a skálavonalhoz tar-

tgZ =GF cosϕGV cosα

tgτ1 sinϕ tgα tgτ

,

xT = GF tgα ,

yT = GVtgϕ

cosα

tozó idõszög, T a fiktív döféspont. A március vége –október vége közti nyári idõszakban a napóra egyórával kevesebbet mutat, mint például a mobiltelefonórája. Ezt egyszerûen, kettõs feliratozással lehet ki-egyenlíteni.

A munkalap

Ha csak az órás-félórás skálavonalak egyeneseit akar-juk felrajzolni, elég lenne például a deklinációs szögkét szélsõértékéhez tartozó pontokkal foglalkozni.Mivel az idõegyenleges korrekció az egyenes óravo-nalakat elnyújtott „S” alakú (vagy 8-asra hajazó) gör-békké formálja, ezekhez a skálavonalak pontjait mársûrûbben kell kijelölni.

A skála óravonalainak koordinátáihoz a legegysze-rûbb út, ha elõször az év egyes Di napjaihoz (1 ≤ Di ≤365, ΔD ~ 5–10 nap) tartozó δi Nap-deklináció és IEiidõegyenleg napi átlagos értékét határozzuk meg. Eztkövetõen az egyes napok tj idõpillanatbeli skálavona-laihoz (célszerûen 4 ≤ j ≤ 20, Δt ~ 15-60 perc) a meg-felelõ τij (Di, IEi, λ stb.) óraszögeket, valamint a Napezekhez tartozó mij magassági és Azij irányszögeitszámoljuk ki. Majd ezeket felhasználva a szálak ár-nyékának metszéspontjait számoltatjuk. Végezetülegy-egy t óravonal köré a zónaidõ szerint rajzolandónyolcas görbe P(t,x,y )ij pontjainak koordinátáit kellmeghatározni a (τ,m,A ) értékcsoportra való hivatkozá-sokkal, majd ugyancsak egységenként kezelve másol-gatni. A feltöltött kéthasábos (x,y ) és a háromhasábos(τ,m,Az ) értékcsoportokat célszerû együtt kezelni.

Ha elkészült az óravonalak P (x,y ) táblázata, jöhet aválogatás a Zodiákus hiperboláinak pontjaihoz. Ameglévõ részbõl kell összeválogatni a kívánt dekliná-ciókhoz tartozó értékpárokat és a hivatkozásaikatolyan elrendezésû táblázatba kell összerendezni, ami

az Excel rajzolási igényét figyelembe veszi. Eme napivonalak Nap deklinációjától függõ sûrûsége erõsenváltozó, ezért általában csak havi-kétheti gyakoriságúidõközökkel érdemes felrajzolni. Még a nem túl sûrûés feliratozott íves óravonalak is nehezen értelmezhetõkusza látványt eredményezhetnek. Ezért én csak a kétállatövi jegyhez igazodó skálavonalat, azaz a téli ésnyári napforduló szélsõ íveit és a tavaszi/õszi napéj-egyenlõség egyenesét tettem a falra. További lehetõ-ségként az Állatöv nevezetes osztáspontjaihoz tartozó±(23,45; 20,31; 11,73) és a 0,0 fokos deklinációk hétfolytonos görbéjét, vagy a hónapok kezdõnapjáhoztartozó 12 görbének csak az óravonalakhoz tartozópontjait érdemes rajzoltatni. Némi kiegészítõ adatokmiatt így a rajzfelületen körülbelül 25–30 görbe lesz.

Az egyes órákhoz tartozó P (x,y ) oszloppárokategyenként kell felvenni a tervezési menüfül adatme-zejébe. Nyilván csak idõ és türelem kérdése (az Excelbírja, rajzolási tudománya elegendõ és jól is használ-ható…), hogy az egész értékû skálavonalak melléakár további fél- vagy negyedórás osztásvonalak ke-rüljenek. A hiperbolákat is ugyanígy egyenként kellbeiktatni a tervezési menüfül adatmezejébe.

A skálakép tervezésekor az év egészére, elsõ, illetvemásodik felére vonatkozó skálarész óravonalainakadattartományát a nyomtatás elõtt ki kell választani.Ehhez a teljes évi táblázat megfelelõ soraira kell korlá-tozni a rajzolandó részt. A váltást a képernyõn úgy le-het kényelmesen kivitelezni, ha egy külön oszlopbafeltételtõl függõként másoltatjuk a korrekcióra szánt IEidõegyenlegrészt. Ehhez a váltáshoz a HA() logikaifüggvényeket lehet jól használni, akárcsak a választha-tó helyi és a zónaidõ közti váltáshoz. A 3. ábrán lát-ható skála esetén az azonos napi idõbeli árnyékmet-széspontok lefelé haladnak a december–június köztielsõ félévi idõszakban, miközben a Nap deklinációjafolyamatosan növekszik. A 4. ábra skálavonalain azazonos idõbeli árnyékmetszéspontok felfelé másznaka június–december közti második félévi idõszakban,amikor a Nap deklinációja csökken. A már említett –http://fizikaiszemle.hu/extra/1807molnar.xlsx webhe-lyen mellékelt – Excel munkalap táblázata a nyolcaskorrekció jellegére kérdez: milyen legyen? igen – nem– 1. félév – 2. félév – délben válaszok közül szándé-kunk szerint kell választani. Választani kell a helyi idõ –zónaidõ szerinti skálázási lehetõségbõl, továbbá különkell nyilatkozni a félórás óravonalak kívánatos alakjárólis: egyenes vagy nyolcas legyen? Mindez, némi „mace-rás piszmogást” igényel, de ettõl az érdeklõdõt megkí-méli a mellékletként szereplõ munkafüzet, csak az újalapadatokat kell beírni. Ezután mozgósítva az Excelrajzoló képességeit, a különálló munkalapon, léptékhe-lyes, feliratozott stb. skálát kapunk. Ezt papírra nyom-tatva rajzsablonként használhatjuk. Ám a fal feliratozá-sát és díszítését gyorsabb és kényelmesebb közvetle-nül, csak a falon megvalósítani.

A képernyõn már elfogadhatónak tûnõ skálarajz kétcsapdát tartalmaz! A fõ gondot az okozhatja, hogy azalkalmanként beírt (új) alapadatoktól függõen a skálaegyes óravonalaihoz tartozó néhány (x,y ) értékpár 1-1

MOLNÁR JÁNOS ALBERT: KÉTSZÁLAS NAPÓRA – EGY RITKASÁG SIÓFOKON 249

Page 36: Fizikai Szemle 68/7–8 – 2018. július–augusztusgoliat.eik.bme.hu/~f.simon/publications/Media/FizSzem-20180708.pdf · okokból standard modellnek hívunk, a világot anyagi és

tagját ideiglenesen törölni kell, mert egyes görbék ön-

3. ábra. Az I. félévi skála rajza (a fényképen ezüst színû).

január

február

március

április

május

május

június

december

XIII

XIV

XII

XI

X

IX

VIIIVIII

VII

9

10

1112 13

nyáriidõszámítás

4. ábra. A II. félévi skála rajza (a fényképen bronz színû).

júliusjúlius

június

augusztus

szeptember

október

november

december

XIII

XIV

XII

XI

X

IX

VIIIVIII

VII

9

10

1112 13 14

nyáriidõszámítás

metszõdését (arcsin, arctg stb.! és elõjeleik miatt) leg-egyszerûbben így lehet megszüntetni. Alkalmazhatómódszer az is, ha egyes vonalak rajzolási tartományátalkalmanként módosítjuk. A másik gond, hogy a (gép+ képernyõ + nyomtató) rendszer ritkán van pontosrakalibrálva. Ezért a papírra szánt, rajzsablonként hasz-nálandó nyomat x és y tengelyeinek léptékazonossá-gát ellenõrizni kell. Szükség esetén a kinyomtatottskálarajz méretarányát még a képernyõn kell megfele-lõre, esetleg több lépésben is módosítani.

Magától értetõdik – de azért érdemes külön is meg-jegyezni –, hogy mindezek a számítások és lehetõsé-gek használhatók a GV = GF esetben is, azaz egy ha-gyományos, elvben gnomon formájú árnyékvetõt al-kalmazó fali napóra meglehetõsen egyedi formájúmegvalósításakor is.

További érdekes kiegészítés (alkalmazási lehetõ-ség), ha az árnyékvetõ eresz helyett valódi árnyékve-tõ szálat (vékony, egyenes fémpálcát, betonvasat vagykötelet, láncot) használunk. Kisebb méretek esetén avégeinél megtámasztott fémpálca jó lehetõség, de habehajlik, érdemes ténylegesen behajló, jól látható(parabolával közelíthetõ) láncgörbét alkalmazni. Ek-kor a függõleges szálnál a falhoz közelebb, azzal pár-huzamosan felfüggesztett lánc változó magasságamiatt az árnyékpontok korábban kiszámított y koordi-nátáját egy +p4U paraméterû járulékos másodfokú tag-gal kell növelni:

y = −GV tg(m )

cos(Az − α )p4U x 2.

Ha a lánc két végpontja közti távolság H [cm], a beló-gása B [cm], akkor

Ha a lánc közepét kellõ módon megterheljük, úgy az

p4U = 4 B

H 2

⎛⎜⎜⎝

⎞⎟⎟⎠

1 −GV

GF

2

.

eredetileg vízszintes (y = 0 koordinátájú) árnyékvetõszál helyett az origóhoz illeszkedõ V alakot kapha-tunk. Ennek hatását egy +p4V paraméterû elsõfokújárulékos taggal kell figyelembe venni:

és ekkor

p4V = 2 BH

⎛⎜⎜⎝

⎞⎟⎟⎠

1 −GV

GF

Ezeket a változtatásokat a csatolt Excel táblázat U

y = −GV tg(m )

cos(Az − α )p4V x .

skála és V skála nevû munkalapjai tartalmazzák.

Zárszó

A siófoki kétszálas napóra terv szerint készített skála-vonalai papíralapon jelentek meg közvetlenül látható-fogható A4 méretû lapon, továbbá az Excel generáltavektoros formában digitális adathordozón. Ennek

250 FIZIKAI SZEMLE 2018 / 7–8

Page 37: Fizikai Szemle 68/7–8 – 2018. július–augusztusgoliat.eik.bme.hu/~f.simon/publications/Media/FizSzem-20180708.pdf · okokból standard modellnek hívunk, a világot anyagi és

alapján készült egy nagyítás 1:1 léptékû papírlap for-májában. Ezt kellett a falra másolni, majd akrilfesték-kel felfesteni, hogy az 1. ábra szerinti fénykép elké-szülhessen.

Irodalom1. Dominique Collin: Theorie surle cadran solaire bifilaire vertical

declinant. Journal of the Royal Astronomical Society of Canada94/3 (2000 95–111.—: Les cadrans solaires bifilaires a gnomons rectilignes quel-conques. Société Astronomique de France, Calais (2002)—: Theorie surle cadran solaire bifilaire généralisé. Observa-tions & Travaux 55 (2003) 12–31.—: Vers une finalisation des cadrans bifilaires a fils rectilignes.Société Astronomique de France, Nice (2005)

2. Peter Duffett-Smith: Practical astronomy with your calculator.Cambridge, Univerity Press, Third edition 1988, reprint 1995.

3. G. Ferrari: A curious property of bifilar sundials. The Compendi-um 7/4 (2000) 11–16.

4. Rafael Soja Gaya: Application de la méthode analitique aux tra-cés de cadrans solaires bifilaires. Analemma 32

5. M. A. Hacar: Relojes de sol bifilares. Analemma 2 (1991) 16–19.6. Lothar Loske: Die Sonnenuhren. Springer Verlag, Berlin (1970)7. Hugo Michnik: Theorie einer bifilar-sonnenuhr. Astronomische

Nachrihten 217/5190 (1923) 81–90.

8. Jean Meeus: Astronomical formilae for calculatos. Willmann-Bell, Richmond (1988)—: Astronomical algorithms. Willmann-Bell, Richmond (1991)

9. Molnár János: A napórákról. Kairosz kiadó, Budapest (2012)10. J. Moreno Bores: Relojes bifilares horizontales. Analemma 18

(1997) 3–8.—: Ampliacion del concepto de relojes bifilariesa relojes de dossuperficies… Analemma 16 (1996) 3–7.

11. B. Rouxel: Cadrans bifilaires: étude géométrique. Cadran Infode la CCS de la SAF 9 (2004)

12. F. W. Sawyer: Bifilar gnomonic. Journal of the British Astrono-mical Association 88/4 (1978) 334–351.—: Of analemmas, mean time and the sundial. SciathericNotes North American Sundial Society Press (1998)

13. Henz Schumacher: Sonnenuhren. Callwel (1985)14. Arnold Zenkert: Fascination sonnenuhr. VEB Verlag Technik,

Berlin (1984), Verlag Harri Deutsch, Thun (1995)

Hasznos internetcímekhttp://2.iap.fr; http://astro.unibas.ch; http://astrosurf.com;http://cadrans/_solaires.scg.ula; http://nass.com; http://val.ca;http://cadrans-solaires.org; http://de-zonnewijzerkring.nl;http://gnomonica.org; http://infraroth.de; http://home2.scarlet.de;http://members.kabsi.at; http://members.ping.at; http://sonne.de;http://orologisolari.it; http://shadowspro.com; http://wikipedia.hu;http://sundials.co.uk; http://sundials.org; http://sundialsoc.org.uk

TÍZ ÉVES AZ ELTE FIZIKA DOKTORI ISKOLA»FIZIKA TANÍTÁSA PROGRAMJA«GYAKORLÓ FIZIKATANÁROK SZÁMÁRA

A tanulmány elkészítését a Magyar Tudományos Akadémia Tan-tárgy-pedagógiai Kutatási Programja támogatta.

Juhász András az ELTE Anyagfizikai Tan-szék nyugalmazott egyetemi docense. El-sõsorban fizikatanár-jelöltek szaktárgyi ala-pozásában és szakmódszertani képzésébendolgozott. Kutatómunkája is ehhez, vala-mint az anyagtudományhoz kapcsolódott.Több mint 160 tudományos cikke mellettszak- és ismeretterjesztõ könyvek (társ)-szerzõje, szerkesztõje. A Fizika Tanításadoktori program 2007-es megindításánakkezdeményezõje. 2016 óta az MTA–ELTEFizika Tanítása Kutatócsoport tagja.

Tél Tamás az ELTE-n szerzett fizikus diplo-mát 1975-ben. Azóta – külföldi vendégku-tatói tartózkodásaitól eltekintve – az ELTEElméleti Fizikai tanszékén dolgozik külön-bözô beosztásokban. Kutatási témái anemegyensúlyi rendszerektôl a klímadina-mikáig terjednek. 2007 óta vezeti a Fizikatanítása doktori programot, 2011 óta azMTA–ELTE Elméleti Fizikai Kutatócsopor-tot. 2016 óta az MTA-ELTE Fizika TanításaKutatócsoport vezetõje.

Juhász András – ELTE Anyagfizikai Tanszék

Tél Tamás – ELTE Elméleti Fizikai Tanszék és MTA–ELTE Elméleti Fizikai Kutatócsoport

Az ELTE Fizikai Intézet Tanácsa és a Fizika DoktoriIskola 2007 tavaszán elfogadta a Fizika tanítása prog-ramot, mint a három korábbi kutatási programmal(anyagtudomány, részecskefizika és statisztikus fizi-ka) egyenértékû, önálló doktori diszciplínát. Ezzelkifejezte, hogy a szakmódszertani képzést éppenolyan fontosnak tarja, mint a három nagy kutatásiterület bármelyikét. A 2007–2008 tanévben szervezettformában elindult a fizikatanári doktori program. Acél olyan, a fizika tudományában és a szakmódszertan

területén egyaránt jól tájékozott szakemberek képzé-se, akik képesek a fizikatanítás megújítására, a diákokérdeklõdésének felkeltésére, a magas szintû tehetség-gondozásra, tananyagfejlesztésre, új tanítási progra-mok és módszerek kidolgozására [1]. A célok – azeredményesnek tûnõ elsõ lépések megtétele után is –változatlanul aktuálisak. A folytatás szempontjábólminden bizonnyal hasznos lehet az eltelt tíz év mérle-gének elkészítése, a tapasztalatok összegzése. Azalábbiakban összefoglaljuk a doktori program doku-mentált eredményeit, a 10 évnyi mûködés tapasztala-tait, majd a képzésben résztvevõk körében végzettfelmérésünk eredményét ismertetjük.

JUHÁSZ ANDRÁS, TÉL TAMÁS: TÍZ ÉVES AZ ELTE FIZIKA DOKTORI ISKOLA »FIZIKA TANÍTÁSA PROGRAMJA« 251

Page 38: Fizikai Szemle 68/7–8 – 2018. július–augusztusgoliat.eik.bme.hu/~f.simon/publications/Media/FizSzem-20180708.pdf · okokból standard modellnek hívunk, a világot anyagi és

A doktori program „iskolaszerû” mûködése,

1. ábra. A Fizika Tanítása Program által szervezett konferenciák referált kiadványai.

dokumentált adatai, eredményei

A Fizika tanítása doktori programra az eltelt 10 évalatt 56 fõ iratkozott be és 43 fõ folytat/folytatott kitar-tó tanulmányokat, közülük 39-en gyakorló középis-kolai tanárként, 4-en a BSc képzésben dolgozó okta-tóként. A program nyitott minden hazai fizikatanár ésa határon túl magyar nyelven fizikát tanító pedagóguselõtt. Doktoranduszaink közel 60%-a korábban azELTE-n végzett, a többiek nagy vidéki egyetemeinken(Debrecen, Pécs, Szeged), illetve a határon túl diplo-máztak. Az elmúlt tíz évben 13 jelölt védte meg sike-resen disszertációját és nyert PhD fokozatot, 5-enközvetlenül a védés elõtt állnak, 16 fõ disszertációjabefejezésén dolgozik.

A tanári doktori képzés mûködését alapvetõen azELTE Doktori Szabályzata határozza meg, de néhányvonatkozásban különbözik más doktori programok-tól. Ilyen lényeges különbség, hogy doktorandu-szaink aktív tanárok, akik iskolai munkájuk mellettvégzik tanulmányaikat és kutatómunkájukat. Mivelmunkaviszonyuk van, ösztöndíj-támogatást nem kap-nak, sõt a külsõ kutatóhelyekre vonatkozó mérték-ben a részvétel térítésköteles is. A doktori képzés kétmunkaterületre koncentrál: az elsõ négy félévben arésztvevõk korábbi szakmai ismeretei felfrissítéséreés látókörüknek a mai fizika válogatott fejezeteiveltörténõ bõvítésére helyezzük a hangsúlyt. Ezt sze-meszterenként 4 (összességében 4 × 4 = 16), speciáli-san tanárok számára szervezett elõadássorozat ésszámos, a fizika tanításához kapcsolódó egyéni fela-dat (szakanyagok, számítógépes segédanyagok,programok készítése, oktatási kísérletekben történõrészvétel stb.) szolgálja. A doktori iskola hallgatóiminden hónapban egy szombati napot elõadásokhallgatásával töltenek az egyetemen. A kurzusok té-máit úgy igyekeztünk összeállítani, hogy a fizikataní-tás alapvetõ kérdései, módszertani lehetõségeinekáttekintése mellett, a fizika új, a tanárok és a középis-kolás diákok számára is érdekes területeibe adjunkbetekintést. A 4 × 4 kurzusról a doktori program hon-

lapján, http://csodafizika.hu/fiztan található részlete-sebb tájékoztatás. Ugyanitt, a Közkincs-Hallgatókszakmai anyagai (http://csodafizika.hu/fiztan/kozkincs/szakmhallg) menüpont kínál válogatást adoktoranduszok elektronikus oktatási anyagaiból, ésugyanígy „közkincs”-ként letölthetõk a hallgatókmagyar és angol nyelvû publikációi.

A doktori programhoz kapcsolódóakkreditált tanártovábbképzés

A doktori képzés elõadássorozatára alapítva már négyéve mûködik a félévente 60 órás akkreditált tanárto-vábbképzõ tanfolyamunk: http://pedakkred.oh.gov.hu/PedAkkred/Catalogue/CatalogueDetails.aspx?Id=5268.

Az erre jelentkezõ tanárok ugyanazokat az elõadá-sokat látogatják, mint a doktoranduszok, de a tanfo-lyam teljesítésének követelményei természetesen ala-csonyabbak a doktori követelményeknél. Azok, akika tanfolyam tapasztalatai alapján úgy döntenek, hogya teljes doktori képzésre átjelentkeznek, megtehetik.A rendszer – úgy tûnik – mûködik. Az utóbbi évekdoktoranduszai közül többen a tanártovábbképzõtanfolyamon kaptak kedvet a munka folytatásához.

A tudományos kutatómunka gyakorlata

A doktori munka második része a doktorandusz általvégzett tudományos igényû kutató-fejlesztõ munka. Emunka során kapott eredményeket a doktoranduszelõször a szakmai közönségnek szóló angol és ma-gyar nyelvû publikációkban kell megjelentesse, majdösszegezve, tézisekbe szervezve disszertációban isösszefoglalja. A jelölt kutatómunkáját tudományosfokozattal rendelkezõ egyetemi oktató, témavezetõirányítja és segíti. A gyakorló tanárok számára, aziskolai tanítás mellett, a doktori munka kutatási részea nehezebb. Ez a munkafázis még a legjobbak eseté-ben is évekre elhúzódik, aminek – sajnos – érthetõvelejárója lehet a végleges lemorzsolódás is. Az okokbizonyára összetettek. Az elsõ nehézséget az okozza,

252 FIZIKAI SZEMLE 2018 / 7–8

Page 39: Fizikai Szemle 68/7–8 – 2018. július–augusztusgoliat.eik.bme.hu/~f.simon/publications/Media/FizSzem-20180708.pdf · okokból standard modellnek hívunk, a világot anyagi és

hogy a jelöltek napi iskolai munkájától eleinte távoláll a sajátos protokollal rendelkezõ kutatómunka [2,3]. Aki mégis vállalja, az az egész folyamat és a tézi-seik megalkotása során megérti, hogy mit jelent atantárgy-pedagógiai kutatás, egy új tudományos ered-mény elérése. A megvédett disszertációk és az alapju-kat adó tézisek a honlapon megtalálhatók.

A kutatási munka másik nehézsége, hogy a tanároknehezebben értelmezik az egyéni kutatómunka szo-ros és lényegi kapcsolódását a hazai és nemzetköziszakmai közösséghez. Az egyéni eredmények értékéta szakmai közösség elõtti bemutatás, illetve az utóbbirészérõl történõ elfogadás adja meg. E megmérettetéslegfontosabb fórumát a referált külföldi és hazai fo-lyóiratokban, lektorált kiadványokban megjelenõpublikációk jelentik.

A doktori iskola része, hogy a tanárok rendszere-sen részt vesznek hazai és nemzetközi konferenciá-kon, ahol eredményeik megismertetésén túl szakmaikapcsolataikat is bõvíthetik. A Doktori Program azelmúlt években négy nemzetközi konferenciát szerve-zett, illetve vett rész annak szervezésében. A magyarnyelvû konferenciákon a hazai fizikatanárok mellett ahatárokon túli területekrõl is jelentõs számban jöttekkollégák, az angol nyelvû konferencia résztvevõinekköre egész Európára kiterjedt. A konferencia-elõadá-sok referált írott változata (1. ábra ), mintegy 2000oldalnyi összterjedelemben, nyomtatásban is megje-lent, és a Doktori Program honlapjáról elektronikusformában is letölthetõ.

A Fizika Tanítása Doktori Programa hallgatói vélemények tükrében

A tíz éve folyó doktori program értékelése szempont-jából legfontosabb tényezõ a résztvevõk véleménye.Ennek összegyûjtésére minden beiratkozott résztve-võnek – függetlenül attól, hogy hol áll a képzési fo-lyamatban – 25 kérdést tartalmazó elektronikus kér-dõívet küldtünk ki, és kértük annak anonim kitöltését.A kérdések a doktorandusztanárok iskolai helyzeté-nek felmérésére, a doktori program tartalmi vélemé-nyezésére, a vállalt terhelés mértékére, a doktori kép-zésben végzett munka tényszerû eredményeinek ösz-szegzõ felmérésére és a képzés hasznosságának szub-jektív megítélésére vonatkoztak. Kérdõívet az 56 cím-zett 70%-a (39 fõ) küldte vissza. Az alábbiakban ezeklényegi tapasztalatait foglaljuk össze. A teljes kérdõíva kapott válaszok összesítésével a http://csodafizika.hu/fiztan/kerdoiv.pdf linken tekinthetõ meg.

A doktori munka motivációja ésannak iskolai elismerése

A válaszok szerint jelentkezéskor elsõdlegesen aszaktárgyi ismeretek felfrissítésének igénye motiváltaa kollégákat. Emellett szerepet játszott a tanári életpá-lya-elõmenetel és a szakmódszertani kutató-fejlesztõmunka iránti érdeklõdés is. A tanári életpályán törté-

nõ elõmenetel nem közvetlenül kapcsolódik a doktoricím megszerzéséhez. A kérdésekre kapott válaszok-ból kitûnik azonban, hogy doktoranduszaink munka-helyi elõmenetelének sikeressége meghaladja az or-szágos átlagét. Így a doktori program résztvevõi közül2 PhD fokozattal rendelkezõ kollégánk lett kutatóta-nár, 7-en mestertanári címet nyertek, 9-en pedig apedagógus I.-bõl átléptek a II. fokozatba. Ennek való-színû oka, hogy a doktori munka jól dokumentáltszakmai eredményei megkönnyítik az elõrelépéshezszükséges pályázati portfolió elkészítését.

A doktori munka elismerése az iskolavezetés, illet-ve kollégák részérõl nem ennyire egyértelmûen pozi-tív. Az iskolavezetés 11 kolléga esetén közömbösentudomásul vette tanára jelentkezését a doktori kép-zésbe, 12 fõnél erkölcsileg támogatást adott, bíztatta,míg 15 résztvevõ esetén a képzés költségeihez anya-gilag is hozzájárult. A tanártársak részérõl iskolánbelül mutatkozó szakmai elismeréssel kapcsolatban avégzettek általában pozitívan nyilatkoztak, kiemelve,hogy a közvetlen kollégák szakmai kérdésekben ad-nak véleményükre, gyakran kérnek konkrét kérdé-sekben tanácsot, segítséget. (Ezzel szemben voltolyan kolléga is, aki közvetlen kollégái részérõl már ajelentkezéstõl szakmai féltékenységet érzékelt.)

A doktori kurzusok tartalmi megítélése

Több kérdés is foglalkozott a vizsgaköteles kurzusoktartalmi véleményezésével. A résztvevõktõl elõszörazt kértük, hogy az iskolában megszokott ötfokozatúosztályozási rendszer szerint, összességében minõsít-sék a négy féléves elõadássorozat 16 kurzusát. Ehhezfontos szempontként adtuk meg, hogy a kurzusok té-máit azzal a szándékkal állítottuk össze, hogy a részt-vevõk fizikaszemléletét szélesítsék, de egyúttal aziskolai munkában is hasznosíthatók legyenek. A be-küldött 39 válasz osztályzatainak átlaga 4,6.

Ezután egy-egy pontban külön rákérdeztünk, van-eolyan tárgy, amit feleslegesnek és így elhagyhatónakéreznek, illetve mely tárgyat/tárgyakat tartanak ki-emelkedõen jónak, hasznosnak. Végül megkérdeztük,van-e olyan témakör, aminek feldolgozását a jelenlegitematikában hiányolják és a jövõben feltétlenül szük-ségesnek és hasznosnak tartanák. Itt, mivel a képzésiidõ korlátozott, kértük, nevezze meg azt is, hogy az újtémakört melyik jelenlegi tantárgy helyett javasolnák.

A kérdéscsoportra adott válaszok egyértelmûenmutatják, hogy a jelenlegi kurzuskínálatban a legnép-szerûbbek azok a modern fizikai témák (például kör-nyezeti áramlások fizikája, relativitáselmélet alapjai,kaotikus mechanika ), illetve interdiszciplináris vonat-kozású témák (például fizika a biológiában, energia-termelés és környezet, kooperatív jelenségek – inter-diszciplináris vonások ), amelyek a korábbi évekbenaz egyetemi tanárképzés tanterveiben nem vagy csakkis súllyal szerepeltek. A válaszadók többsége nemnevezett meg kihagyható tárgyat. A kérdéscsoportraadott válaszok tükrében tehát azt a következtetéstlehet levonni, hogy a tantárgyi struktúra lényeges

JUHÁSZ ANDRÁS, TÉL TAMÁS: TÍZ ÉVES AZ ELTE FIZIKA DOKTORI ISKOLA »FIZIKA TANÍTÁSA PROGRAMJA« 253

Page 40: Fizikai Szemle 68/7–8 – 2018. július–augusztusgoliat.eik.bme.hu/~f.simon/publications/Media/FizSzem-20180708.pdf · okokból standard modellnek hívunk, a világot anyagi és

átalakítására nincs szükség. A válaszadóknak lehetõ- 1. táblázat

A PhD hallgatók publikációs és konferencia-aktivitása1

angol nyelvû külföldi közlemények száma 81

külföldi szakmai rendezvények

részvételek száma 79

elõadások 55

poszter-elõadások 19

magyar nyelvû közlemények száma 192

hazai szakmai rendezvények

részvételek száma 173

elõadások, bemutatók 161

1 39 választ adó alapján.

sége volt tetszõleges megjegyzéseket, bõvítési javasla-tokat is megfogalmazni a kérdõíven. Így megfontolan-dó észrevételeket, javaslatokat kaptunk a kurzusokbelsõ tartalmának kiegészítésére, a hangsúlyok módo-sítására is. Ezeket a javaslatokat a következõkben akurzusvezetõkkel részletesen megbeszéljük, és be-építjük a képzésbe.

A tudományos kutatómunka gyakorlata a FizikaTanítása Doktori Programban

A munka során az elsõ lépés a kutatási célok és akonkrét munkaprogram meghatározása. Ebben a dok-torandusz a témavezetõjétõl kap segítséget. A megva-lósítás döntõ része egyéni feladat, aminek eredményeita munka lezárásakor egyes szám elsõ személybenmegfogalmazott „tézisek” tartalmazzák. A munka alap-vetõen egyéni jelegû, mégis akkor lehet igazán ered-ményes, ha azt sikerül beágyazni a szakmai közösség-be. Úgy látjuk, hogy ez utóbbi megvalósítása a gyakor-latban nem egyszerû. Fontosnak tartjuk tehát, hogyszoros munkakapcsolat alakuljon ki a doktorandusz éstémavezetõje között, illetve nyitott, segítõkész együtt-mûködés a doktoranduszok között. Ez utóbbi azért iskiemelten fontos, mert a szakmódszertani kutató-fej-lesztõ munka lényegi részéhez tartozik a feldolgozottúj témák, témakörök, illetve új módszerek iskolai ki-próbálása is, amiben a társak segítsége rendkívül fon-tos. Kérdõívünk rákérdez a témavezetõvel, illetve adoktorandusztársakkal kialakult munkakapcsolatokra.A válaszok azt mutatják, hogy a témavezetõkkel kiala-kított kapcsolat jó. A mentorok megfelelõ segítségetadnak a doktori munkához és az eredmények szaksze-rû publikációjához egyaránt. A doktorandusztársakkalkialakított munkakapcsolatok vonatkozásában a vála-szok azonban nem ilyen kedvezõk. Aktív, egymástsegítõ munkakapcsolatot a válaszadók körülbelül 40%-nak sikerült kialakítania kollégáival. A doktorandu-szok felének csak laza (formális) kapcsolata van, mígkörülbelül 10% sajnos úgy nyilatkozott, hogy semmi-lyen munkakapcsolata sincs.

A kutatási eredmények igazi értékmérõje a szakmainyilvánosság. A tapasztalatok szerint a gyakorló taná-rok anyanyelvi szóbeli kommunikációs készsége kivá-ló, a szakcikkek megírása azonban nehezebbenmegy. Fokozottan igaz ez a nemzetközi elõadások éscikkek vonatkozásában. A doktori képzés részekéntnagy hangsúlyt fektetünk arra, hogy doktorandu-szaink közvetlen szakmai tapasztalatokat szerezhesse-nek külföldön is [4]. Ezt segítendõ eddig 1 erdélyi, 3nyugat-európai és 1 kanadai egyetemmel alakítottunkki közvetlen együttmûködést, aminek keretében módnyílik rövid tanulmányutakra, közös kutatási munkák-ra, publikációkra is.

Kérdõívünk kiterjedt a külföldi tanulmányutak, anemzetközi és hazai konferenciákon történõ részvételés a publikációk felmérésére is. A 39 visszaküldöttválasz alapján az összesített adatokat az 1. táblázattartalmazza.

Itt érdemes megjegyezni, hogy az esetek többségé-ben az eredményes doktori munka a sikeres védésselnem záródik le. Sõt, a folytatáshoz általában mindenérdekelt részérõl megvan az igény. Jelenleg a folytatáslehetõsége korlátozott mértékben adott a DoktoriProgram bázisán szervezõdött MTA–ELTE Fizika Taní-tása Kutatócsoport keretében http://mta.hu/tantargy-pedagogiai-kutatasi-program/mta-elte-fizika-tanitasa-kutatocsoport-107105. A kutatócsoport 2020-ig szó-lóan „próbaidõs” jelleggel mûködik az MTA anyagitámogatásával. Nagyon fontosnak tartjuk, hogy a szak-módszertani kutatás ezen formája az MTA támogatásá-val hosszú távon és rendszerszerûen mûködjön. Meg-gyõzõdésünk, hogy az erre fordított támogatás orszá-gos szinten és komplex módon hasznosul: a sikeresszakmódszertani kutatás támogatásán kívül hatékonysegítséget jelent a doktori program jelenlegi mûködé-séhez (doktoranduszok részvételi költségeinek támo-gatása külföldi konferenciákon, szakmai rendezvénye-ken, nemzetközi kutatási együttmûködések kialakítá-sa, a doktori munkák eredményeinek gyakorlati kipró-bálását jelentõ oktatási kísérletek finanszírozása stb.).A Kutatócsoport létezése egyúttal hátteret adva stabili-zálja az egyetemi tanárképzés helyzetét is.

A fizikatanítás megújulásának fontos feltétele a ta-nárok szaktárgyi tájékozottságának szélesítése. A kér-dõív két kérdésében arra kértünk választ, hogy a részt-vevõk e téren miként értékelik a doktori programmunkáját:

„Változtatott-e a fizikaszemléletén a doktori prog-ram keretében végzett munka?” A 39 válaszadó közül11 fõ „lényeges” szemléletváltozást jelzett, 24-en a„részleges szemléletváltozás”-t, míg négyen szemlélet-változásuk mértékét az „alig” válasszal minõsítették. A„nem” feleletet senki nem jelölte meg.

„Változtatott-e a fizika tanításával kapcsolatos szem-léletén a doktori program keretében végzett munka?” Aválaszadók közül 9 fõ „lényeges”-nek, 22 „részleges”-nek minõsítette a változást, 5 fõ számára „alig” jelentettszemléletváltozást, míg 1 fõ szerint „nem” történt ilyenváltozás.

Megkérdeztük a résztvevõket, összességében mi-ként értékelik azt a terhelést, amit a négy féléven ke-

254 FIZIKAI SZEMLE 2018 / 7–8

Page 41: Fizikai Szemle 68/7–8 – 2018. július–augusztusgoliat.eik.bme.hu/~f.simon/publications/Media/FizSzem-20180708.pdf · okokból standard modellnek hívunk, a világot anyagi és

resztül tartó elõadások hallgatása, a vizsgák teljesíté-se, majd az önálló kutatómunka, az eredmények pub-likációja (és a disszertáció elkészítése) jelent. A vá-laszt négy lehetõség közül lehetett kijelölni. A 39 vá-laszadó közül 3 fõ „irreálisan nagy”-nak értékelte aterhelést (kicsit ellentmondva ennek, más kérdésekreadott válaszaikból kitûnik, hogy mindhárman sikere-sen teljesítették az elvárásokat és már védtek, vagyközvetlenül védés elõtt állnak.) A terhelés mértékérea többség (21 fõ) „nehéz, de elviselhetõ” minõsítéstadott, 14 kolléga szerint a terhelés „elviselhetõ”, egyválaszadó számára pedig „nem számottevõ”.

Arra a kérdésre, hogy ajánlaná-e kollégájának,esetleg szakmabeli hozzátartozójának a jelentkezéstaz ELTE fizikatanári doktori képzésére, valamennyiválaszadó pozitív értelemben felelt. Többen feltétel-ként fûzték hozzá, hogy csak jól képzett, a munkájairánt elkötelezett és igényes kolléga esetén mernékajánlani.

A felmérés – a konkrét kérdések megválaszolásántúl – kérte a résztvevõket, hogy írják meg minden ész-revételüket, javaslatukat, ami a képzés jobbítását szol-gálhatja. Számos ilyen javaslat érkezett, konkrét tartal-mi kérdésektõl, a vizsgák megszervezésén át, az egye-temi elektronikus adminisztráció nehézségéig. Lehetõ-ség szerint igyekszünk megoldani a felvetett problé-mákat, az elõadók bevonásával megfontoljuk a javas-latokat és, ha lehet, beépítjük a képzési rendbe.

A fentiek alapján úgy értékeljük, hogy a doktoriprogram – bár számos területen javítandó – alapve-tõen mûködõképes és eredményes. A hosszú távúeredményes mûködés azonban külsõ feltételektõl isfügg. Így a szaktanárok érdeklõdésétõl, ezen belül adoktori fokozat oktatáspolitikai elismerésétõl, defügg a szakmódszertan terület mûködésének egyete-mi helyzetétõl is. Az jelenti az alapvetõ problémát,hogy az utóbbi évtizedben nyugdíjba vonult kollé-gák helyét nem lehetett betölteni, és ezért a felada-tok folyamatos átadása-átvétele nem zavartalan.Mivel az ELTE helyzete a magyar fizikatanár-képzés-ben meghatározó, országos érdek, hogy az egyetembiztosítsa a szakmódszertani oktatógárda színvona-las utánpótlását és az ezen a területen végzett mun-ka elismerését.

A doktori program folyamatosságát az elõadói körelöregedése különösen is érinti. A mûködése az el-múlt 10 évben a jó célokért lelkesedõ kollégák ön-kéntes munkáján alapult, az itt végzett munka nemszámított az egyetemi óraterhelésbe és nem járt anya-gi elismeréssel sem. Induláskor a 16 elõadó között 2fõ volt nyugdíjas, ma 10 fõ. 2007-ben a tudományosmunkát irányító témavezetõk az Intézet fõállású okta-

tói voltak, ma felerészben nyugdíjasok [5]. Ilyen felté-telek mellett a Fizika Tanítása Doktori Program hosz-szú távú mûködése sajnos bizonytalan.

Összegzés

A tíz éve indított Fizika Tanítása Doktori Program szer-vezõiként a tapasztalatok és a megkérdezett résztve-võk véleményét összegezve jó érzéssel állapíthatjukmeg, hogy a befektetett munka oktatói és résztvevõioldalról sem tûnik hiábavalónak. Akár sikertörténet-nek is nevezhetnénk, hogy a hazai fizikatanár-képzés-ben egy komoly terheléssel járó, de értékteremtõ, mi-nõségi szintû tanári „elitképzés” szervezett alapjaitraktuk le. Természetesen a rendszer nem tökéletes éssaját tapasztalatainkat, valamint a résztvevõk javaslataitátgondolva igyekezni fogunk javítani azokon a tartalmiés szervezési problémákon, amelyek saját hatáskörbenváltoztathatók. Meggyõzõdésünk, hogy Magyarország-nak szüksége van magas szinten képzett fizikatanárok-ra. Ennek rendszerszintû biztosításához azonban nemelegendõ néhány egyetemi oktató ügy iránti elkötele-zettsége, személyes lelkesedése. Szükséges lenne,hogy a hallgatókat az oktatásirányítás hosszú távontámogassa (megpályázható tandíjtámogatás, ösztöndíj,külföldi tanulmányutak költségtámogatása, esetlegesiskolai órakedvezmény a tanulmányok idejére), deszükség lenne az egyetemi, kari, intézeti vezetés támo-gatására is, ami az ország legnagyobb egyeteménhosszú távon biztosítaná a szaktanárok tudományosszintû képzésének anyagi és személyi feltételeit. Mimagunk, az ügyért lelkesedõ kollégáinkkal együtt, ké-szen állunk a további munkára.

U.i.: A Fizika Tanítása program elõadásai mindenérdeklõdõ tanárkolléga számára nyilvánosak, a hely-szín, az elõadások címe és ideje a http://csodafizika.hu/fiztan/index.html honlapon megtalálhatók.

Irodalom1. Juhász A.: Az ELTE Fizika Doktori Iskolája „A Fizika tanítása”

címmel PhD-programot indít fizikatanárok részére. FizikaiSzemle 57/9–10 (2007) 333–340.

2. Tasnádi P., Juhász A.: Hagyományok és valóság. Szükség van-etudós tanárokra a természettudományban? Természet Világa 141(2010) 26–29.

3. Tasnádi P.: Motiváció, interpretáció, empátia: oktatási feladatok,tudományos kihívások a természettudományi szakmódszertan-ban. In: Tanulmányok a tudós tanárképzés mûhelyeibõl. (szerk.:Károly K., Perjés I.), ELTE Eötvös Kiadó, Budapest (2015) 77–91.

4. T. Tél, A. Juhász: Physics Education PhD Program at Eötvös Uni-versity, Budapest. Physics Competitions 12 (2010) 38–39.

5. Tél T.: Tanári doktori program fizika szakmódszertanból azELTE-n: célok és tapasztalatok. Új Pedagógiai Szemle (2015/9–10) 23–29.

Az Eötvös Társulat

fönt van a -on!

JUHÁSZ ANDRÁS, TÉL TAMÁS: TÍZ ÉVES AZ ELTE FIZIKA DOKTORI ISKOLA »FIZIKA TANÍTÁSA PROGRAMJA« 255

Page 42: Fizikai Szemle 68/7–8 – 2018. július–augusztusgoliat.eik.bme.hu/~f.simon/publications/Media/FizSzem-20180708.pdf · okokból standard modellnek hívunk, a világot anyagi és

A FIZIKA TANÍTÁSA

FIZIKA AZ ÉLÕ TERMÉSZETBEN ELTE TTK Anyagfizikai Tanszék

Az írás a szerzõ azonos címû könyve (ismertetve folyóiratunk ideifebruári számában) alapján készült.

Rajkovits Zsuzsanna PhD, ny. egyetemidocens. Az ELTE kémia-fizika szakán vég-zett, ahol az Anyagfizikai Tanszék fémfizi-kai kutatásaiban vett részt. Az oktatásku-tatásba a tehetséggondozás új módszerei-nek bevezetésével, új típusú tanulmányiversenyekkel kapcsolódott be. 1994-bennemzetközi versenyt alapított (ICYS),amelynek azóta is elnöke. Általános és kö-zépiskolásoknak írt újszerû fizika tanköny-vek társszerzõje, interdiszciplináris szem-léletû internetes gyûjtemény összeállítója.

Rajkovits Zsuzsanna

Az iskolai oktatásban – célszerûségi megfontolások-ból – a természet egészére vonatkozó ismereteketrészekre bontva, egymástól elválasztva szervezzüktantárgyi tartalmakká. Az élõ és élettelen természetbonyolultnak tûnõ világában a diákok a hosszú idõóta elfogadott, akadémiai módon felosztott tantárgy-szerkezetnek megfelelõen a fizika-, a kémia-, a bioló-gia- és a földrajzórán elsajátított ismeretek segítsé-gével próbálnak eligazodni. E folyamatot könnyebbétehetjük, ha felhívjuk a figyelmet e tárgyak kapcsoló-dási pontjaira. A továbbiakban a fizika és az élõvilág-ból vett jelenségek kapcsolatát keressük, de hasonlóvizsgálatot végezhetünk más tudományterületek kö-zött is.

Biológia a fizikaórán

Manapság, amikor a természettudományok népszerû-sége egyre csökken, mérlegelnünk kell azt is, hogytanításunk során a tantárgyakat diákjaink számáramég érdekesebbé, még vonzóbbá tegyük. A sok kí-nálkozó lehetõség közül egy módszer, ha például„észrevesszük” a fizikát a körülöttünk lévõ élõ termé-szetben. Minden korosztály számára érdekes lehet ésmindkét tantárgy szempontjából hasznos is, ha „meg-hívjuk” az élõvilágot a fizikaórára.

Tehetjük ezt például a fizika néhány törvényénektanítása során azzal, hogy a tanórai fizikai kísérletekmellett „demonstrációként” ismert biológiai jelensége-ket is bevezetünk. A segítségükkel megismert fizikaitörvények birtokában az élõvilág jelenségeit is mé-lyebben megérhetjük.

Megváltozott a világ. Manapság nem csak az iskolaoktat, nem csak a pedagógus az ismeretek fõ forrása.A diákot a médiából folyamatosan információáradatéri, amelynek egyes elemeit okosan felhasználhatjuka fizikaórán. A fizika törvényeire épülõ, a minden

tudományágban használatos egyre tökéletesedõ vizs-gálati technikákkal (nanotechnológia) az élõvilágeddig nem ismert finom részletei is feltárulnak. Az újeredményekrõl diákjaink számtalan, igen igényesenillusztrált népszerûsítõ könyvbõl, természetfilmbõl, dema már az internetrõl is, a felfedezést követõen szinteazonnal tudomást szerezhetnek. Napjainkban – a ter-mészettudományok egyes területei közötti kölcsönha-tásnak köszönhetõen – az újabb felfedezések eredmé-nyeit szinte a diákok szeme láttára kell beépíteni egy-egy tantárgy korábban „változtathatatlannak” hitt tan-anyagába. Ennek lehettünk tanúi a közelmúltban,például amikor a szén módosulatainak tanításakor afullerének bekerültek a tananyagba.

Nem csak egy-egy fizikai törvény tanításához „ve-hetünk” példát az élõvilágból. Egy-egy élõlény felépí-tésén, életterének bemutatásán keresztül rávilágítha-tunk azokra a fizikai jelenségekre is, amelyek az evo-lúció folyamatában hozzájárultak ahhoz, hogy az élõ-lények éppen az adott formában fejlõdtek ki. Így le-het a fizika hasznos „vendég” a biológiaórán is.

A biológiai példák a fiatalabb korosztály számáranéha csak figyelemfelkeltõ jelleggel használhatók, dekésõbb, a diákok fizikai ismeretanyagának bõvülésé-vel, a felsõbb osztályokban az élõvilág jelenségeinekmélyebb elemzésére is sort keríthetünk.

A mindennapi tanári gyakorlatban a hiányzó isme-retek gyûjtéséhez a nagy internetes tapasztalattal ren-delkezõ tanulókat feltétlenül érdemes bevonni. Az ígyelõálló ismeretanyagból tanári segítséggel választhat-juk ki a szakmailag helyes, és a számunkra megfelelõrészleteket. A diák érezheti, hogy kutatómunkájávalnem csak, mint egyik alanya, de a tanári munka segí-tõjeként aktív részese is lesz a tanítási folyamatnak. Eza részmunka, az „együtt-tanulás” egyben jó lehetõséga tanár-diák kapcsolat építésére, jobbítására.

Az alábbiakban a hidrosztatika témaköréhez kap-csolódóan szeretnék mintát mutatni a természettudo-mányok kapcsolódási pontjainak keresésére, s arra,miként használható e módszer a mindennapi tanításigyakorlatban, a tanórákon, szakköri foglalkozásokon.A témakör elõször az általános iskolában kerül elõ,számos, az alábbiakban ismertetett részlet a hetedikosztályos korosztálynak változatlanul tanítható.

Arkhimédész törvénye az élõvilágban

Vízben élnek a Föld legnagyobb testû élõlényei. Akacsa a földön totyogva nehézkes madár benyomásátkelti. Nem így a vízben! A vízfelszínen könnyedénsiklik (1. ábra ).

256 FIZIKAI SZEMLE 2018 / 7–8

Page 43: Fizikai Szemle 68/7–8 – 2018. július–augusztusgoliat.eik.bme.hu/~f.simon/publications/Media/FizSzem-20180708.pdf · okokból standard modellnek hívunk, a világot anyagi és

Mi okozza ezt a különbséget? A testek a vízben köny-

1. ábra. Szárazföldön suta, de a vizen könnyedén sikló tõkés récék.

2. ábra. Az úszóhólyag és helye a hal belsejében.

nyebbek, és mozgásukat a víz sem fékezi annyira, mint-ha a talajon súrlódnának. A kisebb súrlódás és a testek-re a vízben ható felhajtóerõ jelentõsen befolyásolta a ví-zi élõlények felépítését, viselkedését. Miért, hogyan?

A válaszhoz elõször elevenítsük fel az ide vonatko-zó fizikai ismereteinket.

A nyugvó folyadékok tulajdonságait a hidrosztatikatörvényeivel írhatjuk le. A nyugvó folyadékok felszíne aFöldön vízszintes, a nehézségi erõre merõleges. A folya-dékban egy adott mélységben a nyomás, a hidroszta-tikai nyomás, a Ł mélységgel egyenesen arányosan nõ,

p = ρgŁ,

ahol ρ a folyadék sûrûsége, g a nehézségi gyorsulásértéke azon a helyen. A levegõ nyomásával – Torricel-li kísérlete szerint – 76 cm magas higanyoszlop hid-rosztatikai nyomása tart egyensúlyt. A légnyomás ér-téke a fenti összefüggésbõl könnyen kiszámítható,

p = 105 Pa = 100 kPa.

Hasonló módon gyõzõdhetünk meg arról, hogy a víz-ben közelítõleg 10 méter mélyen ugyanekkora a nyo-más. Az ott tartózkodó testekre tehát a légköri nyo-más kétszerese, körülbelül 200 kPa nehezedik. Min-den újabb 10 méter a nyomást 100 kPa-lal növeli.

A hidrosztatikai nyomás következménye a testekreható felhajtóerõ, értéke a test folyadékba merülõ tér-fogatának megfelelõ térfogatú víz súlya:

F = Vtest ρv g.

Úszáskor a testekre ható erõk eredõje nulla, a nehéz-ségi erõt a felhajtóerõ kiegyenlíti. Számítással gyõzõd-jön meg mindenki maga arról, hogy igazak az alábbikijelentések:

1. a víznél kisebb sûrûségûtestek úsznak a víz felszínén,

2. ha a test sûrûsége meg-egyezik a víz sûrûségével, ak-kor a test bármely mélységbenegyensúlyban van, lebeg,

3. a víznél nagyobb sûrûsé-gû, tömör testek elmerülnek avízben,

4. a víznél nagyobb sûrûsé-gû anyagból készült üreges tes-tek úsznak a víz felszínén, haezek átlagos sûrûsége kisebb avíz sûrûségénél.

Víziállatok egyedülálló merülési „praktikái”

A tengerek, óceánok, tavak élõvilága igen gazdag,egy-egy vízi élõlény élettere sokszor a vízfelszíntõlkezdve több száz méter mély térségekre is kiterjed.Ahhoz, hogy az állat energiáját a leggazdaságosab-ban használja, a megélhetéséhez szükséges cseleke-detekre fordítsa, minden mélységben valamilyen„trükkel” biztosítania kell – szakirodalmi elnevezés-sel – a „semleges úszóképesség” állapotát, ami nemmás, mint a mindenkori mélységnek megfelelõ lebe-gés beállítása. A vízmélységgel nõ a nyomás,csökken a hõmérséklet, s e körülményekhez mindenvíziállatnak alkalmazkodnia kellett. Minden tartózko-dási helyükön az állatra ható nehézségi erõnekegyensúlyt kell tartania a felhajtóerõvel, s ehhez aszabályozáshoz bizonyos fortélyokra van szükség. Avíziállatok merülése és felszínre emelkedése gyorsfolyamat, de a kívánt mélységekben való huzamo-sabb tartózkodáshoz szükségesek azok a fortélyok,amelyek többnyire az állat átlagsûrûségének változ-tatásával valósulnak meg.

Vajon a vízben élõ állatok milyen fogásokat alkal-maznak átlagsûrûségük változtatására?

A csontos halak

A vízben élõ állatok átlagos sûrûsége a vízével közelazonos, ezért a rájuk ható felhajtóerõ a nehézségi erõtteljesen kiegyenlíti, lebegnek. Ez a körülmény felépí-tésükben úgy tükrözõdik, hogy a csontvázuk tömegea teljes testtömegnek csupán néhány százalékát tesziki, nincsen szükség olyan masszív tartó szerkezetre,mint amilyen a szárazföldön élõ társaiknál tapasztal-

ható. Míg a halak csontváza ateljes testtömegük körülbelül8%-a, a vízi és szárazföldi éle-tet is élõ békáké körülbelül11%, addig a nyúlnál ez az ér-ték már a 15%-ot is eléri.

A csontos halak érdekesszerve az úszás mélységét fi-noman szabályzó úszóhólyag(2. ábra ).

A FIZIKA TANÍTÁSA 257

Page 44: Fizikai Szemle 68/7–8 – 2018. július–augusztusgoliat.eik.bme.hu/~f.simon/publications/Media/FizSzem-20180708.pdf · okokból standard modellnek hívunk, a világot anyagi és

Az úszóhólyag fala figyelemre méltó, rugalmas

3. ábra. Porcos halak képviselõje, a cápa.

4. ábra. A fejlábúakhoz tartozó közönséges tintahal és a szépiacsont.

tulajdonságokkal rendelkezik. A hal az úszóhólyagjá-val „állítja be” a mindenkori mélységnek megfelelõátlagos sûrûségét. A zárt úszóhólyagot gázzal töltimeg, ha a térfogat növelésére van szükség, ha viszontmélyebbre kerül, az ott uralkodó nagyobb nyomásmiatt az úszóhólyag összenyomódik, az állat átlagsû-rûsége nõ. A hólyag méretével változik a hal térfoga-ta, emiatt átlagsûrûsége is.

Akár a gáztörvények tanításánál is használhatjuk ahalak merülési szokásait. Hangozzék a feladat a kö-vetkezõképpen: körülbelül 50 méter mélységben ahal úszóhólyagjának térfogata a teljes testtérfogat 5százaléka legyen. Hányszorosára változik az úszóhó-lyag térfogata 300 méter mélységben? (A víz hõmér-séklet-változásától számításainkban tekintsünk el!)

Az állandó hõmérséklet feltételezése miatt a Boyle–Mariotte-törvény felhasználásával adhatjuk meg aválaszt. A nyomás 50 méter mélyen 6 atmoszféra, 300méter mélyen pedig 31 atmoszféra.

Az úszóhólyag térfogata 300 méter mélységben ötö-

V0 6 atm = V 31 atm,

V0

V≈ 5.

dére csökken. Ezzel a hal átlagsûrûsége az 50 méteresmélységbeli átlagsûrûséghez viszonyítva megnõ, kü-lönös erõfeszítés nélkül folyamatosan tartózkodhat300 méter mélyen [1]. A gáztörvények tanításakorilyen jellegû egyszerû feladatokkal változatosabbátehetjük a tanítást, mert egyúttal feleleveníthetünkmás témákhoz, illetve más tudományterületekhezkapcsolódó ismereteket is.

A cápák

A cápák (3. ábra ) porcos halak, nincsen úszóhólyag-juk. Más merülési praktikához kell folyamodniuk.Vázuk már azzal könnyebbé válik, hogy csont helyettporcból áll. De átlagsûrûségük változtatására máslehetõségük is van.

A cápamáj a mindenkori átlagos sûrûség beállításá-hoz használt szervük, amely az állat teljes testtömegé-nek akár 25-30 százaléka is lehet (emlõsöknél csak 5százalék). A cápamáj körülbe-lül 70 százalékban a tengervíz-nél kisebb sûrûségû olajokatés egyéb szerves anyagokat(squalene) tartalmaz. A cápa atartózkodási helyének meg-felelõ átlagsûrûségét a májátalkotó anyagok térfogati há-nyadának változtatásával ál-lítja be. A tengervíz sûrûségeρt = 1,026 g/cm3, az olajok sû-rûsége ρo = 0,90–0,92 g/cm3, asqualene sûrûsége pedig ρs =0,855 g/cm3. Mivel a cápákmája, mint sûrûségszabályozó

nem annyira hatékony szerv, mint az úszóhólyag acsontos halaknál, a cápák állandóan mozgásban van-nak, hogy elkerüljék az elmerülést.

A tintahal

A tintahal vagy szépia nevével ellentétben nem hal,hanem puhatestû, lábasfejû (más néven fejlábú), apolipok rokona. Merüléskor a szabályozást a szépia-csonttal oldja meg, amely a teljes testtérfogat közel 10százalékát (9,3%) teszi ki (4. ábra ).

A szépiacsont celluláris anyag, lemezes szerkezetû,a CaCO3 aragonit kristályaiból épül fel. Az aragonitle-mezek szervezõdését ilyenné egy szerves molekula(kitin) irányítja. Az állatból frissen kivett csont átlag-sûrûsége a ρ = 0,57–0,64 g/cm3 tartományban mozog.Elég könnyû, az állat átlagsûrûségét csökkenti, éskiváló mechanikai tulajdonságai is vannak, így jólellenáll a mélységváltozás okozta nagy nyomásválto-zásnak is. Védi az állat létfontosságú szerveit, példáulkiugró fejlettségû idegrendszerét. A szépiacsont másikfontos feladata a semleges úszóképesség beállítása,amelyet a tintahal a csont víztartalmának változtatásá-val valósít meg. A víztartalom változtatása – más ten-geri állatokhoz hasonlóan, – az úgynevezett ozmore-gulációs folyamattal történik [1].

Az ozmózis részletes ismertetése a középiskolás tan-tervekbe sem mindig kerül be, pedig az élõvilág gyak-ran elõforduló jelensége. Itt csupán egy kísérlettel mu-tatjuk be a jelenséget, a kvantitatív elemzést elkerüljük.

258 FIZIKAI SZEMLE 2018 / 7–8

Page 45: Fizikai Szemle 68/7–8 – 2018. július–augusztusgoliat.eik.bme.hu/~f.simon/publications/Media/FizSzem-20180708.pdf · okokból standard modellnek hívunk, a világot anyagi és

Az ozmózis jelensége

5. ábra. Kísérlet az ozmózisnyomás demonstrálására.

cukoroldat

celofán

víz

6. ábra. A csigaházas polip és házának hosszanti metszete.

Ozmózisnak nevezzük az olyan „egyirányú” diffú-ziót, amely oldat és oldószer között jelentkezik, haezeket féligáteresztõ (más néven szemipermeábilis)fal (hártya) választja el. Féligáteresztõ az olyan fal,amely az oldószer molekuláit átengedi, de az oldottanyag molekuláit nem. Az ozmózis az oldat hígulá-sával jár együtt. Ha az oldat zárt edényben hígul,akkor nyomása megnõ. A nyomás sok esetben tekin-télyes lehet.

Egyszerû kísérlettel magunk is elõállíthatjuk a je-lenséget.

Ragasszunk celofánhártyát egy körülbelül 3-4 cmátmérõjû, hosszúnyakú üvegtölcsérre! A ragasztástúgynevezett akváriumragasztóval könnyen elvégez-hetjük. A tölcsérbe töltsünk cukoroldatot, és a tölcsérszárán jelöljük meg az oldat szintjét. A tölcsért áll-ványba fogva, a 5. ábrán látható módon helyezzükvízzel telt fõzõpohárba úgy, hogy az oldat és a vízszintje egy magasságban legyen.

A víz diffúziója a féligáteresztõ hártyán keresztülrövidesen mindkét irányban megindul, de az oldatfelõli diffúzió kisebb mértékû. A folyamat végered-ményeként az oldat hígul, térfogata nõ, a tölcsér szá-rában emelkedik a folyadékszint. A szintkülönbség-nek megfelelõ nyomás addig nõ, amíg a nyomás avíz további oldatba hatolását már megakadályozza.Ezt a nyomást az oldat ozmózisnyomásának nevez-zük. Az oldatok ozmózisnyomása meglepõen nagy,

például az 1%-os cukoroldat ozmózisnyomása a lég-köri nyomás körülbelül 2/3 része, amely majdnem 7méter magas vízoszlop nyomásának felel meg. Acelofánhártya ekkora nyomást nem bír ki, a demonst-ráció már néhány deciméteres emelkedés után kény-szerû véget ér. A nagy ozmózisnyomás teszi lehetõvépéldául azt, hogy a talaj nedvessége a fák koronájábais eljusson.

A szépiacsonthoz visszatérve, ezután már megért-hetjük a tintahal átlagsûrûség-beállító folyamatát. Aszabályozás nagy körvonalakban abban áll, hogy azállat a szépiacsontban lévõ folyadék-gáz arányt képesváltoztatni. Nagyobb gázhányad könnyebb csont,kisebb arány mellett viszont nehezebb csont áll elõ, sígy változik az állat átlagsûrûsége is. Az ozmózisnyo-másnak a csontbeli folyadék mozgatásában van szere-pe. Kutatók kimutatták, hogy a szépiacsontnak az ál-lat lágy részeihez csatlakozó felületén található leme-zei között mindig található folyadék, amely a tenger-vízhez hasonlóan fõleg oldott NaCl-ot tartalmaz. Ha acsonttal érintkezõ testfolyadékban nagyobb az ion-koncentráció, akkor olyan folyamat indul meg a félig-áteresztõ szövet- és sejthártyákon keresztül, amelyhígítja a nagyobb koncentrációjú tartományt, azaz vízkerül a csontból a testbe. A folyamat teljes részletezé-sét e helyen mellõzzük, itt csak az ozmózis szerepérekívántuk felhívni a figyelmet [2].

A csigaházas polip

A Nautilus pompilius vagy csigaházas polip a lábas-fejûek közé tartozik (6. ábra ). A mintegy 20 cm-remegnövõ, gyöngyházas Nautilus ma is élõ faja azonpuhatestûeknek, amelyek már 550 millió évvel ez-elõtt megjelentek. „A csigáspolip élõ fosszília” – szok-ták mondani. A polipok közül az egyetlen, amelyházat növeszt.

A csigáspolip a házát szigorú rend szerint építi. Alapos, szinte egy síkban készült építmény logaritmi-kus spirál 1 (7. ábra ) módjára tekeredik. Házának

1 A görbe úgy képzõdik, hogy akiinduló O pontból a görbe Mi pont-jába húzott egyenes és az Mi ponthoztartozó érintõ által bezárt α (< 90°)szög állandó, ekkor az OMi = ri távol-ság logaritmikusan nõ.

belsejében kamrák vannak, az állat a legkülsõ kam-rában lakik.

Házát szinte szakadatlanul építi, nagyobbítja, a már„kinõtt” helyiséget lefalazza. A lezárt ház térfogataarányos megnövekedett testtömegével, így biztosítja

úszóképességét. Az állat aspirál kezdõpontjánál lévõkamrához csatlakozó szifójánkeresztül áll összeköttetésbena kamráival (8. ábra ) [3]. Akamrák gázt tartalmaznak,csak a néhány frissen „lefala-zott”, legkülsõ kamrában vanegy kevés tengervíz.

A FIZIKA TANÍTÁSA 259

Page 46: Fizikai Szemle 68/7–8 – 2018. július–augusztusgoliat.eik.bme.hu/~f.simon/publications/Media/FizSzem-20180708.pdf · okokból standard modellnek hívunk, a világot anyagi és

Azt is szokták mondani, hogy a Nautilus „élõ ten-

7. ábra. A logaritmikus spirál görbéje.

O

M1

r1

M2

r2

a

a

8. ábra. A szifó spirálisa.

9. ábra. A „vízsugárhajtású” kalmár sematikus rajza.

szelepek

szelepek

tölcsér

szelepek

tölcsér

geralattjáró”. A Nautilus 50 és 500 méteres mélység-ben is elõfordul. Ekkora mélységbeli különbségekigen nagy nyomás- és hõmérséklet-változásokkal jár-nak. Az állatnak tehát alkalmazkodnia kellett e szélsõ-séges körülményekhez. A csigáspolip nappal általá-ban a tengerfenéken él, csak éjszaka merészkedikmagasabbra. Ilyenkor néhány tíz méter mélységben, amelegebb vízrétegben tartózkodik. Tojásait – amelyekbõrrel borított kapszulákhoz hasonlítanak – is iderakja, általában kemény felületre, sziklákra vagy ko-rallokra. Szaporodását még most is titok övezi. Élõembriót elõször – akváriumi körülmények között –csak 1985-ben figyeltek meg [4].

A csigaházas polip nagyon rövid idõ alatt több százméter mélyre tud süllyedni, s onnan felemelkedni.

A mélytengeri expedícióknak köszönhetõen sokfilm készült a csigaházas polip életérõl. Jellegzetes„himbálódzó” mozgásáról az állat messzirõl megis-merhetõ. E mozgás a lebegés jelenségével kapcsola-tos, amely állapotot az állat minden mélységben sajá-tos módszerrel maga állít be. Mivel a ház és a puha-testû állat együttes térfogata – és emiatt a rá ható fel-hajtóerõ is – a mélységtõl függetlenül mindig ugyan-akkora, az egyes eltérõ mélységekben tömegét kellszabályoznia.

A frissen lezárt kamrák tengervizet is tartalmaz-nak. Egy adott mélységben a lebegését a csigáspolipis úgynevezett ozmoregulációval biztosítja. A víz akamrából ozmotikus úton – a szifó szövetein, sejt-jein keresztül –, a testnedvek ozmózisnyomásátmegfelelõen beállítva tûnik el. Ezzel csökken az állatátlagsûrûsége, így a csigáspolipra ható nehézségierõ is, és az állat a felhajtóerõnek köszönhetõenmagasabb vízrétegekben lebeghet. Ha mélyebbrekerül, akkor néhány hátsó kamrájába ismét – a szifó-szövetei, sejtjei membránjain keresztül – folyadékotjuttat, megnövelve ezzel az átlagsûrûségét, így álla-pota a mélyebb vizekben, a nagyobb nyomású he-lyeken stabilizálódik.

Házában túlnyomásnak kell lennie, amely néhányszáz méter mélyen már jelentõs, ellenkezõ esetbengyöngyházból készült háza a nagy külsõ nyomásmiatt összeroppanna. A csigáspolip tökéletesebb,

mint a hasonló elven mûködõ tengeralattjáró, mert atengeralattjáró a víz kiszorításához használt levegõttöbbletsúlyként viszi magával.

Fejlábúak helyváltoztatása

A Nautilus, puhatestû társaihoz hasonlóan, vízsugár-hajtással halad. A rakétameghajtást Ciolkovszkij a 19.század végén írta le. Mint ismeretes, a rakétákat a ma-gukkal vitt üzemanyag elégetésekor keletkezõ gázokkiáramlásánál fellépõ reakcióerõ hajtja elõre. A poli-pok, kalmárok, tintahalak, amelyek igen aktív vadá-szok, helyváltoztatáshoz a vízsugárhajtást már sokkalrégebben használják. Meneküléskor (és vadászatkor) atestük nagy hányadát elfoglaló köpenyüregbe vizetszivattyúznak, s a tölcsérré alakult szervükön keresztülazt kilövellik, amitõl – az impulzusmegmaradás törvé-nyének megfelelõen – õk az ellenkezõ irányba moz-dulnak el. A veszélyt érzékelõ állat hirtelen megnövelia köpenyüreg térfogatát, benne a nyomás lecsökken, saz elõálló nyomáskülönbségnek köszönhetõen a test-felszín megfelelõ helyén található szelepeken keresztüla tengerbõl víz áramlik a köpeny belsejébe. Az elvet akalmár sematikus ábráján mutatjuk be (9. ábra ), mert

260 FIZIKAI SZEMLE 2018 / 7–8

Page 47: Fizikai Szemle 68/7–8 – 2018. július–augusztusgoliat.eik.bme.hu/~f.simon/publications/Media/FizSzem-20180708.pdf · okokból standard modellnek hívunk, a világot anyagi és

ennél az állatnál szembetûnõbbek a részletek. Az állat

10. ábra. A hengeralakkal közelítõleg leírható kékbálna.

11. ábra. A mintegy 500 lóerõs ámbráscet és borja.

izma segítségével még be is görbítheti a tölcsért, ígymozgása során irányváltoztatásra is képes.

Tengeri emlõsök

A vízben lévõ testekre a Föld vonzásán kívül a felhaj-tóerõ is hat, a testek súlya éppen ezzel az értékkelkisebb, mint a levegõben. Ez a magyarázata annak,hogy vízben élnek, s könnyen mozognak a világ leg-nagyobb tömegû élõlényei.

Míg az elefánt, 3-5 tonna tömegével a legnagyobbtömegû szárazföldi emlõs, addig a kékbálna (10. áb-ra ) tömege a 130 tonnát is eléri.

A bálna méreteinek ismeretében az Arkhimédésztörvénye segítségével már általános iskolás tanulókkalis megbecsültethetjük a bálna tömegét. Jacques-YvesCousteau, a közismert Cousteau kapitány tengerkuta-tó mélytengeri megfigyeléseibõl tudjuk, hogy egyeskékbálnapéldányok hossza a 30 métert is meghaladja,átmérõjük ennek körülbelül a tizedrésze. Durva becs-léshez elég, ha az állatot hengeralakúnak feltételez-zük. A bálna által kiszorított víz súlya

Gvíz = Vbálna ρvíz g.

amelybõl a térfogatra a

Vbálna = d2 π4

Ł = Ł2

102

π4

Ł = Ł3

400π

összefüggést kapjuk. A Ł = 30 méteres hosszat fel-használva a térfogatra 212 m3 adódik.

A kiszorított víz súlya, a tengervíz sûrûségét ρvíz =1030 gk/m3-nek tekintve

A bálna tömege néhányszor 105 kg, azaz néhány száz

Gvíz = 212 m3 1030 kg

m310 m

s2= 2,18 106 N.

tonna. (A valóságos tömegnél természetesen nagyobbadódott, mert az egyszerûbb számítás miatti hengeresbálnaalak durva, felsõ közelítés.)

Ezzel a hatalmas tömeggel akár 30 km/óra sebes-séggel is képes úszni. (A motorcsónak sebessége kö-rülbelül ekkora.) A 60 tonna tömegû ámbráscet (11.ábra ) néhány méter magasra még ki is ugrik a vízbõl.De a vízi életmódhoz alkalmazkodott csontvázuk aszárazföldön képtelen lenne ekkora terhet megtartani.Érthetõ, ha ezek az állatok – valamilyen oknál fogva –partra vetõdve képtelenek levegõt venni, ekkor tüde-jük a hatalmas tömegû izom és zsírpárna súlya alattszinte teljesen összenyomódik, elpusztulnak.

Bálnák merülése

Természetfilmekbõl ismeretes, hogy ezek az emlõsál-latok vízszintes, lapátszerû farokúszójukkal – amely-nek teljesítménye elérheti az 500 lóerõt, ez egy traktorteljesítményének közel kétszeresével egyenlõ, 1 lóerõ= 736 watt – csapnak a vízre.

A bálna a reakcióerõnek – a víz által az állat 4 mé-teres vízszintes kiterjedésû farokúszójára ható erõnek– köszönhetõen bukik a mélybe, ahol a légköri nyo-más 5-6-szorosa várja. E nyomás hatására tüdeje alehetõ legkisebb térfogatúra nyomódik össze, emiatt abálna teljes testének térfogata csökken. A kisebb tér-fogat már kevesebb vizet szorít ki, a felhajtóerõ le-csökken, tartósan a mélyben maradhat. A felszínrejutva levegõt vesz, tüdeje kitágul, a felhajtóerõ meg-nõ. A változatlan nehézségi erõ, de a megnövekedettfelhajtóerõ miatt nem is szükséges teljesen elmerül-nie, gyakran látható, hogy a test egy része a vízbõlkiemelkedve úszik. Azt is mondhatjuk, hogy a bálnaváltozatlan tömege mellett a ρ = m /V átlagsûrûségéttérfogata változtatásával befolyásolja.

Nem így az ámbráscet!

Hordó alakú feje a testhossznegyedét teszi ki, és a teljestesttömeg harmad része. Fejé-ben és törzsében a gerincosz-lop mentén különleges lágy,hõre rendkívül érzékenyanyaggal, cetvelõvel töltöttkamrák vannak. A fejébentalálható szerve, amely a cet-olaj vagy bálnaolaj névenismert anyagot tartalmazza,felelõs a semleges úszóképes-

A FIZIKA TANÍTÁSA 261

Page 48: Fizikai Szemle 68/7–8 – 2018. július–augusztusgoliat.eik.bme.hu/~f.simon/publications/Media/FizSzem-20180708.pdf · okokból standard modellnek hívunk, a világot anyagi és

ség biztosításáért (12. ábra ). A körülbelül 2000 liter

12. ábra. Az ámbráscet cetvelõt tartalmazó szerve.

cetvelõs szerv bal orrjáratbal orrjárat

jobb orrjáratjobb orrjárat

cetvelõ

kifújó orrnyílás

13. ábra. A víznél sokkal kisebb átlagsûrûségû szárcsa.

cetolaj – amely viasz-észterek és trigliceridek keveréke– egy „zsákban” helyezkedik el, és rendkívül érdekestermodinamikai tulajdonságokkal rendelkezik. A cet-olaj körülbelül 29 °C hõmérsékleten folyadék-szilárdhalmazállapot-változáson megy keresztül. 30 °C körülviszonylag kis viszkozitású folyadék, amelynek atmosz-férikus nyomáson vett sûrûsége 862 kg/m3, 29 °C alatttérfogatcsökkenéssel is járó szilárd viasszá kristályoso-dik, ekkor sûrûsége 22,5 °C-on 889 kg/m3.

Merüléskor a tengervíz hõmérséklete a mélységgelcsökken, eközben a cetvelõ lágy állapotból nagyobb sû-rûségû szilárd állapotúvá válik. A fentiekbõl látható,hogy már nagyon kicsi, csupán 1-2 °C hõmérséklet-vál-tozás jelentõs sûrûségváltozással jár. Mivel a szerv tö-mege nagy, e módszerrel hatékony tömegnövekedés ér-hetõ el. A cetvelõ hõmérséklet-szabályozásában a szer-ven áthaladó érhálózat melegítõ hatása és a szervet át-szelõ orrjáratokon átáramló tengervíz hûtõ hatása játszikszerepet [5]. Az állat tömege tehát így változtatható. Azállat térfogata bármely mélységben ugyanakkora, az át-lagsûrûség-változtatás ebben az esetben a tömeg csök-kentésén, illetve növelésén keresztül történik.

E helyen csak megjegyezzük, hogy ezzel a szervvelvalósul meg a bálnák és delfinek ultrahangokkal tör-ténõ tájékozódása, az echolokáció is. A szerv ekkor„viaszlencse” módjára viselkedik, amely az állat fejé-ben a megfelelõ helyre fókuszálja a bálnák által kibo-csátott, a tengervízben a levegõhöz viszonyítva ötszörnagyobb sebességgel terjedõ hanghullámokat. Ebbõlis láthatjuk, hogy ez az állat is számos vonatkozásbankapcsolódik a fizikához, érdemes jól megismerni min-den tulajdonságát, majd a más tudományterületekenszerzett ismereteink segítségével megpróbálni megér-teni a viselkedését.

Miért nem merülnek el a vízi madarak?

A vízi madarak (13. ábra ) tollukat zsírozzák, a vízzelnem nedvesedõ tollak és a pehelytollak között jelen-tõs mennyiségû levegõt tudnak tárolni. A sok megkö-tött levegõtõl a madarak átlagsûrûsége csökken, tes-tüket sajátos „buborék” veszi körül, ezért úszva csaktérfogatuk kis hányada merül a vízbe.

Ha tolluk víztaszító tulajdonsága megváltozik, pél-dául környezetszennyezés miatt, akkor a fogva tartottlevegõ mennyisége csökken, elmerülnek. Ha a tavak-

ba olyan szennyezõ anyag (például mosószer) jut,amely csökkentheti a víz felületi feszültségét, akkor atoll jobban nedvesedik, az állat akár vízbe is fulladhat.Látható, hogy a környezetszennyezés az állatot kétoldalról is veszélyezteti.

Az úszás munkával jár

Elgondolkodhatunk azon, hogy mekkora teljesítményszükséges ahhoz, hogy a víznél nagyobb sûrûségûélõlény mozgása során a vízben mégse merüljön el.

Végezzünk számításokat! A V térfogatának f há-nyadával vízbe merülõ ρ sûrûségû állatra ható Ff fel-hajtóerõ:

Ff = fVρv g,

ahol ρv a víz sûrûsége. Az állatnak munkavégzéssel anehézségi erõ és a felhajtó erõ

eredõjének megfelelõ nagyságú, de azzal ellentétes irá-

F = g V ρ − f ρ v

nyú erõt kell kialakítania ahhoz, hogy el ne merüljön.Ez az erõ elõállítható, ha az állat valamely testrészé-

vel lefelé mutató irányban a vízre csap. Ekkor ugyanis avíz lefelé gyorsuló mozgást végez, a víz ellenereje hataz állatra, amely a vízfelszínen tartja õt.

Ha a mozgó testrész területe A, a csapással gyorsí-tott víz végsebessége v, akkor a mozgás során idõegy-ség alatt gyorsított víztömeg

m = Avρv.

A víz mv impulzusa pedig éppen az úszó test lendü-letigénye is. Az impulzus idõegység alatt történõ vál-tozása éppen a meglökött víz állatra ható – a testétfenntartó – reakcióereje. Egyenletekkel a következõformában adható meg:

ahonnan

g V ρ − f ρ v = A v 2 ρ v ,

v 2 =g V ρ − f ρ v

A ρ v

.

262 FIZIKAI SZEMLE 2018 / 7–8

alsó állkapocs

Page 49: Fizikai Szemle 68/7–8 – 2018. július–augusztusgoliat.eik.bme.hu/~f.simon/publications/Media/FizSzem-20180708.pdf · okokból standard modellnek hívunk, a világot anyagi és

A mozgásban lévõ testrészek által végzett munka avíz kinetikus energiájává alakul. Így a testrészek tel-jesítménye éppen a víz egységnyi idõ alatt történõmozgásienergia-változásával lesz egyenlõ, amely

képlettel fejezhetõ ki [6].

P = 12

A ρ v v v 2 = 12

⎡⎢⎣

⎤⎥⎦

ρ g V⎛⎜⎝

⎞⎟⎠

1 −f ρ v

ρ

3

A ρ v

A fenti formula felhasználásával számos érdekesjelenségre kvantitatív magyarázatot adhatunk. Pél-dául: kiszámíthatjuk, hogy az 50 kg tömegû úszónõ-nek ahhoz, hogy az orrát a vízfelszín felett tartsa,mozgó végtagjaival 7,8 watt teljesítményt kell kifejteni.

(A számításhoz használt további adatok: a test 95 szá-zaléka merül vízbe, a mozgó végtagok teljes területe600 cm2, s legyen ρúszó = ρv. )

Számítással magunk is ellenõrizzük a teljesítményfenti értékét!

Irodalom1. Paul Davidovits: Physics in Biology and Medicine. 3rd edition,

Academic Press, Elsevier Inc. (2008) 93–95.2. Kundt Schmidt-Nielsen: Animal Physiology. 4th Edition, Cam-

bridge University Press, Cambridge (1990) 433–448.3. Greguss Ferenc: Eleven találmányok. Móra Könyvkiadó, Buda-

pest (1976)4. http://en.wikipedia.org/wiki/Waikiki_Aquarium5. M. R. Clarke: Physical properties of spermaceti oil in sperm

whales. J. Mar. Biol. Ass. U.K. 58 (1978) 19–26.6. Paul Davidovits: Physics in Biology and Medicine. 3rd edition,

Academic Press, Elsevier Inc. (2008) 82–87.

RELATIVITÁSELMÉLETRÕL KÖZÉPISKOLÁBAN – MÁSKÉNT,KIEGÉSZÍTÉS ELTE Biológiai Fizika Tanszék

Kürti Jenõ (1954) fizikus, az MTA doktora,az ELTE Biológiai Fizika Tanszékének pro-fesszora. Kutatási területe elsõsorban szénnanoszerkezetek elméleti vizsgálata, külö-nös tekintettel azok rezgési spektroszkó-piájára. A Fizika III egyetemi tankönyvRelativitáselmélet, valamint Atomhéjfizikarészeinek szerzõje.

Kürti Jeno

A Fizikai Szemle 2018. áprilisi számának A fizika ta-nítása rovatában jelent meg Kiss Miklós remek írása,hogy miként lehet az érdeklõdõ középiskolások fi-gyelmét fölhívni arra, hogy az elektromos és a mágne-ses mezõk mindig együtt kezelendõk. Külön-külön azelektromos mezõnek és a mágneses mezõnek nemcsak a nagysága, hanem még a léte vagy nem léte isfügg attól, hogy milyen vonatkoztatási rendszerbõlnézzük. Ez valóban elgondolkodtató lehet az érdeklõ-dõ diákoknak.

A konkrét példa két párhuzamos, nagyon („végte-lenül”) hosszú, egyenletesen töltött szigetelõ egyenesközötti erõhatás vizsgálata volt. A levezetés soránazonban egy kis „kegyes csalás” történt. Az az állításugyanis, hogy „A vonatkoztatási rendszertõl nemfügghet az erõ nagysága” nem igaz, ha – amint a cikk-ben is történt – a klasszikus, úgynevezett hármas erõ-rõl beszélünk. Még akkor sem, ha itt a kölcsönösmozgás irányára merõleges komponensrõl van szó!

Szerencsére még egy pongyolaság történt, ugyanisa szövegben nem egyetlen ponttöltésre ható erõrõlvan szó, hanem az egységnyi hosszúságú darabra ha-tó erõrõl. Így a levezetés végül mégis helyes, köszön-

hetõen annak, hogy két ellentétes tényezõ kompen-zálja egymást. Mivel ez azonnal nem látható, ezértszeretném ezt kicsit részletesebben kifejteni ebben azírásban.

Válasszunk egy még egyszerûbb esetet, mint ame-lyet Kiss Miklós írása tárgyal: az egyik töltött egyeneshelyett legyen egy q ponttöltésünk. (A két ponttöltésesete fizikailag még egyszerûbb, azonban ott matema-tikai komplikációk merülnek föl: a töltés-, illetveáramsûrûségeknél az úgynevezett Dirac-deltával kel-lene dolgoznunk.) A másik töltött egyenes pedig le-gyen egy vékony, tömör egyenes henger (ennek elõ-nye késõbb világos kell legyen). A mezõket a henge-ren kívül vizsgáljuk.

A feladat tehát a következõ: egy végtelen hosszú,egyenes, A keresztmetszetû, henger alakú szigetelõtegyenletesen feltöltünk ρ (térfogati!) töltéssûrûséggel.A henger tengelyétõl d távolságra egy, a hengerhezképest nyugalomban lévõ pontszerû q töltést helye-zünk el. Vizsgáljuk a ponttöltésre ható erõt két vonat-koztatási rendszerbõl:

a) laboratóriumi rendszerbõl, amelyben mindentöltés nyugalomban van és

b) a henger tengelyével párhuzamosan állandó vsebességgel mozgó inerciarendszerbõl.

A számításokhoz – a jobb áttekinthetõség kedvéért– relativisztikus mértékegységrendszert, vagyis olyathasználjunk, amelyben a vákuumbeli fénysebesség

sõt külön az ε0-t (és ezzel persze automatikusan a

c ≡ 1

ε0 μ0

= 1,

μ0-t) is válasszuk 1-nek! Hangsúlyozzuk, hogy emiatt

A FIZIKA TANÍTÁSA 263

Page 50: Fizikai Szemle 68/7–8 – 2018. július–augusztusgoliat.eik.bme.hu/~f.simon/publications/Media/FizSzem-20180708.pdf · okokból standard modellnek hívunk, a világot anyagi és

ebben az írásban a v nem a hagyományos, m/s-banmért sebességet jelenti, hanem a fénysebességegysé-gekben mért dimenziótlan sebességet! A kapott kép-letek – kis odafigyeléssel – bármikor átírhatók az SI-mértékegységekkel kifejezett alakjukra.

Annak érdekében, hogy a probléma mélységeihezjobban hozzáférjünk, használjuk a relativisztikus, úgy-nevezett négyes formalizmust, persze „lájtos” módon.Az alább leírtak jobb megértéséhez segítséget nyújt-hat például a Fizika III tankönyv középiskolások szá-mára is megemészthetõ Relativitáselmélet fejezete [1].

Foglaljuk össze nagyon röviden a téridõbeli né-gyesvektorok legfontosabb tulajdonságait!

Minden négyesvektor egy idõ- és három térkom-ponensbõl áll. Legyen például a μ egy téridõbeli né-gyesvektor (μ = 0,1, 2, 3), amelynek idõkomponensea 0 ≡ at , a három térbeli komponense pedig (ax,ay,az ),röviden összefoglalva a. Ekkor írhatjuk:

A négyesvektorok mintapéldánya a téridõ-helyvektor

a μ ≡ (a 0, a 1, a 2, a 3) ≡ (at , ax , ay , az ) ≡ (at , a).

(idõ-hely négyesvektor):

Másik nevezetes példa a négyesimpulzus (energia-

x μ ≡ (x 0, x 1, x 2, x 3) ≡ (t, x, y, z ) ≡ (t, r).

impulzus négyesvektor), vagy – Károlyházy Frigyesszemléletes elnevezésével – anyagvektor:

ahol

az energia és p az impulzus (lendület).

p μ ≡ (p 0, p 1, p 2, p 3) ≡ ( , px , py , pz ) ≡ ( , p).

Számunkra fontos lesz még a négyesáram-sûrûség:

ahol ρ a töltés- és j az áramsûrûség.

j μ ≡ (j 0, j 1, j 2, j 3) ≡ (ρ , jx , jy , jz ) ≡ (ρ , j).

Ezek a példák azt sugallják, hogy egy négyesvektorhárom térbeli komponense egy klasszikus fizikai (hár-mas)vektor, idõkomponense pedig egy klasszikusfizikai skalár. Ez azonban nincs mindig így! Éppen anégyeserõ egy olyan példa, ahol nem ilyen egyszerû ahelyzet, és ez kulcsfontosságú lesz.

Elõtte azonban ejtsünk pár szót a négyesvektoroktranszformációs tulajdonságairól. Egy téridõmennyiségdefiníciószerûen akkor négyesvektor, ha komponenseia koordinátarendszer forgatásakor úgy transzformá-lódnak, ahogy a téridõ-helyvektor komponensei. Ittnemcsak tisztán térbeli forgatásról lehet szó, hanemigazi téridõbeli „forgatásról” is! Az utóbbi hétköznapinyelvre lefordítva annak felel meg, amikor egyik iner-ciarendszerrõl áttérünk egy hozzá képest mozgó má-sik inerciarendszerre [1]. A megfelelõ transzformációka Lorentz-transzformációk. A legegyszerûbb eset ami-kor két inerciarendszer a közös x tengely mentén mo-zog egymáshoz képest v sebességgel, az y és z tenge-lyek párhuzamosak. Ebben az úgynevezett standardelrendezésben a t és x koordináták transzformálódnak,a v -re merõleges y és z koordináták nem. Ilyenkor be-

szélünk a t-x téridõsíkban (Minkowski-síkban) történõforgatásról, mivel a transzformációs képletek nagyonhasonlók a közönséges tér egy síkjában végzett forga-tást leíró kifejezésekhez. A geometriai kép a képlete-ket egyszerûbbé, átláthatóbbá teszi. A Lorentz-transz-formációt nem a bonyolult, (1−v 2)1/2-es faktort tartal-mazó képlettel célszerû leírni (ne felejtsük, c = 1). He-lyette sokkal egyszerûbb, ha az euklideszi síkbeli for-gatáshoz nagyon hasonlóan, tehát szögfüggvényekkelkezeljük a Minkowski-síkban történõ forgatást is.Egyetlen nagy különbség az, hogy míg az euklideszisíkban a trigonometrikus függvényeket (cos, sin, tg,…) használjuk, a Minkowski-síkban hiperbolikus függ-vényekkel (ch, sh, th, …) kell dolgozni. Az utóbbiakdefiníciója és tulajdonságai – sok más tankönyv vagyinternetes oldal mellett – [1]-ben is megtalálhatók. Ittcsak az alapdefiníciókat idézzük föl:

Egy nagyon fontos „hiperbolikus azonosságot” célsze-

chθ = eθ e−θ

2, shθ = eθ − e−θ

2, thθ = shθ

chθ.

rû még leszögezni:

A Minkowski-síkban történõ forgatás θ szöge (úgy-

ch2θ − sh2θ ≡ 1.

nevezett sebességparaméter) és az egyik inerciarend-szer másikhoz képesti (dimenziótlanított!) v sebessé-ge közötti kapcsolat:

v = thθ.

(Miközben a θ sebességparaméter −∞-rõl +∞-re válto-zik, tangens hiperbolikusza −1-rõl +1-re nõ.)

A θ sebességparaméter használatával egy tetszõle-ges a μ négyesvektor idõ- és térkomponenseinek Lo-rentz-transzformációja a standard elrendezésben akövetkezõ áttekinthetõ alakban írható:

(θ elõjele attól függ, hogy melyik inerciarendszerrõl

at′ = at chθ ax shθ ,

ax′ = at shθ ax chθ ,

ay′ = ay ,

az′ = az .

térünk át a másikra.)Ha valaki θ helyett a v sebességet szeretné használ-

ni, akkor könnyen átválthat a Lorentz-transzformáció„klasszikus”, négyzetgyökös kifejezéseket tartalmazóalakjára. Például:

chθ ≡ chθ1

≡ chθ

ch2θ − sh2θ≡ 1

1 − th2θ≡

≡ 1

1 − v 2

.

Ezek után nézzük magát az eredeti problémát!

264 FIZIKAI SZEMLE 2018 / 7–8

Page 51: Fizikai Szemle 68/7–8 – 2018. július–augusztusgoliat.eik.bme.hu/~f.simon/publications/Media/FizSzem-20180708.pdf · okokból standard modellnek hívunk, a világot anyagi és

Laboratóriumi rendszerben

Itt minden töltés nyugalomban van, ezért explicitenmég nincs szükség a relativitáselméletre. Felhasználhat-juk a Kiss Miklós cikkében leírt gondolatmenetet, csu-pán α lineáris töltéssûrûség helyett ρ térfogati töltéssû-rûséget használunk. A kettõ közti kapcsolat nyilvánvaló:α = ρA. Egy ρ térfogati töltéssûrûségû, A keresztmetsze-tû henger középtengelyétõl d távolságban (persze kívüla hengeren) a henger tengelyére merõleges irányú azelektromos térerõsség, amelynek nagysága

A tengelytõl d távolságra nyugalomban lévõ q pont-

E = 12 π

A ρd

.

töltésre ható erõ tehát

A konkrétság kedvéért legyen ρ és q is pozitív. Labora-

F = q E = q 12 π

A ρd

.

tóriumi rendszerben tehát a fenti képlettel leírható tisz-tán elektromos taszítás lép föl, mágneses mezõ nincs.

A laboratóriumihoz képest mozgóinerciarendszerben

Az elõzõ esethez viszonyítva két változás is van. Azegyik, hogy ebbõl a rendszerbõl nézve a szigetelõ tölté-sei v sebességgel mozognak, tehát megjelenik egy j ′ ≠ 0áramsûrûség is. Ez nem jelenthet problémát egy középis-kolásnak. Az már kevésbé nyilvánvaló, hogy a töltéssû-rûség sem lesz egyforma a két rendszerben, ρ ′ ≠ ρ. ALorentz-transzformáció fenti képleteit alkalmazva (ρ, j ≡0)-ból könnyen megkapjuk ρ ′-t és j ′-t:

Az áramsûrûség összefüggése relativitáselmélet nélkül

ρ ′ = ρ chθ ,

j ′ = ρ shθ ≡ ρ chθ thθ ≡ ρ ′ v.

is minden további nélkül érthetõ egy középiskolás szá-mára. A töltéssûrûség növekedésénél pedig a Lorentz-kontrakcióra – ami miatt adott mennyiségû töltés rövi-debb szakaszon oszlik el a laboratóriumihoz képestmozgó inerciarendszerbõl nézve – lehet utalni.

Ezek után már könnyû dolgunk van, alkalmazhat-juk a középiskolában megismert formulákat az elekt-romos és most már a mágneses mezõre is.

Az elektromos térerõsség nagysága most

Az I ′ = Aj ′ áramerõsség miatt kialakuló mágneses

E ′ = 12 π

A ρ ′d

= E chθ .

mezõ nagysága pedig

B ′ = 12 π

I ′d

= 12 π

A j ′d

= 12 π

A ρ ′ vd

=

= 12 π

A ρ ′d

v = E ′ v.

Vegyük észre, hogy A és d mindkét rendszerbenugyanaz, hiszen a téridõ-helyvektor (mint mindennégyesvektor) kölcsönös mozgásra merõleges kom-ponensei nem transzformálódnak!

A mezõk ismeretében – erre az esetre alkalmazva aLorentz-féle erõtörvényt – megkaphatjuk a q ponttöl-tésre ható erõ nagyágát:

(Megjegyzendõ, hogy a q elektromos töltés Lorentz-

F ′ = q (E ′ − v B ′) = q E ′ − v (E ′ v ) =

= q E ′ 1 − v 2 .

invariáns, tehát számértéke mindkét vonatkoztatásirendszerben ugyanaz.)

Látható, hogy az elektromos taszítás megnövekszik(E ′ = E chθ ), ugyanakkor a megjelenõ mágnesesmezõben mozgó töltésre föllép egy vonzó erõ. Kér-dés, hogy az elektromos taszítás növekedését kom-penzálja-e a megjelenõ mágneses vonzás. Vagyis,vajon ugyanaz-e az erõ értéke a kétféle inerciarend-szerben? Lássuk mit kapunk, ha behelyettesítjük afenti összefüggéseket!

A két erõ tehát NEM egyforma! Az elektromos taszítás

F ′ = q E chθ 1 − th2θ = q E chθ ch2θ − sh2θch2θ

=

= q E chθ 1

ch2θ= F 1

chθ.

növekedését túlkompenzálja a mágneses vonzás, amozgó rendszerbõl nézve – a laboratóriumi rendszer-hez képest – lecsökken a taszítás.

De ez nem is baj, ezt szeretném még röviden meg-mutatni.

Korábban – a standard elrendezés esetére fölírtösszefüggésekbõl – már láttuk, hogy a Lorentz-transz-formáció során egy négyesvektor kölcsönös mozgásramerõleges két térbeli komponense nem változik. Ezazonban csak a négyesvektorok térbeli komponen-seire igaz! Márpedig a középiskolában megtanult

F = q (E+v × B)

Lorentz-erõ ebben a formában nem egy négyesvektorhárom térbeli komponense! Ennek belátását ne az Eés B transzformációs tulajdonságainak vizsgálatáraalapozzuk, mert az még a jó középiskolás diákokszámára is nehezebb gondolatmenet. Helyette köves-sünk egy talán egyszerûbb utat.

A kulcsot az a tény adja, hogy minden olyan eset-ben, amikor idõderiválással származtatunk egy újmennyiséget (sebesség, gyorsulás, …), figyelembekell venni, hogy a koordinátaidõ és az úgynevezettsajátidõ különböznek egymástól. A klasszikus hár-masvektoroknál a koordinátaidõ szerint deriválunk,míg a négyesvektoroknál a sajátidõ szerint.

Annak érdekében, hogy lássuk mire vezet ez atény, induljunk ki egy anyagi objektum sorsának kétközeli eseményét (a téridõben világvonala két közelipontját) összekötõ téridõ-helyvektorból:

A FIZIKA TANÍTÁSA 265

Page 52: Fizikai Szemle 68/7–8 – 2018. július–augusztusgoliat.eik.bme.hu/~f.simon/publications/Media/FizSzem-20180708.pdf · okokból standard modellnek hívunk, a világot anyagi és

dx μ ≡ (dt, dx, dy, dz ) ≡ (dt, dr).

A komponensek (így dt is!) attól függenek, hogy mi-lyen vonatkoztatási rendszerben adjuk meg azokat.Ezzel szemben a speciális relativitáselmélet egyikalapösszefüggése, hogy az idõkomponens négyzeté-nek és a térbeli komponensek négyzetösszegénekkülönbsége olyan kifejezés (négyesvektorunk négy-zetes téridõhossza), amelynek értéke nem függ attól,hogy milyen inerciarendszerben számoljuk ki (inva-riáns). Ez az érték azon dτ sajátidõtartam négyzete,amelyet egy olyan inerciarendszerben mérünk,amelyben a két esemény ugyanott történik (dr = 0).Írhatjuk tehát, hogy

(Ne felejtsük, hogy c = 1, v2 = th2θ és ch2θ− sh2θ ≡ 1!)

dτ := dt 2 − dx 2 − dy 2 − dz 2 ≡ dt 2 − dr2 ≡

≡ dt 1 − v2 ≡ dtchθ

.

A dτ sajátidõ és a nála chθ -szor nagyobb koordiná-taidõ kis sebességekre (v << 1) alig különbözik.

Egy négyesvektorból idõderiválással újabb négyes-vektor nyerhetõ, de csak akkor, ha azt nem a koordi-nátaidõ, hanem az invariáns sajátidõ szerint végezzük.A sebesség, impulzus, erõ relativisztikusan helyeskifejezései tehát a következõk.

Négyessebesség:

Könnyen belátható, hogy ez egy egységvektor a tér-

u μ = dx μ

dτ= dx μ

dtdtdτ

=⎛⎜⎝

⎞⎟⎠

dtdt

, drdt

dtdτ

=

= (1, v) chθ .

idõben, iránya pedig a test világvonalának minden-kori érintõje mentén mutat.

Négyesimpulzus (energia-impulzus négyesvektor,más kifejezéssel anyagvektor):

ahol m az adott részecske invariáns tömege.

p μ ≡ ( , p) = m u μ = m (1, v) chθ,

Végül pedig a négyeserõ:

Az utolsó átalakítás a newtoni mechanikából ismert

f μ = dp μ

dτ= dp μ

dtdtdτ

=⎛⎜⎝

⎞⎟⎠

ddt

, dpdt

chθ =

= (F v, F) chθ .

munkatétel, illetve impulzustétel (amibõl nekünkmost a második a fontos), ahol F a klasszikus három-dimenziós erõ.

Azt kaptuk tehát, hogy nem a klasszikus hármaserõmerõleges komponense marad változatlan a Lorentz-transzformáció során, hanem a négyeserõé, és azutóbbi chθ -szorosa az elõbbinek! Pontosan ezt tük-rözi amit korábban kaptunk, amikor a Lorentz-féleerõképletet nemrelativisztikusan, a chθ szorzó nélkülalkalmaztuk.

Annak oka egyszerû, hogy az eredeti írásban miértnem könnyen vehetõ észre a kegyes csalás. Az egy-ségnyi hosszú darabban lévõ ponttöltések darabszá-ma a Lorentz-kontrakció miatt éppen egy chθ szorzó-faktorral növekszik meg, ami pont kompenzálja azegyetlen töltésre ható hármaserõ csökkenését. Vagyis(legalábbis ebben a speciális esetben) az egységnyihosszra ható hármaserõ nagysága valóban megegye-zik a két inerciarendszerben.

A leírtaktól teljesen függetlenül még egy értelemza-varó nyomtatási hibát is megemlítek: az eredeti cikkelején, a 2. Maxwell-egyenlet sztatikus elektromosmezejénél az „örvénymentes” helyett tévedésbõl „for-rásmentes” szerepel.

Végül, nehogy félreértés legyen, szeretném leszö-gezni: Kiss Miklós cikke középiskolások számára na-gyon tanulságos és figyelemfelkeltõ, érdemeit semmi-képpen sem kívánom kisebbíteni, sõt. Jelen írássalcsupán az alapprobléma még pontosabb megértésé-hez szeretnék hozzájárulni.

Irodalom1. Erostyák János, Kürti Jenõ, Raics Péter, Sükösd Csaba: Fizika

III. (szerk.: Erostyák János, Litz József), Nemzeti Tankönyvki-adó, Budapest–Debrecen–Pécs 2006, V. rész: Relativitáselmélet,201–252.

Hogyan

érkezett

a Curiosity

a Marsra?

V FAN ÚJ A ÖLD FELETT

Tölt

sed

le!

Nézzed

meg

!M

utasd

meg

másokn

ak!

Tan

ítsd

meg

diá

kja

idn

ak!

Keresd a fizikaiszemle.hu mellékletek menüpontjában!

266 FIZIKAI SZEMLE 2018 / 7–8

Page 53: Fizikai Szemle 68/7–8 – 2018. július–augusztusgoliat.eik.bme.hu/~f.simon/publications/Media/FizSzem-20180708.pdf · okokból standard modellnek hívunk, a világot anyagi és

XXI. ORSZÁGOS SZILÁRD LEÓ NUKLEÁRIS

1. táblázat

Nevezések megoszlása 2014-ben, zárójelbena 2013. évi adatok

I. kategóriás II. kategóriás

fiú lány fiú lány

budapesti 106 (72) 21 (8) 34 (33) 17 (5)

vidéki 94 (116) 10 (16) 38 (35) 13 (4)

határon túli 1 (0) 0 (0) 0 (0) 0 (0)

összesen 201 (168) 31 (24) 72 (68) 30 (9)

TANULMÁNYI VERSENY – 1. rész

Sükösd Csaba (1947) a BME címzetes egye-temi tanára, az ELFT elnökségi tagja. Kísér-leti magfizikus, aki kísérleti munkáját nagy-részt külföldi kutatóintézetekben végezte.Kutatási területe a magreakciók, óriásrezo-nanciák és némely asztrofizikailag relevánsmagreakció vizsgálata radioaktív ionnyalá-bokkal. Marx György tanítványaként résztvett a 70-es évek MTA oktatási kísérletében.Azóta is szoros kapcsolata van a fizikataná-rok közösségével, több tanár- és oktatássalkapcsolatos program vezetõje.

Sükösd CsabaBME Nukleáris Technikai Intézet

Éppen 20 évvel ezelõtt, Szilárd Leó születésének cen-tenáriuma alkalmából, Marx György professzor kez-deményezésére 1998-ban került elõször megrende-zésre a Szilárd Leó Országos Középiskolai Tanulmá-nyi Verseny. Azóta a Szilárd Leó TehetséggondozóAlapítvány, az Eötvös Loránd Fizikai Társulat, azEnergetikai Szakgimnázium és Kollégium és a MagyarNukleáris Társaság minden évben megrendezi a ver-senyt. A verseny célja a modern fizikai ismeretek,ezen belül az atomfizika iránti érdeklõdés felkeltéseés annak minél szélesebb és mélyebb megismerteté-se. A verseny elõ kívánja segíteni a nukleáris szakte-rület számára szükséges szakember-utánpótlás kine-velését az elkövetkezendõ években, évtizedekben.Erre az atomerõmû üzemidejének meghosszabbításaés a tervezett új blokkok építésének és üzemeltetésé-nek érdekében elengedhetetlenül szükség van. 2006óta határon túli magyar anyanyelvû iskolák tanulóirészére is megnyitottuk a részvétel lehetõségét. 2015után idén ismét volt jelentkezõ a határon túlról, igaz,csak egyetlen fõ: a Székely Mikó Kollégium (Sepsi-szentgyörgy, Románia) egyetlen elsõ kategóriás (11–12. osztályos) fiút nevezett a versenybe. Sajnos, Szer-biából és Horvátországból, valamint Kárpátaljáról és aFelvidékrõl ez évben sem kaptunk nevezést. Össze-sen 232 elsõ kategóriás és 102 junior kategóriás neve-zés érkezett. Örvendetesen, ez több tanuló érdeklõ-dését jelzi, mint az elmúlt években, megoszlásukatmutatja az 1. táblázat.

Feltûnõ a budapestiek létszámának és arányánaknövekedése, valamint – örvendetes módon – a lányokjelentkezési kedve is megnõtt, különösen a II. (Junior)kategóriában.

A 2018. február 19-én megtartott elsõ forduló (válo-gató verseny) tíz feladatát az iskolákban, három óraalatt lehetett megoldani. Kijavítás után a tanárok azonmegoldásokat küldték be a BME Nukleáris TechnikaTanszékére, ahol a 9–10. osztályos (junior) verseny-zõk legalább 40%-os, a 11–12. osztályos (I. kategó-riás) versenyzõk legalább 60%-os eredményt értek el.A verseny fordulóin másokkal (és az internettel) való

kommunikációt lehetõvé tevõ eszközök kivételévelbármilyen segédeszköz használható volt.

Az alábbiakban ismertetjük a válogató verseny,valamint a döntõ feladatait, és a megoldókulcsot,amely a javító tanárok számára jelentett iránymutatást.A tanárok természetesen minden helyes ötletet ésmegoldást is pontozhattak. Olyanokat is, amelyekeltértek a megoldókulcsban bemutatott megoldástól.Minden feladatra maximálisan 5 pontot lehetett adni.

A válogató verseny feladatai és megoldásuk1. feladat

Válaszoljunk az alábbi tudománytörténeti kérdésekre:a) Ki szabadalmaztatta a neutronos láncreakciót?b) Kirõl mondják, hogy az elsõ reaktormérnök volt?c) Heisenbergnél doktorált, majd a hidrogénbomba

atyjának is nevezték. Ki volt õ?d) A radioaktív nyomjelzés ötlete tõle származik. Ki

volt õ?e) Ki adta a radioaktivitás nevet a sugárzási jelen-

ségeknek?

Megoldása) Szilárd Leó, b) Wigner Jenõ, c) Teller Ede, d) He-vesy György, e) Marie Curie. (Minden helyes válasz1-1 pontot ér.)

2. feladat

Hogyan változna meg a H-atom mérete és energia-szintjei, ha

a) az elektron tömege kétszeresére nõne,b) az elektron tömege felére csökkenne,c) a proton tömege kétszeresére nõne,d) az elektron töltése felére csökkenne,e) a proton töltése kétszeresére nõne?

A megoldásnál használd a mellékelt táblázatot!

A FIZIKA TANÍTÁSA 267

Page 54: Fizikai Szemle 68/7–8 – 2018. július–augusztusgoliat.eik.bme.hu/~f.simon/publications/Media/FizSzem-20180708.pdf · okokból standard modellnek hívunk, a világot anyagi és

Megoldás

Csillagok által kibocsátott fény maximális intenzitásnál mért hullám-hossza a felszíni hõmérséklet függvényében.

12

10

8

6

4

2

0hu

llám

ho

ssz

10m

egys

égb

en–7

0 5000 10000 15000 20000 25000hõmérséklet (K)

y x= 29944 · –1,004

R = 0,9999

Az ajánlott irodalmakban több helyen is szerepel-nek a következõ, H-atomra vonatkozó képletek:

Ezek alapján a válaszok könnyen megadhatók. (Alap-

rn =ε0 Ł2

π me q 2e

n 2 , valamint En = −me q 4

e

8 Ł2 ε 20

1

n 2.

állapotban n = 1.) A töltésváltozások hatásának meg-ítéléséhez vegyük figyelembe, hogy a töltések a Cou-lomb-energiából származnak, azaz például a -benq 2

e

az egyik qe a proton töltésébõl, a másik qe az elektrontöltésébõl származik. Hasonlóképpen, a -benq 4

e q 2e

származik a proton, és az elektron töltésébõl.q 2e

(Megjegyzés: egyes függvénytáblázatokban hibásanszerepel a H-atom energiájának képlete: a töltés hat-ványkitevõje hiányzik. Ennek alapján készült megol-dásokat nem lehet elfogadni.)

H-atom

paraméterénekváltozása

méreténekváltozása

energiaszintjeinekváltozása

a) Elektron tömegekétszeresére nõ

Csökkenne azelektron kvantumosnyüzsgése, ezértfelére csökkenne azatom egyensúlyimérete

A negatívenergiaszintekkétszereséremélyülnének

b) Elektron tömegefelére csökken

Nõne az elektronkvantumosnyüzsgése, ezértkétszeresére nõneaz atom egyensúlyimérete

A negatívenergiaszintekfelérecsökkennének

c) Proton tömegekétszeresére nõ

A méretet a protontömege nembefolyásolná

Energiaszintekváltozatalanokmaradnának

d) Elektron töltésefelére csökken

A proton vonzásacsökkenne, a méretkétszeresre nõne

A negatívenergiaszintekenergiájanegyedérecsökkenne

e) Proton töltésekétszeresére nõ

A proton vonzásanõne, a méret afelére csökkenne

Az energiaszinteknégyszer mélyebbeklennének

Mind a 10 részfeladat fél-fél pontot ér.

3. feladat

Egy 1 MeV energiájú gamma-foton kölcsönhatásba lépa) egy nyugvó elektronnal;b) egy nyugvó H-atommal;c) egy nyugvó atommaggal.

Mindhárom esetben csak foton – részecske között lépfel kölcsönhatás, és tegyük fel, hogy a fotonnal köl-csönható atomi részek egyben maradnak.

Kérdések:1) Mely kölcsönhatás(ok)nál nyelõdhet el (részecs-

keként megsemmisül) a foton?2) Ha elvileg létrejöhet a kölcsönhatás, akkor mi-

lyen feltételnek kell teljesülni?

Megoldás1–2) Csak az atomaggal való kölcsönhatás során

történhet foton elnyelõdése. Ennek feltétele, hogy azatommagnak legyen olyan energiaszintje, amelyregerjeszthetõ a mag úgy, hogy a lendület- és energia-megmaradás is teljesüljön. (2 pont)

A másik két esetben ez nem jöhet számításba:Nyugvó elektronnál azért nem, mert az elektronnak

nincs belsõ szerkezete, így az nem gerjeszthetõ.Ugyanakkor, mint nyugalmi tömeggel rendelkezõ ré-szecske, nem tudja átvenni a fotontól annak lendüle-tét, és az azzal arányos energiáját is. Könnyen belát-ható, hogy ellentmondásra jutunk. Tegyük fel, hogy ameglökött elektron átveszi a foton pγ lendületét. Ekkoraz energiamegmaradás a következõképpen írható fel:

hiszen a kezdõ állapotban van egy foton és egy nyug-

pγ c m0 c 2 = pγ c 2 m0 c 22

,

vó elektron (az egyenlet bal oldala, m0 az elektronnyugalmi tömege), végállapotban pedig csak egyelektron van, amelynek lendülete megegyezik a fotonlendületével (az egyenlet jobb oldala).

Négyzetre emelés után azt kapjuk, hogy az egyen-lõség akkor teljesülhet, ha

Mivel m0 > 0, ez csak akkor teljesíthetõ, ha pγ = 0 len-

2 pγ c m0 c 2 = 0,

ne. Ekkor viszont nincs foton. (2 pont)b) Ugyanez a helyzet a H-atom esetében is. Igaz

ugyan, hogy a H-atom gerjeszthetõ, de a foton ener-giája túlságosan nagy (MeV) a 13,6 eV ionizációsenergiához képest, tehát a H-atom szétesne, ha ekko-ra energiát nyelne el (1 pont).

4. feladat

Az alábbi ábra néhány csillag által kibocsátott fénymaximális intenzitásnál mért hullámhosszát és a csil-lag felszíni hõmérsékletének (K) kapcsolatát mutatja.A hullámhosszak a függõleges tengelyen 10−7 m egy-ségben vannak. Az ábrán szereplõ csillagok névsor-ban: Achernar, Arcturus, Betelgeuse, Deneb, ProximaCentauri, Rigel, Sirius, Spica.

268 FIZIKAI SZEMLE 2018 / 7–8

Page 55: Fizikai Szemle 68/7–8 – 2018. július–augusztusgoliat.eik.bme.hu/~f.simon/publications/Media/FizSzem-20180708.pdf · okokból standard modellnek hívunk, a világot anyagi és

Melyik törvény olvasható ki a grafikonból? Határoz-zuk meg a törvényben szereplõ állandó értékét a gra-fikon alapján!

MegoldásAz ábrán szerepel az Excel által megadott illesztés

képlete is. Ezt a hullámhosszal és a hõmérséklettelfelírva kapjuk (jó közelítéssel)

Azaz a két paraméter között jó közelítéssel fordított

λ = 29 944T

.

arányosság áll fenn. (Ne felejtsük el, hogy itt a hul-lámhossz 10−7 m egységekben van!) (2 pont)

Átrendezve ezt a következõ módon is írhatjuk:

λT = konstans.

Ez a Wien-féle eltolódási törvény. (2 pont)Ha a hullámhosszat méterben, a hõmérsékletet kel-

vinben adjuk meg, akkor a konstans az illesztés alap-ján: 2,9944 10−3 m K (1 pont). A Wien-féle konstansirodalmi értéke: 2,8978 10−3 m K.

5. feladat

Korunk egyik legnagyobb mûszaki teljesítményénekszámító, a CERN-ben megépített LHC (Large HadronCollider = Nagy Hadronütköztetõ) gyorsítóját 2008-bankapcsolták be elõször. A föld alá helyezett, közel köralakú 26,7 km kerületû gyorsítóban 7 TeV (tera = 1012)energiájú protonok keringenek és ütköznek. A teljeskerület mentén 2808 csomagban keringenek a proto-nok. Egy csomagban 1,15 1011 darab proton van.

a) Mekkora egy protoncsomag teljes energiája?b) Ha egy 150 kg tömegû kismotor ekkora mozgá-

si energiával rendelkezne, mekkora sebességgel mo-zogna?

c) Mekkora a teljes kerület mentén egy iránybanmozgó protonok energiája?

d) Mekkora tömegû 25 °C hõmérsékletû aranytöm-böt lehetne megolvasztani ekkora energiával?

Adatok: az arany C fajhõje 126 J/(kg °C), olvadás-pontja: 1337,6 K, L olvadáshõje 64,9 kJ/kg.

Megoldása) Egyetlen részecske energiája:

Tehát E = 1,15 1011 1,12 10−6 J = 1,29 105 J egy

7 TeV = 7 1012 eV 1,6 10−19 JeV

= 1,12 10−6 J.

csomag teljes energiája. (1 pont)b) A kismotor

v = 2 Em

= 2,58 105

150= 41,47 m

s≈ 149 km

h

sebességgel száguldana. (1 pont)

c) A teljes kerület mentén egy irányba mozgó ösz-szes proton Eössz =1,29 105 2808 = 362,2 MJ energiá-val rendelkezik. (1 pont)

d) Ezen összenergia az arany olvadáspontig valómelegítésére és ott megolvasztására fordítódik, azaz

amibõl kapjuk hogy

Eössz = C m Δ T L m ,

tömegû aranyat olvaszthatnánk meg. (2 pont)

m =Eössz

C Δ T L= 1849 kg

6. feladat

Legalább mekkora a nukleonok keV egységekbenkifejezett kinetikus energiája az A = 125 tömegszámúatommagon belül? Adjunk rá becslést!

Adatok: r0 = 1,2 10−15 m, mnukleon ≈ 1,67 10−27 kg,¥ = 1,055 10−34 Js.

MegoldásEzen atommag sugara

A határozatlansági relációból:

r = r0

3A = 1,2 10−15 5 = 6 10−15 m.

azaz

Δ x Δ px ≥ ¥,

A hely bizonytalanságát becsüljük az atommag átmé-

Δ px ≥ ¥Δ x

.

rõjével, azaz Δx = 2 r = 1,2 10−14 m. Természetesen amásik két dimenzióra is ugyanígy igaz, azaz Δx = Δy =Δz. Ezekbõl következik, hogy

a nukleon minimális lendülete. (2 pont)

Δ px = Δ py = Δ pz ≥ ¥Δ x

A nukleon kvantumnyüzsgésbõl származó

mozgási energiájára a lendületbizonytalanságból ad-

E = (Δ p )2

2 m=

Δ p 2x Δ p 2

y Δ p 2z

2 m= 3 ¥2

2 m Δ x 2

hattunk becslést. (2 pont)Behelyettesítve a Δx = 1,2 10−14 m helybizonyta-

lanságot, kapjuk: E ≥ 434 keV. (1 pont)Természetesen ez csak a legalacsonyabb energiájú

állapotban lévõ nukleonok kinetikus energiája. APauli-elv miatt a többi nukleon egyre nagyobb kineti-kus energiájú állapotba kényszerül, így a legmaga-sabb energiájú betöltött állapotban lévõ nukleon ki-netikus energiája ennél sokkal nagyobb – akár 30MeV is – lehet.

A FIZIKA TANÍTÁSA 269

Page 56: Fizikai Szemle 68/7–8 – 2018. július–augusztusgoliat.eik.bme.hu/~f.simon/publications/Media/FizSzem-20180708.pdf · okokból standard modellnek hívunk, a világot anyagi és

7. feladat

Létezik-e olyan atommag, amely pozitronkibocsátás-sal bomlik, de elektronbefogással nem? Indokoljukmeg!

MegoldásA pozitronkibocsátás során:

az atommagban egy proton neutronná alakul át, mi-

AZ X → A

Z − 1 Y e ν

közben egy pozitront és egy neutrínót bocsát ki. (1pont)

Az elektronbefogás során:

is az atommag egy protonja alakul át neutronná, mi-

AZ X e− → A

Z − 1 Y ν

vel befog egy elektront az atommag körül elhelyezke-dõ elektronok közül (legvalószínûbben a maghozlegközelebb esõ 1s pályáról), és egy neutrínó bocsá-tódik ki. (1 pont)

Hasonlítsuk össze a két folyamat energetikai felté-telét! Mivel földi világunk semleges atomokból áll, cél-szerû a teljes folyamat (semleges atomokból semlegesatomok képzõdése) teljes energiáját összehasonlítani.

Írjuk fel a fenti reakciókat, figyelembe véve azegyes atomok elektronjainak számát is! Elõször azegyenlet mindkét oldalához adjunk Z darab elektront!

A pozitronemisszió:

Az egyenlet mindkét oldalán a szögletes zárójelek-

AZX Z e− → A

Z − 1 Y (Z − 1) e− e− e ν.

ben a semleges atomok összetétele szerepel. Mivel aleánymag rendszáma eggyel kisebb, azért az egyenletmindkét oldalához hozzáadott Z darab elektronból azegyenlet jobb oldalán, a pozitron mellett egy elektronkülön jelenik meg.

Az elektronbefogás:

Itt az elektron befogása miatt a végállapotban már

AZX Z e− → A

Z − 1 Y (Z − 1) e− ν.

eleve eggyel kevesebb (Z−1) elektron marad. Ezekalapján a pozitronemisszió feltétele:

(1 pont), az elektronbefogás energetikai feltétele pe-

MX c 2 > MY c 2 2 me c 2

dig:

ahol me az elektron/pozitron tömege vonatkozik, MX

MX c 2 > MY c 2,

és MY pedig a semleges atomok tömege. (1 pont)Nyilvánvaló, hogy amennyiben az elõbbi feltétel

teljesül, akkor szükségképpen teljesül az utóbbi is. (1pont)

Ebbõl következik, hogy minden pozitront emittálóatommag befoghat elektront, de fordítva ez nem fel-tétlenül igaz. Tehát nem létezik olyan atommag,amely csak pozitronkibocsátással bomlik, de elekt-ronbefogási folyamata nincs.

8. feladat

Egy laboratóriumból egy igen hosszú, állandó átmé-rõjû kábelcsatornán folyamatosan radioaktív torongáz szivárog. A csatornában 2 m/s sebességgel haladelõre a levegõ, amelyben egyenletesen oszlik el aszennyezõ gáz. A laboratóriumtól 10 m távolságbana csõ mellett a megengedett sugárzás 16-szorosátmérik. A toron a radon 220-as izotópja és felezésiideje 56 s.

a) A laboratóriumtól mekkora távolságra éri el asugárzás a megengedett szintet?

b) A laboratóriumtól mekkora távolságban lesz asugárzás a megengedett érték háromszorosa?

MegoldásA mérés helyén a megengedett érték 16-szorosát

mérik, ezért a felezési idõ négyszeresének kell eltel-nie ahhoz, hogy a sugárzás értéke megfelelõ legyen.Ez t = 224 s. Ezen idõ alatt a gáz a csatornában s = v t= 448 m méter utat tesz meg. Tehát a laboratóriumtól10+448 = 458 m távolságra lesz a sugárzás intenzitásaelfogadható. (2 pont)

A mérés helyétõl 448 méterre a sugárzás intenzitásaa kezdeti A0 érték 1/16-od része. A keresett helyenennek háromszorosát mérhetjük. Az aktivitás t ′ idõmúlva lesz ennyi.

Logaritmálás és az adatok behelyettesítése után a t ′

3A0

16= A0 2

− t ′T1/2 .

idõ kifejezhetõ:

Ennyi idõ alatt a levegõ s ′ = v t ′ = 270,48 métert ha-

t ′ = −56lg 3

16lg2

= 135,24 s.

lad elõre a mérõhelytõl. Tehát a labortól 10+270,48 =280,48 méterre lesz az aktivitás a megengedett értékháromszorosa. (3 pont)

9. feladat

20 MeV energiájú protonok 500 mA-es nyalábja egycéltárgyra esik, ahol teljesen elnyelõdik.

a) Másodpercenként hány proton nyelõdik el?b) Mekkora teljesítménnyel fûti a protonnyaláb a

céltárgyat, ha a protonok teljes kinetikus energiájahõvé alakul?

c) Hány százalékkal nagyobb a protonok relativisz-tikus tömege, mint nyugalmi tömege?

d) Mekkora erõt fejt ki a protonnyaláb az elnyelõcéltárgyra?

270 FIZIKAI SZEMLE 2018 / 7–8

Page 57: Fizikai Szemle 68/7–8 – 2018. július–augusztusgoliat.eik.bme.hu/~f.simon/publications/Media/FizSzem-20180708.pdf · okokból standard modellnek hívunk, a világot anyagi és

Megoldás

2. táblázat

Az I. forduló után beküldött dolgozatok megoszlása,zárójelben a 2017. évi adatok

I. kategóriás II. kategóriás

budapesti 16 (24) 8 (13)

vidéki 28 (29) 15 (12)

határon túli 0 (0) 0 (0)

összesen 44 (53) 23 (25)

a) Az 500 mA áram azt jelenti, hogy másodpercen-ként 500 mC töltés halad át egy felületen (például acéltárgyon). Mivel egy proton töltése 1,6 10−19 C,ezért a másodpercenként

számú proton nyelõdik el. (1 pont)

N = 500 10−6

1,6 10−19= 3,125 1015

b) A 20 MeV energia olyan, mintha 20 MV feszült-séggel gyorsítottuk volna a protonokat. Azaz

a nyaláb (villamos) teljesítménye. (1 pont)

P = U I = 20 106 V 500 10−6 A = 10 000 W =

= 10 kW

c) A teljes energia és a mozgási energia kapcsolata:mc 2 = Ekin + m0 c 2. Ebbõl a teljes relativisztikus tömegéppen a mozgási energiának megfelelõ tömeggel na-gyobb a nyugalmi tömegnél. Azaz

a keresett növekmény. (1 pont)

Ekin

m0 c 2= 2,13%

d) A protonnyaláb által kifejtett erõt az idõegység(1 s) alatt a céltárgynak átadott lendületbõl tudjukmeghatározni. Mivel a protonok mozgási energiájasokkal kisebb, mint a nyugalmi tömegébõl számítottenergia, ezért számolhatunk klasszikusan. Ekkoregyetlen proton lendülete:

Elnyelõdésekor minden proton ennyi lendületet ad át

p = 2 m0 Ekin =

= 2 1,67 10−27 kg 20 106 eV 1,6 10−19 JeV

=

= 1,03 10−19 kg ms

.

a céltárgynak. Azt viszont korábban kiszámoltuk,hogy 1 s alatt 3,125 1015 proton nyelõdik el, így azösszes lendületátadásból (mivel Δt = 1 s)

a nyaláb által kifejtett erõ. (2 pont)

F = Δ pΔ t

= 3,125 1015 1,03 10−19 =

= 3,22 10−4 N

10. feladat

Mekkora a felszíni hõmérséklete annak a Naphozhasonló csillagnak, amelynek sugara 750 ezer kilomé-ter, és a csillagtól mért 220 millió kilométer távolság-ban a neutrínófluxus értéke 5,9 1013 1/(m2 s) (vagyis1 m2 merõleges felületen másodpercenként ennyineutrínó halad át)?

MegoldásA Naphoz hasonló csillagokban a fúziós energiater-

melés a proton-proton ciklusban történik, amelybensok részfolyamat során összességében a

folyamat zajlik. (1 pont)

4 11H → 4

2He 2 e 2 νe 26,22 MeV

A csillagtól mért 220 millió kilométer távolságbantalálható φν neutrínófluxusból a csillagban másodper-cenként keletkezõ neutrínók száma meghatározható, hi-szen ha ismerjük, hogy mennyi neutrínó halad át má-sodpercenként 1 m2-en, akkor ki tudjuk számolni, hogy

halad át a d = 220 millió kilométer sugarú gömb teljes

Δ Nν

Δ t= φ ν 4 π d 2 = 3,5885 1037 1

s

felszínén. (1 pont)Mivel minden 26,22 MeV felszabadult energia két

neutrínó keletkezésével jár, ezért minden neutrínóhoz13,11 MeV energia felszabadulása tartozik, azaz

a csillag teljesítménye. (1 pont)

P = 12

Δ Nν

Δ t26,22 MeV

s= 7,527 1025 W

A csillag felszínének (R sugarának) ismeretében aStefan–Boltzmann-törvény alapján

felszíni hõmérséklet adódik. (2 pont)

T =

4P

σ 4 π R 2≈ 3700 K

Az elõdöntõ eredményei

Az elõdöntõ feladatait 44 fõ I. kategóriás, és 23 fõjunior versenyzõ teljesítette olyan szinten, hogy dol-gozataikat a javító tanárok be tudták küldeni a BMENukleáris Technika Tanszékére további értékelés ésrangsorolás végett. Ezek megoszlását mutatja a 2.táblázat. (A zárójelben lévõ számok itt is a 2017-eseredmények.)

A vidéki iskolák a hibahatáron belül 2018-ban isugyanolyan eredményesek voltak mindkét kategóriá-

A FIZIKA TANÍTÁSA 271

Page 58: Fizikai Szemle 68/7–8 – 2018. július–augusztusgoliat.eik.bme.hu/~f.simon/publications/Media/FizSzem-20180708.pdf · okokból standard modellnek hívunk, a világot anyagi és

ban, mint 2017-ben. A budapestiek azonban gyen-gébben teljesítettek, ami meglepõ, hiszen 2018-banjóval több budapesti versenyzõ regisztrált (178), mint2017-ben (118). Enyhe és nem arányos emelkedéstvártunk, hiszen sok olyan budapesti tanuló (és isko-la) lehetett, aki elõször ismerkedett az Országos Szi-lárd Leó Verseny feladataival. Viszont örvendetes abudapesti junior kategóriás tanulók tavalyinál lénye-gesen jobb szereplése, ami reményt ad arra, hogy akövetkezõ években felkészültebb tanulók versenyez-nek majd. Sajnos, határon túli iskolában továbbrasem született olyan dolgozat, amely elérte volna a to-vábbküldési szintet.

Az Országos Szilárd Leó Fizikaverseny elsõ fordu-lójából beküldött dolgozatok ellenõrzése után egyegyetemi oktatókból álló bírálóbizottság a legjobb 10junior versenyzõt és a legjobb 20 elsõ kategóriás ver-senyzõt hívta be a 2018. április 21-én megrendezett

döntõre. Sajnos két bejutott elsõ kategóriás versenyzõcsak túl rövid idõvel a verseny elõtt jelezte, hogy nemtud részt venni, így a döntõben szereplõ elsõ kategó-riás tanulók száma 18-ra csökkent.

Idén négy lány jutott a verseny döntõjébe: CsuhaBoglárka és Krasznai Anna (Vajda János Gimnázium,Keszthely), valamint Világos Blanka (Szent IstvánGimnázium, Budapest) az I. kategóriában, továbbáSajgó Anna Mária (SZTE Gyakorló Gimnázium és Ál-talános Iskola) a juniorok között.

A verseny döntõjét – mint eddig minden évben –Pakson, az Energetikai Szakgimnázium és Kollégium-ban (ESZI) rendeztük 2018. április 20. és 22. között.A döntõ zökkenõmentes lebonyolításáért CsajágiSándor igazgatóhelyettes úrnak, valamint CsanádiZoltán igazgató úrnak tartozunk köszönettel.

Folytatása következik.

A TIZEDIK NUKLEÁRIS SZAKTÁBOR az MNT tanári tagozatának elnöke

Mester András Rátz Tanár Úr Életmûdíjasnyugalmazott fizikatanár a Kossuth LajosTudományegyetem matematika-fizika sza-kán diplomázott. A kezdeményezésére indí-tott Városi Fizikavetélkedõt tizennégy évenát szervezte a Diósgyõri Gimnáziumban.Közel húsz éve vesz részt az ELFT és azMNT munkájában. 1998 óta tagja a SzilárdLeó Fizikaverseny versenybizottságának.2007 óta egyik fõ szervezõje az MNT Nuk-leáris Szaktáborának, ahol részt vesz a prog-ramok összeállításában és lebonyolításában.

Mester András

Felsõfokú mûszaki szakemberek iránti igényaz energetikábanA Magyar Nukleáris Társaság (MNT) hamar felismerte,hogy a duális képzések korában fontos, hogy már aközépiskolás diákokat megnyerjék a megfelelõ szak-területek számára. Nagyon sok tehetséges diák azértnem választ bizonyos szakmákat, mert nem rendelke-zik megfelelõ ismeretekkel. Éppen emiatt lényeges,hogy még a pályaválasztás elõtt megfelelõ tájékozott-sággal bírjanak a fiatalok. (A felsõfokú mûszaki vég-zettségûek közül speciális energetikai, atomerõmûvi(szakmérnöki) továbbképzésben részesültek aránya2009-ben 11% volt.)1 Annak érdekében, hogy a nuk-

1 Aszódi Attila: Az energetikai, mûszaki képzés és tájékoztatáshelyzete. Magyar Energetikusok Kerekasztala, 2009.06.10., 22. old.http://www.mee.hu/files/images/Asz__di.pdf

leáris ipar szakemberigénye kielégíthetõ legyen, ajövõ atomerõmûveinek, kutatóintézeteinek, oktatásiintézményeinek humán erõforrásáról idõben gondos-kodni kell (1. ábra). A nukleáris képzés jelentõségemegnõtt, hiszen elkezdõdött a két új paksi blokk épí-tésének elõkészítése. Miután ezek a blokkok 60 évenát mûködnek majd, az atomerõmû és a hozzá kapcso-lódó tudományos, szakmai háttérintézmények ishosszú távra kínálnak perspektívát a fiataloknak.

Nukleáris szaktáborok

A nukleáris szakemberképzés erõsítése, a tehetség-gondozás igénye már korábban is számos alkalommalfelmerült, mielõtt 2007 tavaszán megfogalmazódott anukleáris szaktábor megvalósításának ötlete. Középis-kolások részére az I. Nukleáris Szaktábort 2007 nya-rán indította az MNT. Az elsõ tábor sikerét követõenaz MNT fontosnak tartotta a tábor következõ évekbenvaló megszervezését is.

A tábor kiváló lehetõséget jelent a fiatalok számára aszakmai ismeretek elsajátítására, a területen dolgozószakemberek megismerésére. A táborban megszerzetttudás és képességek alapján a fiatalokat be lehet vonnia tudományos kutatásokba, elõsegítve szakmai fejlõdé-süket, egyúttal megalapozva az utánpótlásképzést.

Az eltelt 10 év lehetõséget ad némi elemzésre is: avisszajelzések alapján a táborozók közül – akik meg-határozó élményként tekintenek a táborra – többenválasztották a fizikusi vagy mérnöki pályát. Örömmeltapasztaljuk, hogy velük egyre többször találkozunkszakmai rendezvényeken.

X. Nukleáris Szaktábor az MNT jelentõstámogatásával

2016-ban a korábbi gödi helyszínrõl Keszthelyre vit-tük a tábort. Ezzel is igyekeztünk vonzóbbá tenni arendezvényt. A X. Nukleáris Szaktábor 2017. július

272 FIZIKAI SZEMLE 2018 / 7–8

Page 59: Fizikai Szemle 68/7–8 – 2018. július–augusztusgoliat.eik.bme.hu/~f.simon/publications/Media/FizSzem-20180708.pdf · okokból standard modellnek hívunk, a világot anyagi és

2-tól 7-ig tartott, az elõadások és a szállás az elõzõ évi

1. ábra. Az energetikában dolgozók létszáma, egy 2006-os felmérés eredményei alapján.2

MOL Magyar Olaj- és Gázipari Nyrt.

Paksi Atomerõmû Zrt.

Mátrai Erõmû Zrt.

Vértesi Erõmû Zrt.

Fõvárosi Gázmûvek Rt.

Siemens Erõmûtechnikai Kft.

Magyar Villamos Mûvek Zrt.

EL&ME Irányítástechnikai Kft.

EGI Energiagazdálkodási Rt.

ETV-ERÕTERV Zrt.

DC. Gépszer Kft.

Radioaktív Hulladékokat Kezelõ Kht.

Tatabánya Erõmû Kft.

Budapesti Kutatóreaktor Mûszerközpont

Országos Atomenergia Hivatal

Partner Betonelemgyártó és Fémipari Kft.

PROTECTA Elektronikai Kft.

Paksi Atomerõmû Zrt. Biztonsági Fõosztály

MIRROTRON Kft.

Konkoly és Kis Mérnöki Iroda Kft.

VEIKI Zrt. Atomerõmûvi Divízió

SOM System Mérnöki Iroda Kft.

ESS-Magyarország Kht.

1 10 102 103 104

munkaerõ összlétszámamunkaerõ összlétszáma

felsõfokú mûszaki végzettségûek létszámafelsõfokú mûszaki végzettségûek létszáma

2. ábra. Pokol Gergõ elõadás közben.

helyszínen, a Keszthelyi VSZK3 tantermeiben és kollé-

2 I. m. 19. old.3 Keszthely Város Vendéglátó, Idegenforgalmi, Kereskedelmi Szak-képzõ Iskolája és Kollégiuma

giumában voltak.A résztvevõk között évek óta többen jelen vannak

az Országos Szilárd Leó Fizikaverseny versenyzõiközül is. A helyezettek kedvezménnyel vehetnek részta táborozáson.

Július 2-án 30 diákkal indítottuk a tábort. A táborlegfõbb céljának – a diákok érdeklõdésének felkelté-se a nukleáris technika iránt – elérése érdekében töre-kedtünk az igényes, változatos program kialakítására,ugyanakkor nagy hangsúlyt fektettünk az elméleti ésgyakorlati oktatásra is. Délelõttök zömében tapasztaltszakemberek, egyetemi oktatók tartottak elõadásokat(2. ábra ). Közöttük a BME Nukleáris Technikai Inté-zetének oktatói (Boros Ildikó, Horváth András, KisDániel, Pesznyák Csilla, Pokol Gergõ ), az MTA Ener-

giatudományi Központjának kutatói (Fábián Margit,Hózer Zoltán ), a Paksi Atomerõmû Zrt. vezetõ szak-emberei (Cserháti András, Lencsés András ), az Eöt-vös Loránd Tudományegyetem (Horváth Ákos, Rad-nóti Katalin ) és a Debreceni Egyetem (TrócsányiZoltán ) tanárai szerepeltek. A mérési, kísérleti, játé-kos programokat a magfizika oktatásában vezetõ sze-repet játszó középiskolai tanárok (Farkas László, Mes-ter András, Oláh Éva, Pántyáné Kuzder Mária, Ujvá-ri Sándor ) vezették.

Szakmai elõadások

A program összeállítását Mester András koordinálta.Összesen 17 elõadás hangzott el. A bevezetõt köve-tõen – többek között – hallhattak– az atomerõmûvekrõl, a fûtõelemekrõl,– a fúzióról,– a hulladéktárolásról,– a sugárvédelemrõl,– a nukleáris eseményskáláról,– az orvosi alkalmazásokról,– a sötét anyagról,– a részecskefizikáról,– az MNT infografikáiról,– a radonról,– a Szilárd Leó fizikaversenyrõl és– a Paksi Atomerõmû mérõkocsijáról.

A több témát érintõ elõadásokkal fontosnak tartot-tuk, hogy a magfizikát ne csak egy-egy szakterülettel

A FIZIKA TANÍTÁSA 273

Page 60: Fizikai Szemle 68/7–8 – 2018. július–augusztusgoliat.eik.bme.hu/~f.simon/publications/Media/FizSzem-20180708.pdf · okokból standard modellnek hívunk, a világot anyagi és

azonosítsák a táborozók. Az MNT ifjúsági szervezeté-

3. ábra. A X. Nukleáris Szaktábor résztvevõi és szervezõi (ebben az évben lila egyentrikóban).

X. Nukleáris SzaktáborX. Nukleáris Szaktábor

Magyar Nukleáris Társaság

4. ábra. Mérési gyakorlat a Vajda János Gimnáziumban Farkas Lász-ló tanár úr vezetésével.

vel a FINE-vel (Fiatalok a Nukleáris Energetikáért)Sárdy Gábor ismertette meg a táborozókat.

Mérések, laboratóriumi gyakorlatoka Vajda János Gimnáziumban

2017-ben hangsúlyt kívántunk helyezni a modernfizika elemeinek gyakorlatban való megismerésére. Alegtöbb iskolában kevés lehetõség van az atomfiziká-val kapcsolatos mérések elvégzésére, kísérletek meg-tekintésére. Nem kis munkával sikerült egy teljes dél-utánnyi programot összeállítani (4. ábra ). A mérések-hez, kísérleti bemutatókhoz az eszközök a keszthelyiVajda János Gimnáziumból, a miskolci Herman OttóGimnáziumból, a paksi Energetikai Szakgimnázium ésKollégiumból, a székesfehérvári Lánczos Kornél Gim-náziumból és a miskolci Diósgyõri Gimnáziumbólkerültek ki.

A laboratóriumi program része volt:– a Planck-állandó meghatározása– a béta-sugárzás eltérülésének mérése– a radon felezési idejének meghatározása– a Franck–Hertz-kísérlet elemzése– színképek elemzése spektroszkóppal– az elektron hullámhosszának meghatározása diff-

rakciós csõvel és– tömegspektroszkópai kísérlet elemzése.

Egyéb szakmai programok

– Kísérleti bemutatót tartott Ujvári Sándor.– Játékos részecskefizika címmel Oláh Éva vezetett

esti programot a táborozók számára– A Paksi Atomerõmû mérõkocsiját Lencsés András

mutatta be a táborozóknak.– Egy magfizika-feladatlapot – amelynek megoldását

a tábor végén értékeltük – állítottunk össze a diá-

kok számára. A táborozók többsége igen komolyandolgozott a feladatokon. Meglepõen szép eredmé-nyek születtek.

Kiegészítõ programok

Keszthely megismerése céljából az elsõ nap FarkasLászló kolléga, a Vajda János Gimnázium matematika-fizika tanára rövid sétával egybekötött idegenvezetésttartott a városban.

A második nap délutánján a Balatoni Múzeumottekintették meg a diákok (5. ábra ).

A tábor résztvevõi meglátogatták a Hévízi Gyógy-fürdõt, ahol egy ismeretterjesztõ elõadás után fürdés-re is volt lehetõség.

Szabadidõs programok

Szerencsénk volt az idõjárással. A kezdeti hûvösebbidõt követõen minden lehetõ alkalommal strandlátoga-

274 FIZIKAI SZEMLE 2018 / 7–8

Page 61: Fizikai Szemle 68/7–8 – 2018. július–augusztusgoliat.eik.bme.hu/~f.simon/publications/Media/FizSzem-20180708.pdf · okokból standard modellnek hívunk, a világot anyagi és

tást szerveztünk, így – a hévízi lehetõséget is beszámít-

5. ábra. Csoportkép a Balatoni Múzeum elõtt.

va – majdnem minden nap tudtak fürödni a táborozók.A kollégium sportpályáján és a tornateremben le-

hetõség adódott focizásra is, amit a diákok természe-tesen ki is használtak.

Az utolsó esti ünnepélyes keretek között megtartottzáróvacsorán a táborozóknak alkalmuk volt a táborralkapcsolatos tapasztalataik elmondására. A vacsoránvendégként részt vett Ördögh Miklós, az MNT elnöke,Pántyáné Kuzder Mária, az ELFT alelnöke, valamintKis Dániel, a BME NTI docense.

✧A tábor teljes idõtartama alatt a diákokkal volt MesterAndrás, Ujvári Sándor és Horváth András, akik gon-doskodtak a felügyeletrõl, illetve a programok lebo-nyolításáról.

Hatalmas munkát végzett és köszönet illeti FarkasLászlót – egyik vendéglátónk, a Keszthelyi Vajda Já-

nos Gimnázium tanárát –, aki az elõkészületi mun-kákban és a helyi szakmai-kulturális programok szer-vezésében is aktívan részt vett.

A táborozók véleménye, összegzés

Minden évben megkérjük a résztvevõket, hogy érté-keljék a tábort. Volt néhány módosításra utaló javaslat(például: legyen több szünet az elõadások között),amelyeket figyelembe veszünk a következõ tábor szer-vezésekor. Elégedettek voltak a helyszínnel, a szakmaiszínvonallal, hasznosnak találták az elõadásokat követõbeszélgetések lehetõségét. Arra a kérdésre, hogy „Ösz-szességében elégedett vagy-e táborral?” igen pozitívválaszokat kaptunk. Ez azért is figyelemre méltó, mertebben az évben több nyolcadik, illetve kilencedik osz-tályt végzett, úgynevezett kezdõ is volt a táborban.

Infografika a 10 éves évfordulóról

A tizedik évforduló kapcsán infografika készült a tá-borról, amely összefoglalja az eltelt tíz év eseményeit.Az infografika elérhetõ az MNT honlapján (http://nuklearis.hu/sites/default/files/Nuktabor_0.jpg).

XI. Nukleáris Szaktábor

A XI. Nukleáris Szaktábort 2018. július 1-tõl 6-ig tar-tottuk az elõzõ évek helyszínén Keszthelyen. Nagyörömünkre a résztvevõk száma meghaladta a 30 fõt. Atáborral kapcsolatos információk az MNT honlapjánérhetõk el: http://nuklearis.hu/nuklearis-szaktabor-0.

RÁTZ TANÁR ÚR ÉLETMÛDÍJ, 2017Mester András tanár úrral Radnóti Katalin beszélget

A 2017-ben kiadott Rátz Tanár Úr Életmûdíj egyikdíjazottja Mester András, amelyhez gratulálok!

Mester András 1978-ban végzett a Kossuth Lajos Tu-dományegyetemen matematika-fizika szakos középis-kolai tanárként. Több középiskolában tanított Miskol-con, legtovább a Diósgyõri Gimnáziumban, innen isment nyugdíjba 2015-ben. Azóta óraadó tanárként dol-gozik az Európa Baptista Gimnázium, Szakgimnáziumés Szakközépiskolában. A tanítás mellett évekig voltszaktanácsadó. Jelenleg is több társadalmi szervezetaktív tagja, az Eötvös Loránd Fizikai Társulat megyeicsoportjának titkára volt, majd fõtitkárhelyettese, ké-sõbb alelnöke, a Társulat Középiskolai Oktatási Szak-csoportjának elnöke. E mellett részt vesz a MagyarNukleáris Társaság (MNT) elnökségének munkájában,ahol jelenleg a Tanári Tagozat elnöke.

Nõs, felesége Mesterné Csordás Judit középiskolaitanár. Két gyermekük született, akik eddig háromfiúunokával ajándékozták meg.

Kitüntetései, elismerései: Ericsson-díj, 2003; SzilárdLeó-díj, 2013; Pedagógus szolgálati emlékérem, 2016;Rátz Tanár Úr Életmûdíj 2017.

Az alábbi interjúban felidézzük a tanár úr gazdagéletpályájának néhány érdekes és fontos állomását.

– Szerinted milyen a jó fizikatanár?– Az a jó tanár, aki érdekes órákat tart. Legyenek

olyan érdekesek az órái, hogy magával tudja ragadnia gyerekeket. Ehhez fontos, hogy maga a tanár is sze-resse a szaktárgyát, és ezt lássák a gyerekek.

– Milyen tantárgy a fizika?– Az a meglátásom, hogy sok gyerek félreérti a

tantárgy jellegét. Azt gondolja, hogy a fizika a mate-

A FIZIKA TANÍTÁSA 275

Page 62: Fizikai Szemle 68/7–8 – 2018. július–augusztusgoliat.eik.bme.hu/~f.simon/publications/Media/FizSzem-20180708.pdf · okokból standard modellnek hívunk, a világot anyagi és

matika egyik „torzszüleménye”. Pedig nem errõl szól,

eln

yelt

zis

(gra

y/h

ón

ap)

0,8

0,6

0,4

0,2

0

Radithor-fogyasztásleállítása

1928. jan. 1929. jan. 1930. jan. 1931. jan. 1932. jan.

hanem arról, hogy ismerjük meg a természet törvé-nyeit. Azon senki nem csodálkozik, hogy a tyúk tojásttojik, de a gravitáción már igen. Pedig a természetarra is alkotott egy törvényt, meg erre is. És az lenne afizikaoktatás célja, hogy ezen fontos és alapvetõ ter-mészeti törvényeket ismerjük meg, de úgy látom, azoktatás során ez elvész valahogy.

– Mikor döntötted el, hogy fizikatanár szeretnéllenni?

– Volt egy nagyon jó, lelkes fiatal fizikatanárom.Másrészt mindig szerettem dolgokat építgetni, össze-rakni, ezért vonzódtam a fizikához.

– Van-e kedvenc témaköröd?– Több is van, de a nukleáris vonalon eléggé kép-

viseltetem magam. 1994-ben voltam tanulmányútona CERN-ben, amelyet Marx György professzor úrszervezett. Alapításától fogva tagja vagyok a SzilárdLeó Verseny zsûrijének. A felkészülést segítendõtöbb példatár szerkesztésében is részt vettem, amelya verseny feladatait és részletes megoldásait mutatjabe a diákok és az õket felkészítõ lelkes tanárok szá-mára. Tehát a fizika számomra legkedvesebb fejeze-te elsõsorban a modern fizika. Továbbá ez a fizikaazon fejezete, ahol szembesülünk, hogy modellt kellalkotni a jelenségek leírásához, a mikrovilág törvé-nyeinek megismeréséhez, azok matematikai megfo-galmazásához, és e modellekkel próbáljunk magya-rázatokat adni a jelenségekre. A mikrovilág leírásá-hoz hullám-részecske kettõsség használandó, eztmeg kell érteni. Itt jönnek rá a gyerekek, hogy iga-zán mi is a fizika.

Fontosnak tartom a diákok versenyeztetését. Igyek-szem minden évben érdekes versenyfeladatokat ki-tûzni a fiatalok számára, amelyek közül kettõt szeret-nék bemutatni.

Az elsõ a 2014. évi Szilárd Leó Fizikaverseny döntõ-jén kitûzött feladatom.1

1 A Scientific American folyóirat 1993. augusztusi számában meg-jelent cikk alapján. A feladat megoldását lásd: Fizikai Szemle 64/10(2014) 364. oldal.

„A nagy rádium botrány” jelzõvel illették azt azesetet, amikor 1932. március 31-én Eben M. Byerstöbbszörös milliomos, egykori golfbajnok testsúlyá-nak jelentõs részét elveszítve, drámai körülményekközött meghalt. Byers – egy sérülést követõen – ro-boráló („erõsítõ”) gyógyszerként Radithort fogyasz-tott. Egy Radithort tartalmazó fél unciás (1 uncia =28,25 gramm) üvegcse desztillált vízben 226Ra és228Ra izotópokat tartalmazott. Az izotópok aktivitásanagyjából azonos, egyenként körülbelül 1-1 μCi(~37 kBq) volt.

a) Mennyi volt az egyes izotópok tömege egy üveg-csében?

b) Mennyi volt az egyes izotópok által egységnyiidõ alatt leadott energiák aránya?

c) Az ábra Byers csontjaiban havonta elnyelt dózisbecsült értékét mutatja. Mire lehet következtetni azábrából?

Adatok: a 226Ra α-sugárzó, Eα = 4,871 MeV, felezésiideje 1600 év, a 228Ra β−-sugárzó, átlagos β-energia Eβ= 7,2 keV, felezési ideje 5,7 év.

2002-tõl 2015-ig szerveztem iskolámban Miskolcon, aDiósgyõri Gimnáziumban a Városi Fizikavetélkedõt. Amásodik példát egy a 2009. évi feladataim közül vá-lasztottam.

Egy vízszintes asztallapra két rugó közé, az ábra sze-rinti elrendezésben egy 0,5 kg tömegû testet helyezünkel. Kezdetben a rugók feszítetlen állapotban vannak.

D1 D2

A

x

Az asztal és a test között a tapadási súrlódási együtt-ható μ0 = 0,3, a rugóállandók megegyeznek: D1 = D2 =50 N/m.

a) Mekkora lesz a két rugóból álló rendszer rugóál-landója? Miért?

b) Mekkora A távolságig mozdíthatjuk a testet,hogy a rugók még ne rántsák vissza?

c) Mekkora a mozgási súrlódási együttható, ha az Ahelyrõl elindult test az egyensúlyi helyzeten való átha-ladás után, a kezdeti x kitérés háromnegyedéig jut?

– Hogyan készültél az óráidra kezdõ pedagógus-ként és hogyan manapság?

– Mindig készülök az óráimra, most is. Minden gye-rekcsoport más és más. Hiába vannak elõre összeállí-tott sémáim, az nem alkalmazható minden osztályra.Minden osztály karakterének megfelelõen kell rájuk,az õ órájukra készülni.

– Megjelennek-e új eszközök a tanítási gyakorla-todban?

– A fizika tanításához tartozó klasszikus didaktikaieszköz a kísérlet. Ellenben a mindennapi életbenmegjelent a számítógép. Én már évek óta digitálistáblán tanítok. Ez sok dologra ad lehetõséget. Pél-

276 FIZIKAI SZEMLE 2018 / 7–8

Page 63: Fizikai Szemle 68/7–8 – 2018. július–augusztusgoliat.eik.bme.hu/~f.simon/publications/Media/FizSzem-20180708.pdf · okokból standard modellnek hívunk, a világot anyagi és

dául rendszeres probléma, hogy a diákok a feladatok

A díjátadóról készült fényképen Mester András (jobbra), a másik díjazott Piláth Károly (balra).Középen a díjat átadó Éry Gábor, az Ericsson Magyarországi igazgatója.

megoldása során nem tudják numerikusan kiszámíta-ni az eredményeket. Még akkor sem, ha esetleg adiák tudja az elméletet – a számítás problémát okoz.Ekkor kiteszem a digitális táblán az „asztalra” a szá-mológépet, kinagyítjuk, és azon végezzük el a számí-tást az osztály számára teljes mértékben követhetõmódon. De emellett – természetesen – sok animáció,szimulációs program, videó is helyet kap a tanórái-mon. A diákok számára sokat jelent az is, ha a tudó-sok képeit is látják, nem csak a róla elnevezett ésmegtanulandó törvényt.

A feladat megoldásánál a feladat szövegét – amely-hez kép, ábra, esetleg videórészlet tartozik – kivetí-tem. A feladat megoldása mindig ott helyben, a gyere-kekkel közösen születik meg. A digitális táblán létre-jött dokumentumot, valamint az elõre elkészített pre-zentációkat is megosztom a diákokkal. Így könnyûkorábbi anyagokra hivatkozni, visszakeresni azokat.A diákok számára fontos segítséget jelentek ezek aszakanyagok a tanulásban, például a dolgozatokravaló felkészülésben.

Az évek során kialakítottam a dolgozatok szerkeze-tét, amelyekben szerepel 8-10 tesztkérdés és 3-4 fel-adat. A tesztek általában könnyebbek, az elméletianyagot tartalmazzák. Ha valaki ezeket jól megoldja,már elérte a kettes szintet. A jobb jegyhez a feladatokmegoldásából is kell teljesíteni.

Szeretem alkalmazni az Excel programot, amikor adiákok számára állítok össze hasonló számításokatigénylõ feladatokat. Például elektromosságtan téma-körében a soros és a párhuzamos kapcsolások eseté-ben mekkora az eredõ ellenállás, mekkora feszültségesik az egyes áramköri elemeken, mekkora az áram-erõsség stb. Erre írtam egy programot, amelyben meglehet adni a bementeti adatokat és az Excel gyorsankiszámítja a többi hiányzó adatot. Így egyszerû több,különbözõ, de azonos számítási metódust alkalmazó

feladatot elõállítani. Így 8-10csoportot is létre tudok hozniegy osztályban.

A csúszási és tapadási súr-lódással kapcsolatos jelenség-hez tartozó számítások is egy-szerûen végezhetõk. Példáulkülönbözõ erõk hatására atest gyorsul, nem gyorsul stb.Néha beviszem a diákokat azinformatikaterembe, hogy akitud, írjon programot külön-bözõ jelenségekre.

– A Nukleáris Tábor szerve-zése mit jelent számodra?

– 2007-ben a Magyar Tudo-mányos Akadémia Nagyter-mében volt egy Aszódi Attilaprofesszor úr vezette találko-zó, ahol az atomerõmûvetgyártó cégek mutatkoztak be.Arról viszont nem esett szó,

hogy kik fognak dolgozni a majdani létesítményben.A tervezõk rajzokon mutatták be, hogy milyen leszegy nagy erõmû és néhány helyre pici embereket israjzoltak. De azzal senki nem foglalkozott, hogy kikazok a pici emberek? Ekkor merült fel az ötlet: jólenne olyan tábort szervezni, amely elõsegítené anukleáris témák iránt érdeklõdõ gyerekek felkarolá-sát, és ez egyben fontos utánpótlásbázist is jelenthet-ne a nukleáris szakma számára. Az ötlet valóra vált ésidén a tábort már 11. alkalommal rendeztük meg aközépiskolás diákok számára a Magyar Nukleáris Tár-saság szervezésében. Az egy hetes, bentlakásos tábor-ba mindig a nukleáris szakma képviselõi jönnek elõ-adást, különbözõ interaktív foglalkozást tartani a lel-kes fiatalok számára.

Táboronként korábban átlagosan 23 fiatallal lehe-tett számolni – tavaly már 30-an voltak, idén még aztis meghaladták – 16 elõadással és 3-4 látogatással,mint Paksi Atomerõmû Rt., ELTE TTK Magfizikai labo-ratóriumai, BME Tanreaktor, MTA EK laboratóriumai.A leggyakoribb elõadási témák: nukleáris alapfogal-mak, atomerõmûvek, hulladéktárolás, sugárvédelem,nukleáris medicina, képalkotó eljárások, a sugárzás amindennapokban, nukleáris történelem. De azért nemmarad ki a tábor programjából a szórakozás, sport és– nyár lévén – a fürdõzés.

– Hogyan éled meg a fizika tantárgy fokozatosvisszaszorulását?

– Nehezen. De ennek az is oka lehet, hogy annyitváltozott a mindennapi világunk, hogy az átlagembermár nehezen érti meg a körülöttünk lévõ egyre bo-nyolultabb eszközök mûködését. Nagy lett a szaka-dék a szakmai ismeretekkel bíró szûk kör és az átlag-felhasználók között.

– Milyen útravalót adnál általánosságban a fiata-loknak, és konkrétan a fizikatanári hivatást válasz-tóknak?

– Szeressék a fizikát és szeressék a gyerekeket!

A FIZIKA TANÍTÁSA 277

Page 64: Fizikai Szemle 68/7–8 – 2018. július–augusztusgoliat.eik.bme.hu/~f.simon/publications/Media/FizSzem-20180708.pdf · okokból standard modellnek hívunk, a világot anyagi és

NAGYFREKVENCIÁS JELEK KÁBELBENI TERJEDÉSÉNEKFIZIKAI ALAPJAI – 1. rész

A szerzõ köszönetet mond a cikksorozat alapjául szolgáló egyetemilaboratóriumi gyakorlat leiratának elkészítésében közremûködõGyüre-Garami Balázsnak, Márkus Bencének, Fülöp Ferencnek ésHalbritter Andrásnak.

Simon Ferenc fizikus, egyetemi tanár (BME),az MTA doktora. Érdeklõdési területei: kísér-leti szilárdtest spektroszkópia, a spintronikaelméleti és kísérleti aspektusai, a fizika nép-szerûsítése. Legfontosabb eredményei: ESR-jel felfedezése új fémekben (MgB2, bórraldópolt gyémánt, alkálival dópolt grafén);spinrelaxáció egyesített elméletének kidol-gozása; elektron- és magspinnel nyomjelzettszén nanocsövek elõállítása. ERC- és Lendü-let-pályázat vezetõje. 8 TDK, 31 BSc/MSc és3 PhD témavezetõje.

Simon FerencBudapesti Muszaki és Gazdaságtudományi Egyetem, Fizika Tanszék

és MTA–BME PROSPIN Lendület Kutatócsoport

Az egyetemi fizikaoktatásban azt találjuk, hogy a rá-diófrekvenciás jelek terjedésének leírása sötét foltotképez. Már középiskolában is megfelelõen oktatjukaz RLC-áramköröket, egyetemen pedig az optikaiterjedési és törési törvényeket, azonban az 1 MHz –10 GHz-es frekvenciatartományba esõ jelekre úgytekintünk, mintha ez a mérnöki tudományok „felség-területe” lenne. Pedig a modern méréstechnika ésmérési módszerek (például nanoelektronika, mágne-ses rezonancia) bemutatásához és megértéséhez ezenterület magabiztos ismerete szükséges. Fizikus szem-mel nézve a témakör nem más, mint a Maxwell-egyenletek alkalmazása egy speciális esetre, azaz ve-zetékekre, kábelekre, adott esetben úgynevezett hul-lámvezetõkre. Amennyiben az olvasót valaha is izgat-ta, miért van minden nagyfrekvenciás mûszerre a „bû-vös 50 Ω” írva, mit jelent a kábelvégi reflexió és azimpedanciaillesztés, akkor kérdéseire a jelen és az eztkövetõ cikkünkben választ kap.

A vizsgálódásunk egyik legfontosabb üzenete,hogy az alacsony frekvenciás hálózatok vizsgálatakormegszokott leírásmód nagyobb frekvenciákon érvé-nyét veszti, és a hagyományos áramköri jelenségekentúlmutató, szokatlan jelenségek lépnek fel, mint pél-dául a jelek visszaverõdése (reflexiója). Ahhoz, hogyaz elsõdlegesen adódó kérdést, „Mi is az a nagyfrek-vencia?” megválaszoljuk, vizsgáljuk meg a terület ki-alakulásának történetét!

A 19. század közepén felmerült az igény a nagy,akár kontinensnyi távolságban történõ adattovábbí-tásra, például tenger alatti kábelek segítségével (azelsõ transzatlanti kábelt 1858-ban helyezték üzembe).Hamar kiderült, hogy a vezetékben történõ jeltováb-bításnál lényeges a hullámjelenségek figyelembevéte-le. Ez a technológiai fejlõdés és az igény az elméletileírásra idõben közel volt a Maxwell-egyenletek

(1861) megszületéséhez. A vezetékben terjedõ hul-lámjelenségek leírását ma mint az úgynevezett távíró-egyenleteket (angolul telegrapher’s equations ) ismer-jük. Ez lényegében a Maxwell-egyenletek által megjó-solt elektromágneses hullámjelenségek egyik elsõgyakorlati alkalmazása. A távíróegyenletek gyakorlatisikere inspirálóan hatott az elektromágneses sugárzáskésõbbi felfedezésére (Hertz, 1886).

A fizikustanulmányok során legtöbbször felmerülõegyenáramú (DC) és alacsony frekvenciás váltóáramú(AC) hálózatok vizsgálatakor nem törõdünk a jel vé-ges terjedési sebességével. Feltételezzük, hogy adottponton feszültséget kapcsolva egy áramkörre, annakhatása pillanatszerûen megjelenik az áramkör egé-szén. Gondoljunk csak egy RLC-áramkör jól ismertanalízisére! Mindez nyilvánvalóan érvényét veszíti,amikor a jel számára szükséges terjedési idõ, t = d /c(itt d a kábel hossza, c a közegben érvényes fényse-besség) összemérhetõ a jel periódusidejével: t ≈ 1/f(a gyakorlatban inkább a 10 t ≈ 1/f feltétel a használa-tos). Például a transzatlanti kábel esetére az így ka-pott frekvencia f = 6 Hz. Ez az eredmény azt jelenti,hogy a hullámjelenségek figyelembe vétele nélkül atranszatlanti kommunikáció csak ennél lényegesenalacsonyabb frekvencián, mai szóhasználattal élve, 6Hz sávszélességen (azaz 6 bit/s) mehetne csak végbe.Ezt az igen alacsony adatátviteli sebességet az elsõkábel lefektetése után a gyakorlatban is tapasztalták,ami elvezetett a távíróegyenletek leírásához.

A hullámjelenségek figyelembevétele a modernkommunikációs eszközöknél még fontosabb, mivelpéldául 10 GHz-es vivõfrekvencián (ami egy elterjedtmûholdas kommunikációs sáv) a hullámhossz mind-össze 3 cm. Másik gyakorlati példánk a számítógépek,amelyek tipikusan 2-3 GHz-es jelekkel dolgoznak (λ ≈10 cm). Ezeket a jeleket 10-20 cm távolságra juttatják el,így itt az áramkörök tervezésekor a hullámjelenségekfigyelembevétele nyilvánvalóan szükséges.

A távíróegyenletek

Tekintsük a jelet továbbító vezeték egy infinitezimáli-san kicsi darabját, amelynek áramköri modellje az 1.ábrán látható. A helyettesítõ képben minden fizikaimennyiség hosszegységre van normálva: legáltaláno-sabban egy soros, úgynevezett elosztott ellenállás, R(egysége ohm per méter), elosztott induktivitás, L(egysége henry per méter), elosztott kapacitás, (egy-Csége farád per méter), és a két vezeték közti elosztottvezetés, (egysége siemens per méter) jellemzi.G

278 FIZIKAI SZEMLE 2018 / 7–8

Page 65: Fizikai Szemle 68/7–8 – 2018. július–augusztusgoliat.eik.bme.hu/~f.simon/publications/Media/FizSzem-20180708.pdf · okokból standard modellnek hívunk, a világot anyagi és

A soros ellenállás oka a vezetékdarabokban lévõ

1. ábra. A jelterjedésben vizsgált vezeték egy darabjának áramkörimodellje.

R~

L~

G~

C~

2. ábra. A koaxiális vezeték keresztmetszete az elektromos és mág-neses tér E, illetve B vonalaival a kábel alapvetõ, úgynevezettTEM00 módusára. A belsõ vezetéken változó feszültség van, míg akülsõ leggyakrabban le van földelve.

E

B

veszteség, az induktivitás oka pedig az, hogy a kétvezeték között mágneses teret találunk, amihez önin-dukció is tartozik. A írja le a két vezetékdarab köztiGelektromos vezetést, szivárgást. A kapacitás oka pedigaz, hogy a két vezeték nincs azonos potenciálon ésköztük valamilyen dielektrikum helyezkedik el. Ideá-lis esetben és értéke 0.R G

A fenti paraméterek közül értéke elsõsorban aRvezetõ anyagi minõségétõl függ (értéke nagy frek-vencián a szkineffektus miatt megnõ), azonban ,L Cés értéke nagyban függ attól, hogy a két vezetékGegymáshoz képest hogyan helyezkedik el (példáulsodort érpárra ≈ 0, de értéke nagy). EgymástólL Cadott távolságra elhelyezkedõ drótpár esetére ér-Ltéke fix, viszont nagyban függ a környezõ dielekt-Crikumtól (utóbbi probléma a sós víz miatt a transzat-lanti kábelnél merült fel).

Mindezen problémákra kínál megoldást a koaxiá-lis kábel (Heaviside, 1880), amiben a földelt külsõvezetéken belül helyezkedik el a másik vezeték.Ennek elõnye, hogy minden paramétere jól definiáltés a környezetre érzéketlen, mivel mind az elektro-mos, mind a mágneses erõvonalak a koaxiális kábelkét vezetéke között vannak, amint a 2. ábra mutatja.

A korábbi, merev falú, levegõvel kitöltött koaxiáliskábeleket mára a rugalmas dielektrikummal kitöltöttkábelek váltották fel, amelyekre tipikusan εr = 2–3és μr = 1,0 értékû.

A koaxiális kábelek hosszegységre esõ kapacitásá-ra és önindukciós együtthatójára e két paraméter defi-níciójából adódik:

és

(1)C =2 π ε0 εr

ln(D/d )

ahol D az árnyékoló vezeték belsõ átmérõje és d a

(2)L =μ0 μ r ln(D/d )

2 π,

kábel belsõ vezetõjének külsõ átmérõje, ε0 és μ0 azismert fizika állandók, εr és μr az anyagra jellemzõrelatív dielektromos állandó és relatív permeabilitás.

A távíróegyenletek bemutatásához, a legegysze-rûbb eset tárgyalásához feltesszük, hogy mindkétvezeték tökéletes vezetõ ( = 0) és tökéletesen szige-Rtelt egymástól ( = 0), tehát a jelterjedés csak ésG L

-tõl függ. Ekkor mind az U (x, t ) feszültség, mind azCI (x, t ) áram hely és idõfüggõ, és leírásukra – az elemivezetékszakaszokra alkalmazott Kirchhoff csomópon-ti és huroktörvénye miatt – a következõ két csatolt,lineáris, elsõrendû parciális differenciálegyenlet adó-dik (Heaviside, 1880):

A (3) távíróegyenletek két ekvivalens, egydimenziós

(3)

∂U (x, t )∂x

= −L ∂I (x, t )∂t

,

∂I (x, t )∂x

= −C ∂U (x, t )∂t

.

hullámegyenletté vonhatók össze mind az áramra,mind a feszültségre:

Az egydimenziós hullámegyenlet általános alakjával,

(4)

∂2U (x, t )

∂t 2= 1

L C

∂2U (x, t )

∂x 2,

∂2I (x, t )

∂t 2= 1

L C

∂2I (x, t )

∂x 2.

(ahol Ψ a hullámfüggvény és v ennek terjedési sebes-

(5)∂2Ψ (x, t )

∂t 2= v 2 ∂2Ψ (x, t )

∂x 2

sége) összehasonlítva látható, hogy a kábelben terje-dõ zavar sebessége,

A legáltalánosabb megoldás a vezetékben terjedõ

v = 1

L C

.

feszültségre és áramra:

A FIZIKA TANÍTÁSA 279

Page 66: Fizikai Szemle 68/7–8 – 2018. július–augusztusgoliat.eik.bme.hu/~f.simon/publications/Media/FizSzem-20180708.pdf · okokból standard modellnek hívunk, a világot anyagi és

ahol ω a terjedõ hullám körfrekvenciája, k = ω /v

3. ábra. Sematikus áramkör szinuszos meghajtó generátorral,amelynek Zf kimenõ ellenállása van és koaxiális vezetékkel, amely-nek végén Zl lezáró impedancia van. A generátorból akkor jön ki ateljes teljesítmény, ha Zf = Z0. A lezárás három értékére vonatkozóvezetékbeli feszültség értékét is mutatjuk 20 pillanatfelvételre, ami-kor d = 5λ. Vegyük észre, hogy Zl = Z0 esetén a feszültség burkoló-függvénye homogén a vezetékben, Zl = 0 esetén mindkét végéncsomópont van (ekkor forráson is 0 feszültséget mérünk), és Zl = ∞esetén mindkét végén duzzadóhely van.

Z Z1 0=

Z1 = 0

Z1 = �

Uf U1

Zf Z1

hullámimpedancia = Z0

(6)U (x, t ) = U f (ω t − k x ) U − f (ω t k x ),

I (x, t ) = I f (ω t − k x ) I − f (ω t k x ),

pedig a hullámszáma. U + és U − a pozitív, illetve nega-tív x irányba terjedõ jel amplitúdója, f egy tetszõlegesfüggvény. Vegyük észre a (2) egyenlet alapján, hogy akábelbeni terjedési sebesség kifejezhetõ a c vákuum-beli fénysebességgel:

v = 1

ε0 μ0 εr μ r

= c

εr μ r

.

A hullámimpedancia

Egy speciális eset az, amikor a kábelen csak egyirányba haladó harmonikus hullám van jelen, megol-dása:

A (7) speciális megoldást a (3) távíróegyenletekbe

(7)U (x, t ) = U0 e i (ω t − k x ),

I (x, t ) = I0 e i (ω t − k x ).

visszaírva kapjuk, hogy a feszültség és áram aránya ahaladó hullám esetén:

(8)U (x, t )I (x, t )

= L

C=

μ0 μ r

ε0 εr

ln(D/d )2 π

= Z0,

ahol a Z0 (ellenállás dimenziójú) mennyiséget a kábelhullámimpendaciájának nevezzük. A távíróegyenle-tek és megoldásuk legfontosabb tanulsága, hogy min-den kábelhez rendelhetõ egy Ω dimenziójú hullámim-pedancia, még ha ideális esetben a kábelnek nincs isvalós ellenállása. A hullámimpedancia fontos szerepetjátszik a kábelen haladó jelek visszaverõdésének vizs-gálatakor, amit a továbbiakban ismertetünk.

A hullámimpedancia fizikai jelentésének bemutatá-sához célszerû a távíróegyenletek és a szabad térbenterjedõ elektromágneses sugárzás egyenletei közöttikapcsolatot bemutatni. Ismert, hogy az elektromágne-ses sugárzásra fennáll E = cB, ahol E és B az elektro-mostérerõsség- és mágnesesindukció-vektorok nagy-ságát jelölik egy adott helyen. Ezt átírva a H mágnesestérerõsségre adódik:

EH

=μ0

ε0

≈ 377 Ω.

Elsõ hallásra meglepõ és kevésbé ismert, hogy azelektromágneses sugárzáshoz is rendelhetõ egy el-lenállás dimenziójú mennyiség, amelyet a vákuumhullámimpedanciájának nevezünk. Ez a mennyiségfelfogható úgy is, mint az ellenállás alapvetõ egysége

a természetben, amelyhez képest minden ellenállás-értéket – mértékegység-választástól függetlenül –kifejezhetnénk.

A (8) egyenletben megadott Z0 hullámimpedanciaértékében láthatjuk, hogy annak a vákuum hullámim-pedanciájától való eltérése az alkalmazott dielektri-kum jelenlétébõl és a kábel geometriájából adódik.Cikkünk következõ részében megmutatjuk, hogy akábel hullámimpedanciájának kitüntetett szerepe vanabban az értelemben, hogy amennyiben egy jel terjeda kábelben, csak akkor nem verõdik vissza a kábelvégérõl, amennyiben ott is egy úgynevezett lezáróellenállás van, amelyre Zl = Z0. Egyéb esetben kábel-végi visszaverõdés történik, hasonlóan, ha a kábelmentén bárhol sérülés éri a vezetéket, és ezért hul-lámimpedanciája megváltozik, visszaverõdést talá-lunk. Ezt a hatást a hosszú kábelrendszerek sérülései-nek felderítésére lehet használni, illetve kémfilmek-ben így veszik észre, ha valaki a kábelre csatlakozvalehallgatja az adást.

Elõször két szélsõ esetet mutatunk be: amikor akábel végén szakadás (Zl = ∞) vagy rövidzár (Zl = 0)van. Mindkét esetben a jel teljes visszaverõdést szen-ved a kábel végérõl, azonban a kábel végén lévõ lezá-ró ellenállás értéke más peremfeltételt ír elõ, amit a 3.ábra mutat be. Az odamenõ és visszavert hullámok in-terferenciájának révén a kábel mentén duzzadóhelyekés csomópontok alakulnak ki, amint azt a 3. ábra mu-tatja. Vegyük észre, hogy az így kialakult állóhullám-kép emlékeztet a hangtanból jól ismert bezárt, illetvenyitott végû síp esetére, amikor is a hanghullámok

280 FIZIKAI SZEMLE 2018 / 7–8

Page 67: Fizikai Szemle 68/7–8 – 2018. július–augusztusgoliat.eik.bme.hu/~f.simon/publications/Media/FizSzem-20180708.pdf · okokból standard modellnek hívunk, a világot anyagi és

valósítják meg az állóhullámok jelenségét. Alább olyankísérleteket mutatunk be, amelyekkel az állóhullámokjelenléte kimutatható a diákok számára. A kábelbenkialakuló állóhullámkép mindenképpen szokatlan aDC-áramköröknél szerzett ismeretek alapján, amikor isazt várnánk, hogy a feszültség a rövidre zárt drótpár-ban végig 0, míg a szakadásos végû drótpárra végig ameghajtó generátor feszültségét veszi fel.

A bûvös 50 Ω és impedanciaillesztésa hétköznapokban

A hullámimpedancia értéke egy adott koaxiális kábel-re jól meghatározott, leggyakrabban 50 Ω-os (példáullabormûszereknél), 75 Ω-os (TV jelek kábeleinél) és100 Ω-os (UTP-s ethernet) kábelekkel találkozunk.Emellett régebben egyéb hullámimpedancia-értékek-kel is találkozhattunk, például 150 vagy 600 Ω. Utóbbikét értéknek történeti háttere van: a hagyományos,nem koaxiális távíróvonalaknak (két rézdrót egymás-tól 1 láb távolságra) körülbelül 600 Ω a hullámimpe-danciája. Az elsõ távolsági telefonok ezeket a vezeté-keket használták, ezért abban az idõben a 600 Ω volt atávközlési standard. Késõbb a csavart rézdrótpár ter-jedt el (felfedezõje A. G. Bell ), aminek 150 Ω a hul-lámimpedanciája.

A koaxiális kábelek elterjedésével felmerült a stan-dardizálás iránti igény. Azonban alapos kutakodásunkellenére sem sikerült a standardizálás történeti hátte-rét egyértelmûen feltárnunk. Több forrásból merítve1

1 http://www.highfrequencyelectronics.com/Archives/Jun07/HFE0607_Editorial.pdfhttp://www.rfcafe.com/references/electrical/history-of-50-ohms.htmhttp://www.microwaves101.com/encyclopedia/why50ohms.cfmhttp://en.wikipedia.org/wiki/Coaxial_cable

a következõ történetet találjuk.A koaxiális kábelek felfedezését követõen felmerült

az a kérdés, hogy a látszólag tetszõleges, és a kábelméreteivel befolyásolható hullámimpedanciának, anyilvánvaló 377 Ω értéken kívül van-e kitüntetett érté-ke. Kiderült, hogy van, de túl sok kitüntetett érték isvan attól függõen, hogy a kábelt mire szeretnénk hasz-nálni, például teljesítményátvitelre, vagy kis amplitú-dójú jelek mérése során történõ jelátvitelre. A koaxiáliskábelen átvihetõ maximális teljesítményre optimalizál-va Z0 = 30 Ω adódik. A nagyfeszültségû elektromosletörésben akkor kaphatjuk a legnagyobb értéket, haZ0 = 60 Ω. Az átvitt jel feszültségében a legkisebb vesz-teség pedig akkor lép fel ha Z0 = 77 Ω. A korai (1920-asévek) rádiófrekvenciás alkalmazásokban a forrásokteljesítménye alacsony volt, ezért a 77 Ω-os kábelekterjedtek el, hiszen a minél kisebb veszteség volt a cél.Mivel nem léteztek jó és rugalmas dielektrikumok,ezért a vezetékek két egymásba helyezett koaxiáliscsõbõl álltak, amelyek között levegõ volt.

Több alternatív elképzelés is létezik arra vonatko-zóan, hogy miért pont az 50 Ω-os koaxiális kábeleklettek a modern méréstechnika standardjai.

Közel 50 Ω hullámimpedanciájú vezetéket kapunk,ha egy 3/4 colos rézcsövet egy 2 colos rézcsõbe he-lyezünk (mindkettõ elterjedt méret az USA-ban) ésbelül levegõ található. Ez az egyszerû és olcsó megol-dás az 1930-as évektõl kezdve elterjedt a nagyteljesít-ményû rádiófrekvenciás adások kisugárzásánál. Egymásik elképzelés szerint, amikor egy levegõvel töltött77 Ω-os vezetéket polietilénnel töltünk fel (εr = 2,3),akkor impedanciája 51 Ω lesz, ami szintén az 50 Ω felémutat. A legvalószínûbb mégis az az elképzelés, hogyaz 50 Ω a veszteség és teljesítményátvitel közöttikompromisszum következménye. A két fent említettérték (30 és 77 Ω) számtani közepe 53,5 Ω, mértaniközepe 48 Ω, ebbõl adódik az 50 Ω. Ez azt jelenti,hogy ugyanazzal a kábellel tudunk nagyenergiájújelet kiadni és egy kis jelet kis veszteséggel venni.Tehát az 50 Ω-os hullámimpedancia használata nagy-ban egyszerûsítette a nagyfrekvenciás híradástechni-kát és az ilyen áramkörök kialakítását.

Végül egy érdekes javaslat az 50 Ω esetleges hátte-rére az, hogy egy koaxiális kábel, amire a belsõ veze-tõ és külsõ árnyékolás átmérõjének aránya ránézésre„szép” (hasonlóan mint az aranymetszés az építészet-ben), körülbelül 50 Ω-os hullámimpedanciával ren-delkezik.

Az 50 Ω elterjedtsége ellenére gyenge videójelekátvitelénél még találunk 75 Ω-os kábelt, ami a veszte-ségre optimális, illetve 93 Ω-os kábelt például számí-tógép és monitorok összekötésére. A 93 Ω-os hullám-impedanciájú kábelnél a hosszegységre esõ kapacitása legalacsonyabb, ezért nagyfrekvenciás levágási frek-venciája a legnagyobb. Emiatt a négyszögjelek is job-ban, torzításmentesen kerülnek átvitelre. Még különérdemes megvizsgálnunk, hogy miért 100 Ω-mal talál-kozunk az UTP kábeleknél: a probléma az, hogy egy-más mellett, kis méretben, több párhuzamos kábelen,nagy távolságra szeretnénk információt továbbítani.Ebben az esetben a helyhiány miatt nem praktikus atöbb párhuzamosan futó koaxiális kábel használata,azonban nagy távolságok esetén az egyes vezetékda-rabok áthallásával is kezdeni kell valamit. Ezért mint-egy újra felfedezték az analóg telefonoknál korábbanmár használt sodort érpárt, hiszen erre a kiszóródómágneses tér (és ezért az áthallás is) minimális. Erreutal az UTP elnevezése is: Unshilded Twisted Pair.Fentebb említettük, hogy a telefonok esetén 150 Ωvolt a használt hullámimpedancia, a modern UTP ve-zetékekben ezt tudatosan kisebb értékre szorítják le,azért, hogy értéke az optimális 77 Ω-hoz közelebblegyen.

A fizika más területén is találkozunk impedanciail-lesztéssel. Hanghullámok esetén például a fúvóshang-szerek tölcsére és a vonóshangszerek „teste” gondos-kodik arról, hogy a rezgõ síp vagy húr hatékonyangerjesszen hanghullámokat. Hasonlóan, a hangszó-rók, hangosbeszélõk kialakítása is hangot keltõ rezgõmembrán és a levegõ közötti energiaátadást szolgálja.Az orvosi célú ultrahangvizsgálatoknál alkalmazottzselé azt a célt szolgálja, hogy az ultrahangos készü-lék adóvevõje és az emberi test között ne legyen a

A FIZIKA TANÍTÁSA 281

Page 68: Fizikai Szemle 68/7–8 – 2018. július–augusztusgoliat.eik.bme.hu/~f.simon/publications/Media/FizSzem-20180708.pdf · okokból standard modellnek hívunk, a világot anyagi és

levegõ jelenléte miatti impedanciaillesztettlenség. Az

4. ábra. A szövegben leírt mérési feladatokhoz tartozó elrendezések. a) Annak bemutatása, hogy miért szükséges koaxiális kábelt használni,b) a kábelvégi reflexió bemutatására szolgáló elrendezés.

a)

b)

RF-generátor oszcilloszkóp

rövid BNC-kábel

hosszú BNC-kábel

BNC „T ”

közönséges szigetelt rézdrót

a végén

rövidzár:

vagy

szakadás, azaz semmi

vagy

50 -os lezárás:�

emberi középfül csontjai pedig a dobhártya és a fo-lyadékkal töltött belsõ fül között látnak el impedan-ciaillesztõ szerepet.

Optikában a közönséges üveg vagy szemüvegekfelületérõl fellépõ visszaverõdés közismert jelenség.Ezt úgynevezett antireflexiós rétegekkel meg lehetszüntetni, ami szintén egy példa az impedanciail-lesztésre. A visszaverõdés oka, hogy a levegõ-üveghatáron a törésmutató ugrásszerûen változik, a visz-szavert hullám r amplitúdóját a jól ismert Fresnel-formula írja le, ami merõleges beesésre a következõalakot ölti:

ahol n1 és n2 a két határos közeg törésmutatója. Nem

(9)r =n1 − n2

n1 n2

,

véletlen, hogy ez nagyfokú hasonlóságot mutat cik-künk második részében bemutatandó kábelvégijel-reflexiós kifejezéssel.

Mechanikában az impedanciaillesztés problémájá-val analóg az a jelenség, hogy testek rugalmas ütkö-zésekor az energiaátadás egyenlõ tömegû testek kö-zött a maximális, amit jól szemléltet a Newton-bölcsõ.A mechanikai hullámok visszaverõdésének szerepevan még robbanóanyagok alkalmazásánál és a külö-nösen nagy rezgésnek kitett eszközök csillapításaesetében is.

Még egyenáramú hálózatok esetén is találkozha-tunk impedanciaillesztéssel: jól ismert tény, hogy egy

adott belsõ ellenállású telepbõl egy terhelõ ellenállá-son akkor vehetjük ki a maximális teljesítményt, ha aterhelés ellenállása megegyezik a telep belsõ ellenál-lásával.

Kísérletek a kábelbeni hullámjelenségekbemutatására

A kísérletekhez egy 10 MHz-es jelgenerátorra, 10MHz-es oszcilloszkópra és 10-20 méter BNC-csatlako-zós, 50 Ω-os kábelre lesz szükségünk (legelterjedtebbaz RG58 típus). Az ilyen frekvenciára képes jelgenerá-torok kimeneti impedanciája 50 Ω, amit a kimenetifelirat jelez általában. A legtöbb oszcilloszkóp beme-neti impedanciája 1-10 MΩ, ami a mérésre megfelelõ,tehát nem szükséges 50 Ω-os bemenetû nagyfrekven-ciás oszcilloszkóp.

Elsõ lépésként mutassuk be, hogy a koaxiális ká-belek miért szükségesek a nagyfrekvenciás jelek to-vábbításához! Az összeállítást a 4.a ábra mutatja.Ehhez 10 MHz-et állítsunk be a jelgenerátoron, és azábrán látható módon kössük össze az oszcilloszkóp-pal, azaz két közönséges szigetelt rézdróttal, amelye-ket banán BNC-csatlakozó segítségével csatlakozta-tunk. Azt fogjuk tapasztalni, hogy az oszcilloszkópáltal mért jel erõsen függ attól, hogy a rézdrótok egy-máshoz képest miként helyezkednek el. A kezünketközel mozgatva is hatást fogunk találni, és a rézdró-tok közepét vízbe (még inkább sós vízbe) merítve iskomoly változást találunk az átvitt jelben. Ügyeljünk

282 FIZIKAI SZEMLE 2018 / 7–8

Page 69: Fizikai Szemle 68/7–8 – 2018. július–augusztusgoliat.eik.bme.hu/~f.simon/publications/Media/FizSzem-20180708.pdf · okokból standard modellnek hívunk, a világot anyagi és

arra, hogy a víz ne teremtsen galvanikus kapcsolatota két rézdrót között, mert a kísérlet lényege, hogy avíz, mint dielektrikum is befolyásolja a drótok körülielektromágneses tér eloszlását és ezen keresztül azátvitt jelet. Ez a kísérletsorozat jól szemlélteti, hogymiért is van szükség koaxiális kábelekre, amelyeketalkalmazva az átvitt jel nagysága nem függ a külsõkörnyezettõl.

A következõkben a kábel végérõl történõ visszave-rõdést szemléltetjük, egyben megmérjük a kábelbenijelterjedés sebességét, azaz a kábelbeni fénysebessé-get. A mérési elrendezést a 4.b ábra mutatja. Az osz-cilloszkópon a jelgenerátor kimenõ frekvenciájánakfüggvényében vizsgáljuk a mért feszültséget, miköz-ben egy hosszabb BNC-kábel is rá van csatlakoztatva,aminek végét különbözõképpen zárjuk le. A jelgene-rátor kimenõ feszültségét jelölje Vgen, a kábel hosszátl. Azt fogjuk találni, hogy amikor a kábel végén szaka-dás vagy rövidzár van, az oszcilloszkópon mért jelfrekvenciafüggõ lesz. Amennyiben 50 Ω-mal zárjuk le,úgy frekvenciafüggetlen jelet találunk. A másik kétesetben a 3. ábrán bemutatottakat figyelhetjük meg.A kábelbeni hullámhossz λk = ck/f, ahol ck a kábelbe-ni fénysebesség (a szokásos kábelekre ck ≈ 0,65c0).

Amennyiben a kábel végén szakadás van, az oszcil-loszkópon (közel) 0, ha a kábel hossza a λk/4 párat-lan, illetve Vgen feszültséget találunk, ha a negyedhul-

lámhossz páros (a 0-t is beleértve) számú többszörö-se. Ha a kábel végén rövidzár van, akkor pont fordít-va, azaz az oszcilloszkópon Vgen, illetve (közel) 0 fe-szültséget találunk, ha a kábel hossza λk/4 páratlan,illetve páros számú többszöröse.

Ez a feladat több mindenre felhasználható: példáulismert hosszúságú kábel esetén segítségével a kábel-beni fénysebesség mérhetõ, majd ezután ismeretlenkábel hosszának meghatározására is használható. Afeladat során – amennyiben rendelkezésre áll – cél-szerû a jelgenerátor triggerkimenetérõl triggerelni azoszcilloszkópot azért, hogy stabil jelalakokat figyel-hessünk meg, még akkor is, amikor az oszcilloszkó-pon kis jeleket mérünk. Ha 10 MHz-es jelgenerátorunkvan, az elsõ minimum méréséhez legalább 5 m-es ká-bel szükséges.

A következõ részben bemutatjuk a kábel végérõl tör-ténõ visszaverõdések egzakt tárgyalását, valamint pél-dát mutatunk a gyakorlatban is alkalmazott, nem re-zisztív, kábelvégi lezárásra, amit többek között mag-mágnesesrezonancia-spektroszkópiában használunk.

Ajánlott és felhasznált irodalomBudó Ágoston: Kísérleti fizika II. – Elektromosságtan és mágnesség-

tan. Nemzeti Tankönyvkiadó, Budapest (1997)Simonyi Károly: Elméleti villamosságtan. Tankönyvkiadó, Budapest

(1967)David M. Pozar: Microwave Engineering. (4th ed.) Wiley (2011)

A CSAPBÓL IS RÉSZECSKEFIZIKA FOLYIK?

Köszönjük Horváth Dezsõnek, Lévai Péternek és Varga Dezsõnek,valamint iskoláink igazgatóinak, hogy segítették és támogattákmunkánkat, lehetõséget nyújtottak a programok szervezésében, le-bonyolításában. A tanulmány elkészítését a Magyar TudományosAkadémia Tantárgy-pedagógiai Kutatási Programja támogatta.

Oláh Éva Mária a Bálint Márton Általánosés Középiskola fizikatanára és a MTA Wig-ner Fizikai Kutatóközpont dolgozója. AzELTE Fizika Doktori Iskolájában szerzettPhD fokozatot, kutatási területe a részecs-kefizika tanítása középiskolában. A CERN-ifizikatanár-továbbképzés szervezõje és elõ-adója, tagja az ELFT elnökségének és a Szi-lárd Leó fizikaverseny versenybizottságá-nak. Elismerései: 2015-ben MTA Pedagó-gus Kutatói Pályadíj; 2016-ban EMMI Bo-nus Bona, A nemzet tehetségeiért díj.

Fülöp Csilla az ELTE TTK-n szerzett mate-matika-fizika-angol szaktanári szakos dip-lomát. A közoktatás széles skáláján szer-zett tanári tapasztalatot: egy angliai ma-gániskolában, fõiskolai vezetõtanárként,reál hatosztályos és humán elitgimnázium-ban, valamint szakközépiskolában. Érdek-lõdési területe az informális fizikaoktatásés a „hands-on, minds-on” didaktika. Dok-tori értekezését 2018-ban védte meg azELTE-n.

Oláh Éva Mária – Bálint Márton Általános és Középiskola, Törökbálint

Fülöp Csilla – Madách Imre Gimnázium, Budapest

Középiskolás fizikatanárként azt szeretnénk körüljár-ni, hogy a mai 14–18 éves diákoknak milyen lehetõsé-gei vannak részecskefizikát tanulni tanórai, vagy bár-milyen más keretek között. Elõször megvizsgáljuk,hogy napjainkban mi a helyzet a keret-, illetve a helyitantervek terén.

Részecskefizika a tantervekben ésaz érdeklõdés középpontjában

Az Oktatási Hivatal kétféle tantervet ajánl. A felméré-sek szerint az iskolák többsége a „B” kerettantervetválasztja. Ebben a tantervben konkrét részecskefizikaifogalmak egyáltalán nem szerepelnek, csak általános-ságban írja le azt, hogy ismertessük a diákokkal azatommag szerkezetét. Az „A” kerettantervben ugyanszerepel a kvark kifejezés, de csak így, önmagában.

A FIZIKA TANÍTÁSA 283

Page 70: Fizikai Szemle 68/7–8 – 2018. július–augusztusgoliat.eik.bme.hu/~f.simon/publications/Media/FizSzem-20180708.pdf · okokból standard modellnek hívunk, a világot anyagi és

A Maxim tankönyvkiadó tanmenetében találtuk alegtöbb részecskefizikához köthetõ fogalmat. Itt azalapvetõ kölcsönhatásokon túl szerepel a kiegészítõtananyagban a standard modell, a mikrorészecskékrõlés a jelenleg mûködõ gyorsítókról, azok gyakorlatifelhasználásáról is lehet beszélgetni a tanulókkal.Néhány helyi tantervben ugyancsak szerepel a kvar-kok, illetve az elemi részecskék világa, de azt gondol-juk, hogy ez inkább azon fizikatanároknál fordul elõ,akik maguk is jártak már a CERN-ben, az EurópaiNukleáris Kutatási Szervezet részecskefizikai laborató-riumában.

Összességében azt állapíthatjuk meg, hogy hiábaismertek csaknem 100 éve a részecskefizika kutatásieredményei, a tananyagban kötelezõ szinten egyálta-lán nem, kiegészítõ szinten is csak nagyon érintõlege-sen szerepelnek.

Viszont mindig azt tapasztaljuk, hogy a modernfizika fejezetei iránt óriási diákok érdeklõdése, ezenbelül a mikrorészecskék fizikája – az LHC indítása óta– is nagyon népszerû. A média egyre többet foglalko-zik vele, internetes hírportálokon, ismeretterjesztõtévémûsorokban gyakran találkozhatunk az LHC,hadronütköztetõ, Higgs-bozon kifejezésekkel. Mivel adiákjaink is kedvelik ezen mûsorokat, illetve olvassáka szenzációhajhász címekkel ellátott cikkeket, termé-szetes, hogy velük kapcsolatban kérdésekkel fordul-nak hozzánk, így a kielégítõ válaszok megadásáhoznekünk is naprakésznek kell lennünk.

A CERN minden év augusztusában egyhetes to-vábbképzést biztosít a fizikatanárok számára, de eb-ben az írásban részletesebben a diákok lehetõségeivelfogunk foglalkozni. A teljesség igénye nélkül áttekint-jük a lehetõségeket: hazai, középiskolai, illetve a Wig-ner Fizikai Kutatóközpont által szervezett programo-kat, végül a CERN 2017-tõl induló új továbbképzésétmutatjuk be.

Részecskefizikai diákmûhelyek

2005 óta a CERN az IPPOG (International ParticlePhysics Outreach Groups) keretében kora tavasszalNemzetközi Diákmûhelyt (Masterclass) szervez. En-nek programja hazánkban az MTA Wigner Fizikai Ku-tatóközpont (korábban KFKI) szervezésében az or-szág középiskoláiba járó diákok számára a Lehetsz Teis kutató egy napra program. Iskolánkként két tanulóttudnak fogadni. Eleinte egy-egy helyszínen 20 fõ ve-hetett részt, viszont a nagy érdeklõdésre való tekintet-tel az utóbbi években már duplájára növekedett alétszám.

A nap folyamán a diákok elõadásokat hallgatnak azalapvetõ kölcsönhatásokról, gyorsítókról, detektorok-ról. A közös ebédet követõen pedig az LHC egyikkísérlete, a CMS adatait számítógépes program segít-ségével elemzik, sõt maguk is találhatnak olyan ese-ményeket, amelyek a Higgs-bozon jelenlétére utal-nak. A program hivatalos elnevezése NemzetköziRészecskefizikai Diákmûhely, ugyanis adott napon

egyszerre több országban is megrendezésre kerül, ésaz adatelemzést követõen a tanulók a kapott eredmé-nyeket angol nyelvû videokonferencia segítségévelmeg is osztják egymással [1].

Ez a program ugyanolyan nagy sikerrel folyik évekóta Debrecenben is a Debreceni Egyetem, és Székes-fehérváron az Óbudai Egyetem Alba Regia Kara szer-vezésében.

Természettudományos önképzõkör

Több iskolában, többek között a Bálint Márton Általá-nos Iskola és Középiskolában is mûködik olyan szak-kör, ahol az érdeklõdõ tanulók a középiskolai tan-anyagon túlmutató tudományos elõadásokat hallgat-hatnak. A biofizikától a kozmológiáig több neves tu-dós fogadja el a felkérést, tart elõadást. De „mindenszentnek maga felé hajlik a keze”, a legtöbb elõadás afizika, azon belül is a részecskefizika legújabb ered-ményeivel foglalkozik.

Az önképzõkörök feladata mindig az volt, hogyfelkeltse a gyerekek érdeklõdését, majd ezt követõenközös projektek kidolgozására is sor kerülhet. Ezekáltalában havi vagy kéthavi rendszerességgel kerül-nek lebonyolításra és alkalmanként több száz tanulóis hallgatja az elõadásokat. Óriási öröm, hogy ezt kö-vetõen a diákok kérdésekkel fordulnak az elõadóhoz,vagy maguk is egyéni kutatásokba kezdenek a téma-körben [2].

Higgs-projekt

Az alkalmazott fizika, azaz a mérnöki tudományokiránt érdeklõdõ szakközépiskolás tanulók körében ta-lán még intenzívebb az érdeklõdés a kutatások ered-ményeinek megismerésére. Informatikai és elektro-technikai, valamint gépésztanulók nem csak fizika, deszakmai órákon is hallottak az LHC-rõl. A Higgs-bozon felfedezésének híre hívta életre a Higgs-pro-jektet a Trefort Ágoston Kéttannyelvû Szakközépisko-lában. A személyes látogatás anyagi hátterét nem si-került megteremteni, ezért teljesen más megoldástkellett találnunk. Az érdeklõdõk egy elõadás soránátláthatták a témát, Mi is a részecskefizika? címmelfelvillantak a történelmi háttér, az alkalmazott beren-dezések, a tudományos elmélet alapjai. A Particle Zoo[5] egy tûpárnaszerû plüssfigurákból összeállítottkészlet, amely a részecskék alapvetõ tulajdonságaittanítja meg. A projektben résztvevõ tanárok és diákokegy-egy ilyen plüss elkészítésével, és az adott részecs-ke jellemzésével készültek a második elõadásra, ame-lyet Horváth Dezsõ tartott a Higgs-bozon felfedezésé-rõl Peter Higgs születésnapján (ezen különleges alka-lomból az elõadás végén elfogyasztottunk egy tortát).A tanulók megismerhették a részecskék típusait (bo-zon, lepton, kvark), és az anyag-antianyag párokat.Az egyéni munkájukhoz elsõsorban angol nyelvûszakirodalmat használtak az internetrõl.

284 FIZIKAI SZEMLE 2018 / 7–8

Page 71: Fizikai Szemle 68/7–8 – 2018. július–augusztusgoliat.eik.bme.hu/~f.simon/publications/Media/FizSzem-20180708.pdf · okokból standard modellnek hívunk, a világot anyagi és

Játékos részecskefizika kockákkal

1. kép. Oláh Éva mûhelyfoglalkozása és tartós, fából készült ré-szecskekocka-készlete.

Nincs könnyû feladatunk, ha diákjaink – az érdeklõ-dés felkeltésén túl – többet szeretnének tudni a mik-rovilág rejtelmeirõl, hiszen a tananyagban csak a ko-rábban említett mennyiségben szerepel a részecskefi-zika-témakör, viszont az itt felsorolt programok kap-csán lépten-nyomon olyan kifejezésekkel találkoznak,mint a „bozon”, „hadron”, „lepton” szavak. Memorizá-lásukhoz fizikatanárként is hosszú idõ kellett. Fontos,hogy a szubatomi részecskék mérettartományában(tipikusan 10−18 m alatt) szemléletes képet tudjunkalkotni. Gyakorlatból tudjuk, hogy a bevésõdés leg-magasabb szintje az, amikor a tanulás gyakorlati fel-adattal is párosul (hands-on, minds-on didaktika).Ehhez egy oktatási segédeszközt – amely a részecske-fizika alapvetõ fogalmainak és törvényeinek játékostanulását szolgálja – dolgoztunk ki. Az eszköz papír-kockákból áll, mert a hat lapján különféle fizikai fo-galmakat lehet feltüntetni, mint a kvarkok ízei, a lep-tonfajták vagy a megmaradó mennyiségek. A diákok –miközben elkészítik saját kockakészletüket – észre-vétlenül tanulják a sokszor érthetetlen és nehezenmegjegyezhetõ kifejezéseket. Az elkészült készletsegítségével pedig a hadronok felépítése, a légkörfelsõ rétegeiben lejátszódó bomlási folyamatok, vagy

akár a középszinten is elvárt béta-bomlás mélyebbszerkezete is érhetõbbé válik.

Az ország különbözõ középiskolái gyakran szer-veznek elõadásokat, amelyek között az „részecskefizi-ka-építés” is többször szerepelt. Az elõadásokat idõn-ként mûhelyfoglalkozással is egybekötik, ahol elké-szítik a kartonpapírból kivágott részecskekockák.Ezen saját készletek felhasználásával késõbb – szak-körök vagy különórák során – a gyerekekkel közösenlehet újabb projekteket kitalálni. A készlet továbbfej-leszthetõ, vele például a kozmikus müonok bomlásais bemutatható, amihez persze újabb kockákat kellgyártani. A különbözõ megmaradási törvényeket is belehet mutatni és gyakoroltatni (1. kép ). A tartósabbváltozat akár fakockákból – festéssel, feliratozással,lakkozással – is készülhet, hosszú ideig használhatóoktatási segédanyagot nyújtva a fizikaszertárak szá-mára [3].

Tanulmányi kirándulások a CERN-be

A világ legnagyobb részecskefizikai kutatóközpontja,a CERN nagy hangsúlyt fektet az oktatásra, mindenévben egyhetes, a fizikatanárok anyanyelvén tartotttovábbképzéseket szervez (például 2016-ban 39 alka-lommal), ezen kívül a nemzetközi tanártovábbképzé-sek során többhetes tanulásra ad lehetõséget, illetverendszeresen fogad középiskolás diákcsoportokat is.Ennek megszervezése nem is olyan nehéz feladat, deidõben kell jelentkezni, mert óriási az érdeklõdés. ACERN látogatói központján keresztül kell megadni azidõpontot, amikor a csoport érkezni szeretne, és azintézet egy teljes napon keresztül enged bepillantástnyerni a leghíresebb és legnagyobb volumenû kísérle-tekbe. Ezek közé tartozik a CMS-detektor látogatása,és ha éppen nem mûködik a gyorsító, akkor közvet-len közelébe is el lehet jutni, 10-15 méter távolságbóllehet megtekinteni a mérnöki tudomány egyik csodá-ját. A diákok az újonnan megépített S’Cool Lab elne-vezésû laboratóriumban saját detektorukat, a még mais nagyon érdekfeszítõ eszköznek számító ködkamrátis megépíthetik (2. kép ).

Két ingyenesen látogatható kiállítást is mûködtet aCERN, a Microcosmosban interaktív módon érthetikmeg a gyorsítókban és detektorokban lejátszódó fo-lyamatokat, a CERN jelképévé váló Globe -ban pedig aRészecskék univerzuma címû kiállítás látható. Mind-két szerzõ iskolájukba járó diákjai számára több alka-lommal szervezett látogatást a CERN-be. Egy-egy al-kalommal 20-30 diák ismerkedhetett – ezen nem ha-gyományos módon – a részecskefizika rejtelmeivel.

A Science Center pedagógia kutatási eredményei éssaját tapasztalataink alapján javasoljuk ezen tanulmá-nyi kirándulásokat egy projekt keretébe ágyazni. Alátogatás is érdekesebb lesz, és több, maradandóbbtudás alakul ki tanítványainkban, ha néhány lépésbenelõkészítjük az utat, majd a projekt lezárásaként, utó-követésként még egy programponttal fejezzük be azt.Például a Madách Imre Gimnáziumban a tanulók há-

A FIZIKA TANÍTÁSA 285

Page 72: Fizikai Szemle 68/7–8 – 2018. július–augusztusgoliat.eik.bme.hu/~f.simon/publications/Media/FizSzem-20180708.pdf · okokból standard modellnek hívunk, a világot anyagi és

rom, egymásra épülõ szerve-

2. kép. Középiskolások a CERN-ben: balra fönt a szuvenírbolt, alatta ködkamraépítés a S’Cool Lab-ben, jobbra a szétszerelt CMS-detektor.

zett elõadás és mûhely kere-tében készülhettek fel a láto-gatásra, valamint egy otthonkitöltendõ munkalapot is kap-tak, amely a National Geog-raphic ismeretterjesztõ filmjé-hez készült. A a genfi kirán-dulást követõen pedig virtuá-lis látogatáson vettek részt,ahol a CERN-ben dolgozó ma-gyar fizikusok, mint kedvesismerõsök zárták a CERN–MIG 2017 projektet.

A svájci tanulmányi kirándu-lás – természetesen – nem csu-pán a fizikát érinti, hanem sze-rep jut a kultúrának is. A diá-kok „kincsvadászat” játékkal,kis csoportokban versenyezvefedezhetik fel Genf nevezetes-ségeit, történelmi helyeit.

CERN Open Days a Wigner FK-ban

A Wigner FK-ban minden év szeptemberében nyílthétvégét rendeznek, amely minden érdeklõdõ számá-ra szabadon látogatható, de – természetesen – a taná-rok és diákok jelenléte a jellemzõ. Ezen a két napon arésztvevõk bepillanthatnak a részecskefizika fellegvá-ra, a CERN legújabb eredményeibe, az ott dolgozómagyar fizikusok aktuális kutatásaiba.

A csillebérci telephelyen mûködõ Adatközpont issok érdekességet tartogat az érdeklõdõk számára. Azegyes laboratóriumokban fejlesztett eszközök külön-féle tematikájú sátrakban tekinthetõk meg, a „SokszínûFizikabusz”-ban pedig látványos, interaktív kiállításvárja az érdeklõdõket. Újdonságnak, hogy idén azok aközépiskolás diákok, akik a CERN által szervezett elsõkéthetes továbbképzésen vettek részt, bemutathatjákaz ott végzett, mini kutatótevékenységüket [4].

Diákok kutatómunkája a Wigner FK-ban

Az MTA Wigner FK több intézete régóta fogad közép-iskolás diákokat különbözõ látogatásokra, vagy akáregyhetes nyári táborra is, de folyamatos tevékenység-re, diákkutatásra csak egyetemista hallgatóknak voltlehetõsége. A Részecske és Magfizikai Intézet Nagy-energiás Fizikai osztályán viszont több éve már hetirendszerességgel dolgozhatnak középiskolás diákok,akik bepillantást nyerhetnek a kutatók mindennapitevékenységébe. Megtehetik mindezt azáltal, hogy azosztályon mûködõ Detektorfizikai Kutatócsoport fel-vállalta, hogy a 14–18 éves korosztály számára is lehe-tõséget biztosít. Ennek keretében megismerhetik, hogymivel foglalkozik egy fizikus, és így pályaválasztásuk-nál már tudatosan választhatják a fizika szakirányt.

A kutatócsoport gáztöltésû detektorok fejlesztésé-vel foglalkozik, és – a középiskolás diákok segítsé-gével – évek óta azt is vizsgálja, hogy miként lehet alegmodernebb észlelõ berendezéseket a középisko-lai oktatásba is bevinni. Jelenleg a fizikaórákon vagya szakkörökön különféle sugárzások bemutatására,észlelésére a még mindig népszerû ködkamra, vagya radioaktív sugárzást bemutató Geiger–Müller-számlálócsõ van használatban. Elõbbi hátránya,hogy mûködtetéséhez nehezen beszerezhetõ száraz-jég szükséges, utóbbit pedig azért nem szeretjükhasználni, mert a radioaktív sugárzás káros lehet azegészségre. Ezzel a Nagyenergiás Fizikaosztály labo-ratóriumában készül detektorok, a sokszálas, pro-porcionális kamrák az egészségre ártalmatlan, koz-mikus müonok vizsgálatára lettek kifejlesztve. Adiákok által összeállított mérete tipikusan 20 × 20 cm(3. kép ) – a professzionális változatok lényegesennagyobb méretben készülnek –, az utóbbi idõben, aköltséghatékonyság érdekében, még kisebbekkel ispróbálkoznak. Az ilyen jelentõs detektorépítésimunkálatok elõtt a diákoknak meg kell ismerked-niük a különbözõ elõkészítõ részfeladatokkal is,eközben megtanulják használni a mûhely gépeit,eszközeit. Azok, akik jobban értenek az elektroniká-hoz, például megtervezhetik és össze is állíthatjákazon árammérõ mûszert, amellyel meg lehet mérni adetektor nagyon kicsiny áramát, és egyben nagyfe-szültség mérésére is alkalmasak.

A professzionális detektorokhoz gyakran használ-nak különleges anyagokat, speciális fóliákat is. Ezektesztelését is – az úgynevezett szórási kísérletekkel –sokszor a diákok végzik. Ez a lehetõség a középisko-lás diákok számára nagy lépést jelent az egyetemitovábbtanulásuk orientációjában, hiszen a KutatásAlapú Oktatás (Research Based Learning) részeseilehetnek.

286 FIZIKAI SZEMLE 2018 / 7–8

Page 73: Fizikai Szemle 68/7–8 – 2018. július–augusztusgoliat.eik.bme.hu/~f.simon/publications/Media/FizSzem-20180708.pdf · okokból standard modellnek hívunk, a világot anyagi és

CERN HSSIP

3. kép. Ismerkedés az eszközök használatával, és egy mûködõ müonkamra.

4. kép. A világelsõ, magyar középiskolás csoport a Globe és a „Mág-nesgyár” látogatásakor.

A fenti kezdeményezések, lehetõségek Magyarorszá-gon adódnak, viszont 2017 májusában fantasztikuskezdeményezéssel állt elõ a CERN oktatásért felelõsbizottsága. Ne csak fizikatanárok vagy legalább másod-évet végzett egyetemisták juthassanak el tanulás céljá-ból Európa legnagyobb részecskefizikai kutatóköz-pontjába, hanem – még azelõtt, hogy döntést hoznánaktovábbtanulásukról – középiskolás diákok is. A cél,hogy egy gyakorlati, bentlakásos továbbképzés sorántapasztalhassák meg, miért is érdemes a fizikát vagy amûszaki tudományokat választani. 2017-ben összesenöt ország nyerte el a részvétel jogát, és ezek között isMagyarország lehetett az elsõ, és 22 diákkal (4. kép )képviseltethette magát ezen a rendkívüli programon.

A tanulók kettesével csatlakoztak egy-egy kint dol-gozó fizikushoz, mérnökhöz vagy informatikushoz,akikkel két héten keresztül mindenben együtt tevé-kenykedtek. Programokat írtak a szupravezetõ mágne-sek teszteléséhez, részt vettek a CMS-kísérlet éjszakaimûszakjában, CNC-gépekkel esztergálhattak, vagy koz-mikus müonokat észlelõ detektorokat szimulálhattak.Ezen a kísérleti jellegû továbbképzésen Magyarország19, az ország különbözõ középiskolájában tanuló diákvehetett részt, többnyire iskolánkként egy fõvel, kivé-telt csak a már régebb óta, a Wigner FK-ban kutató-munkát végzõ tanulók jelentettek. A CERN-nek jelenleg22 tagországa van, a tervek szerint ehhez a programhozminden évben csak öt ország csatlakozhat, így hazánk-ra elõreláthatóan csak öt év múlva kerül újra sor [5, 6].

Összegzés

Átlagemberként is azt tapasztaljuk, hogy a részecske-fizika korunk egyik legdinamikusabban fejlõdõ tudo-mányága, és sajnos a középiskolai tananyagban mégcsak érintõlegesen szerepel. Ezért fontos, hogy a diá-koknak a tanórán kívül is adjunk lehetõséget ezzel anagyon sokrétû, érdekes témakörrel foglalkozni, amikrovilág mélyebb rejtelmeit megismerni. A tanár-kollégák – kezdeményezéseik kidolgozásához – szá-míthatnak a kutatók támogatására. E munkában – aCERN-nel karöltve – élen jár az MTA Wigner FizikaiKutatóintézet. Mindkét intézet hatalmas anyagi ésemberi ráfordítással dolgozik azért, hogy a középis-kolás diákok úgy érezhessék: a „csapból is részecske-fizika folyik”.

Irodalom1. http://www.rmki.kfki.hu/~jancso/Reszecskefizikai_Diakmuhely_

Wigner_FK/2. http://www.balintsuli.hu3. Oláh Éva Mária: Építsünk Részecskefizikát! Nukleon X. (2017)

203. ISSN: 1789-9613, Magyar Nukleáris Társaság 2017.4. http://cernopendays.wigner.mta.hu5. http://wigner.mta.hu/hu/2-het-22-magyar-kozepiskolas-genf-

hssip-20176. http://mta.hu/tudomany_hirei/huszonket-magyar-diakot-latott-

vendegul-ket-heten-at-a-cern-107900

A FIZIKA TANÍTÁSA 287

Page 74: Fizikai Szemle 68/7–8 – 2018. július–augusztusgoliat.eik.bme.hu/~f.simon/publications/Media/FizSzem-20180708.pdf · okokból standard modellnek hívunk, a világot anyagi és

KÖNYVESPOLC

Daniel Whiteson, Jorge Cham: HALVÁNYLILA GÕZÜNK SINCS– Tutikalauz az ismeretlen UniverzumhozEurópa Könyvkiadó, Budapest, 2017, 375 oldal

A könyv eredeti címe We have no idea. Fordította: Kovács József,szakértõ: Szabados László.1 http://phdcomics.com

„Szeretnéd tudni, hogyan keletkezett az Univerzum,mibõl áll, és mi lesz a végsõ sorsa? Szeretnéd megér-teni, hogy mi a tér és mi az idõ? Szeretnéd tudni, hogyegyedül vagyunk-e a Világegyetemben? Pech! Ez akönyv ugyanis egyik kérdésre sem ad választ.”

Ezzel a formabontó felütéssel kezdõdik a Halvány-lila gõzünk sincs címû könyv, és a folytatás is hasonlószellemben tárgyalja az emberiséget régóta foglalkoz-tató, nagy kérdéseket. Ez persze csak azok számáraokoz meglepetést, akik nem isme-rik az amerikai szerzõpáros koráb-bi munkásságát: Daniel O. White-son a University of California, Ir-vine részecskefizikus-professzoraés Jorge Cham, a mérnöki tudomá-nyokból PhD-fokozattal rendelke-zõ képregényszerzõ (a hazai kuta-tói közösség fiatalabb tagjai köré-ben itthon is méltán nagy népsze-rûségnek örvendõ PhD Comics 1

weboldal készítõje) évek óta kül-detésüknek tekintik, hogy a tudo-mány legújabb eredményeit az újgeneráció számára a legjobban fo-gyasztható formában tálalják (többanimációjuk és elõadásuk többekközött a legismertebb videomeg-osztó portálon is elérhetõ).

Whiteson és Cham ezúttal arravállalkozott, hogy egy könyv kere-tében mutassák meg, mit (nem) tu-dunk jelenleg a bennünket körül-vevõ világról – elsõsorban a tinédzser, illetve fiatalfelnõtt olvasóközönséget megcélozva. A Világegye-tem nagy rejtélyeit boncolgató, kifejezetten ismeret-terjesztõ célú anyag készítése (legyen szó könyvrõl,filmrõl, vagy akár képregényrõl) mindig nagy kihívás,hiszen a bonyolult témákról jellemzõen a matematikaiés fizikai háttér szelíd „elrejtése” mellett kell beszélni– mindezt lehetõség szerint élvezhetõ, ugyanakkorhiteles és koherens formában. Ez a szerzõi oldalrólfellépõ nehézségek mellett ráadásul egy olyan nemkívánt mellékhatással is járhat (amivel középiskolaitanár kollégáink közül valószínûleg sokan szembesül-tek már), hogy egyes diákok nagy adag „virtuális tu-

dást” építenek fel magukban az egyébként komplexhátterû fogalmak és jelenségek (fekete lyukak, téridõ,dimenziók, …) szemléltetõ bemutatásából, míg azezek hátterének megértéséhez (is) szükséges fizikaialapismeretek elsajátítását már jóval kevésbé tartjákfontosnak – sõt, sokszor unalmasnak. Szintén fontostényezõnek tûnhet a „piac telítettsége”, hiszen korunklegkiválóbb, ismeretterjesztésben is élen járó elméiután (hogy ezen a helyen is megemlékezzünk a kö-

zelmúltban elhunyt Stephen Haw-kingról, ugyanakkor több más,hasonló kvalitású kortársunkra isgondolva) nehéznek tûnik „újat”mondani a világról – ráadásul alátványos, de sajnos nem feltétle-nül egységesen magas színvonalútematikus TV-mûsorok és interne-tes adások „özönében” könnyenelsikkadhat egy n+1-edik, az érin-tett témákkal foglalkozó, nyomta-tott mû.

Mindezek mellett – ezen sorokírója szerint – a kérdéses könyvetmindenképpen megérte kiadni, ésmindannyian jó szívvel ajánlhatjuka tudomány világa iránt érdeklõdõismerõseinknek (azoknak leg-alábbis mindenképpen, akiktõl azabszurd humor sem áll nagyontávol). Whiteson és Cham csaknem400 oldalon keresztül kalandozikaz anyaggal, a térrel és az idõvel

kapcsolatos (nem)ismereteink tengerén, s mindeztmagabiztosan kormányozva teszik – közben rekesziz-mainkat, illetve szó- és képi viccek feldolgozását szol-gáló agykapacitásunkat is erõsen próbára téve :). A 16fejezet során – a 13. fejezet „természetesen” hiányzik:) – az elemi részecskék és kölcsönhatások világábólindulva a Világegyetem kialakulásának, fejlõdésénekés felépítésének taglalásán keresztül eljutunk annakelképzelt végkifejleté(i)ig, közben olyan kérdéseket isfelvetve – és (nem) megválaszolva –, mint a fényvéges határsebessége, a nagy energiájú kozmikussugárzás eredete,2 vagy éppen a földönkívüli civilizá-

2 Említésre kerül például a CRAYFIS nevû kezdeményezés is, amely-nek révén egy okostelefonra letölthetõ alkalmazás segítségével kap-csolódhatunk be a kozmikus részecskék vadászatába: http://crayfis.io

ciók megtalálásának esélye.

288 FIZIKAI SZEMLE 2018 / 7–8

Page 75: Fizikai Szemle 68/7–8 – 2018. július–augusztusgoliat.eik.bme.hu/~f.simon/publications/Media/FizSzem-20180708.pdf · okokból standard modellnek hívunk, a világot anyagi és

A könyv egyik kiemelkedõ pozitívuma, hogy azelvont fogalmak többségét igyekszik a mindennapitapasztalatokhoz köthetõen bevezetni, s mindezt kel-lõen didaktikusan teszi. Ezen a téren néha talán kicsittúl tömény az abszurd humor (ami néhány kockányiképregényként jól mûködik, oldalakon keresztül egyidõ után sokak számára fárasztóvá válhat), illetve né-hány poén valószínûleg kevéssé jön át a mostani ti-zenévesek számára (mint például az X-akták vagy aStar Trek tévésorozatokra való többszöri utalás) –ugyanakkor várhatóan igen sikeressé teszi majd akönyvet az egyetemisták, fiatal kutatók és pályakezdõtanárok körében. Külön kiemelendõ a fordítást végzõKovács József és a szakértõként felkért, egyben „szójá-tékos kedvében a fordító tettestársaként” is közremû-

ködõ Szabados László kollégáink munkája, akik szá-mos „komolyabb” hangvételû mû magyar nyelvrevaló, magas szintû átültetése után az abszurd humor-ral fûszerezett ismeretterjesztés területén is maradan-dót alkottak, több helyen érezhetõ módon saját nyelvileleményeikkel is gazdagítva a kötetet.

Összefoglalva: a Halványlila gõzünk sincs stílusa,felépítése és a nem-tudás irányából közelítõ nézõpont-ja okán is újszerû kísérlet egy nehéz mûfajban, amely-nek jellemzõ buktatóinak többségén sikeresen lendülát – s még ha a könyv minden része nem is feltétlenülérinti meg a fõ célközönségnek számító tizenéves kor-osztályt, a tudomány világa iránti érdeklõdés erõsítésé-re mindenképpen alkalmas és ajánlott.

Szalai Tamás

HÍREK – ESEMÉNYEK

ÖTVEN ÉVVEL EZELÕTT HUNYT EL GYULAI ZOLTÁN,A HAZAI KÍSÉRLETI SZILÁRDTEST-FIZIKA ÚTTÖRÕJEGyulai Zoltán akadémikus, a Budapesti Mûszaki Egye-tem Kísérleti Fizika Tanszék egykori vezetõje, az Eöt-vös Loránd Fizikai Társulat egykori elnöke halálánakötvenedik évfordulóján, 2018. július 13-án, egykorimunkatársai kezdeményezésére az Eötvös Loránd Fizi-kai Társulat, a BME Fizikai Intézete és a Magyar Tudo-mányos Akadémia Fizikai Tudományok Osztálya ko-szorúzással egybekötött megemlékezést tartott GyulaiZoltán BME kertjében lévõ mellszobránál. Az alkalmonaz egykori munkatársak és diákok, valamint a BME

Fizikai Intézete mai munkatársai mellett szép számbanvettek részt Gyulai Zoltán leszármazottai és családtag-jaik is. Életérõl és munkásságáról korábban már jelen-tek meg cikkek e folyóirat hasábjain, mégis – bár nevétaz ELFT a mai napig is õrzi egy díjjal – személye mamár kevésbé ismert a szélesebb magyar fizikus közös-ség elõtt. Ezért illõ ebbõl az alkalomból a FizikaiSzemlében is újra megemlékeznünk róla. Sólyom Jenõ,az ELFT elnöke koszorúzáson elmondott beszédétHartmann Ervin Gyulai-iskoláról szóló írása követi.

EMLÉKBESZÉD GYULAI ZOLTÁN MELLSZOBRÁNÁLSólyom Jeno

MTA Wigner Fizikai KutatóközpontELTE, Fizikai Intézet

Gyulai Zoltán a „boldog békeidõkben”, egy látszólagnyugodt korban, 1887-ben született egy csendes erdé-lyi, döntõ többségében magyarok lakta faluban, aKisküküllõ vármegyei Pipén. Innen indulva egy rend-kívül mozgalmas, a két világháború által súlyosanbefolyásolt, végül mégis magasba ívelõ életutat futottbe. Az elsõ világháború az alig kezdõdõ tudományospályát megtörve hosszú szibériai hadifogságba juttat-ta, a második miatt pedig kétszer is egyetemet kellettváltania, Debrecenbõl Kolozsvárra, majd onnan Buda-pestre, és mindig újra kellett kezdenie az oktatói és

kutatói elképzeléseinek megfelelõ tanszék megszer-vezését, tudományos munkáját, amelyet azután Kos-suth- és Állami díjjal is elismertek.

A szülõfalujában végzett elemi iskola után elõbbTordára, majd Kolozsvárra került. Ott érettségizett azunitárius gimnáziumban, majd matematika–fizika sza-kos tanári oklevelet szerzett a kolozsvári egyetemen.Ezután székely honvédõk leszármazottjaként egyévesönkéntes katonai szolgálatra vonult be. Így csak 1912õszétõl kezdhetett tanársegédként dolgozni TanglKároly mellett, aki Eötvös Lorándnál töltött tanárse-

HÍREK – ESEMÉNYEK 289

Page 76: Fizikai Szemle 68/7–8 – 2018. július–augusztusgoliat.eik.bme.hu/~f.simon/publications/Media/FizSzem-20180708.pdf · okokból standard modellnek hívunk, a világot anyagi és

gédség után a fizika tanára volt Kolozsvárt.

A Gyulai-család az ünnepségen.

Mindössze két év adatott neki, mert az elsõvilágháború kitörése után szinte azonnalbevonult katonának, és kikerült az oroszfrontra. Egy év múlva orosz fogságba esett,és több mint hat évet töltött hadifogolytá-borokban, többek között Novonyikola-jevszkben, a mai Novoszibirszkben.

Amikor 1922 elején hazatérhetett, egy-kori egyeteme, ahol fenntartottak számáraegy tanársegédi állást, már Szegeden mû-ködött. Így itt fejezhette be közel nyolcévnyi kényszerû szünet után a Kolozsvártmegkezdett doktori munkáját. Ezután nõ-sült meg, vette el Grátz Mártát, a korábbikolozsvári evangélikus lelkész egyik leá-nyát, akit valószínûleg még Kolozsvárrólismert, hiszen az evangélikus paplak és azunitárius gimnázium alig száz méterre vanegymástól.

1924-ben ösztöndíjjal Göttingenbe került, ahol kétéven keresztül Robert Wichard Pohl mellett dolgozha-tott. Hazajövetele után elõbb magántanárként, majdrendkívüli tanárként tartott Szegeden elõadásokat, ésvégzett kutatómunkát. Ezek elismeréseként 1935-benegyetemi tanári kinevezést kapott a debreceni egye-tem orvosi kara fizikai intézetébe. Elõdje égi mechani-kával foglalkozott, ezért az általa elképzelt, a kísérletifizikára alapozott tanszéket szinte a semmibõl kellettfelépítenie. Õ hívta meg akkor oda tanársegédnekSzalay Sándort, aki azután utóda lett, miután 1940-ben, Észak-Erdély visszatérte után a kolozsvári egye-tem kísérleti fizika tanszékére nevezték ki.

A kolozsvári tanszék és az oktatás megszervezéséreismét csak rövid idõt hagytak a külsõ körülmények. ABudapesti Mûszaki Egyetem tanácsülése már 1944tavaszán döntött arról, hogy pályázati kiírás nélkülGyulai Zoltánt hívják meg a kísérleti fizika tanszékreegyetemi tanárnak. A háborús front közeledte, majd aháború vége azonban közbeszólt. Bár egy idõre el-hagyta Kolozsvárt, a harcok befejezése után visszatértoda, és két évig még az akkor Bolyai Tudományegye-tem nevet viselõ magyar egyetemen tanított. Ezutánfoglalhatta el az állást a Budapesti Mûszaki Egyete-men, és 1947-tõl nyugdíjba vonulásáig vezette a BMEKísérleti Fizika Tanszéket.

Miért emlékezünk ötven év után is Gyulai Zoltánra?Egykori mesterérõl, Pohlról a Nobel-díjas Sir NevillMott így nyilatkozott: „véleményem szerint a göttinge-ni R. W. Pohl a szilárdtest-fizika igazi atyja”. Ezt to-vábbvíve Gyulai Zoltánról azt mondhatjuk, hogy õ amagyar kísérleti szilárdtest-fizika úttörõje volt.

Még egyetemi hallgatóként kezdett foglalkozni azakkor Hallwachs-effektusként emlegetett fényelektro-mos jelenséggel, pontosabban a belsõ fényelektromosvezetéssel. Fény hatására szigetelõk vagy félvezetõkbelsejében az elektronok egy része magasabb energiá-jú, korábban üres sávba gerjesztõdhet, ami által jelen-tõsen megváltozhat az anyag vezetõképessége. Ez kü-lönösen markánsan jelenik meg szelénben. Gyulai Zol-

tán a szelén fényelektromos vezetésével kapcsolatosvizsgálatairól írta elsõ cikkeit 1912-ben. Hasonló té-mákkal foglalkozott göttingeni tartózkodása idején is.Ezután jött az ötlete, hogy vezetésre képes elektronokatnemcsak fénnyel lehet ionkristályokban kelteni, hanemmechanikai deformációval is. Ezt ma is Gyulai–Hartly-jelenségként emlegetik a szakirodalomban.

Mielõtt 1935-ben a debreceni egyetemre kineveztékvolna, a vallás- és közoktatásügyi miniszter a felterjesz-tésben többek között a következõket írta: „Gyulai Zol-tán … kutatásainak tárgyai a modern, kísérleti fizikánakolyan területein mozognak, amelyek egyre fokozódómértékben állnak az érdeklõdés középpontjában. Dol-gozataiból megállapítható, hogy igen jó kísérletezõ,világosan meglátja a maga elé tûzött problémát, nagykísérleti készséggel és gyakorlati érzékkel vizsgálja azt,és viszi a megoldás felé. Kiemelkedõ kísérletezõi lele-ményessége, amellyel egyszerû eszközökkel és arány-lag kis pénzen is elõ tudja állítani az intenzív és ered-ményes kutatói munkához szükséges berendezéseketés mûszereket. Nemcsak hazai, hanem külföldi szakkö-rök is elismerik és nagyra értékelik Gyulait mint kísérle-tezõ és kutató szakembert.”

Debreceni professzorsága idején kezdett a kristá-lyok növekedésével foglalkozni. Vezetése mellett ek-kor készítette doktori értekezését Tarján Imre. Azakkor kezdett munka kiteljesedésébõl született meg aGyulai–Tarján-féle kristályfizikai iskola. Míg a Tarján-iskola az alkalmazás, az ipar számára fontos kristá-lyok növesztése felé ment el, Gyulait inkább a növe-kedés mechanizmusa érdekelte. Innen fakadt érdek-lõdése a tûkristályok iránt, amelyekrõl megmutatta,hogy szakítószilárdságuk majdnem eléri az ideáliskristályokra elméletileg meghatározott értéket.

Gyulai Zoltán aktív részese volt a magyar fizikus köz-életnek. A Magyar Tudományos Akadémia már 1932-ben levelezõ tagjává választotta, 1954-ben lett rendestag. Az ELFT 1952-ben választotta elnökének, s ezt atisztséget haláláig betöltötte.

Tisztelettel emlékezzünk rá halála 50. évfordulóján.

290 FIZIKAI SZEMLE 2018 / 7–8

Page 77: Fizikai Szemle 68/7–8 – 2018. július–augusztusgoliat.eik.bme.hu/~f.simon/publications/Media/FizSzem-20180708.pdf · okokból standard modellnek hívunk, a világot anyagi és

TANÍTVÁNYOK TANÍTVÁNYAI MTA Wigner FK SZFIHartmann Ervin

Gyulai Zoltán tudományos munkássága a kristályoknövekedése és a kristályhibák vizsgálatának területéreesik. Világhírû cikkei (NaCl-kristályok elektromos veze-tésének megváltozása plasztikus deformációkor, tûkris-tályok „whiskerek” szakítószilárdságának méretfüggõ-sége) mellett [1], legnagyobb érdemének az tartható,hogy egy tudományos iskola alakult ki körülötte [2],amelynek hatása napjainkig nyomon követhetõ. Az el-sõ tanítványok egyike Tarján Imre [3], aki Gyulai tanár-segéde volt Debrecenben, és nála is doktorált 1939-ben. Tarján Imrét 1970-ben az MTA levelezõ, 1976-banrendes tagjává választotta. Míg Gyulait fõleg elvi jelen-tõségû témák (vannak-e kristályhibák, vannak-e „töké-letes kristályok”), addig Tarjánt inkább a gyakorlatia-sabb témák (kristálynövekedés helyett a kristálynövesz-tés) érdekelték. Legtöbbször idézett cikke az extrémtisztaságú alkálihalogenid-kristályok elõállításával fog-lakozott. Gyulainak voltak még tanítványai Kolozsvá-rott, majd életének utolsó évtizedeiben a BME-n. Gyu-lai közvetlen tanítványai körül tanítványok újabb sorajelent meg. Szinte szimbolikus jelentõségû, hogy GyulaiZoltán halálát követõ évben a Tarján Imre vezette MTAKristályfizikai Kutatólaboratórium munkatársa lettJanszky József (1943–2018), aki elsõ cikkét a töltöttdiszlokációkról Tarján Imrével közösen írta. Janszkyt2007-ben az MTA rendes tagjává választották. MindTarján, mind Janszky mellett kristályfizikához nem kö-tõdõ iskolák is kialakultak. Gyulai egyik utolsó cikkétvelem közösen írta. Húsz évvel késõbb nálam kezdtetudományos munkáját a kristályfizikai csoport jelenlegivezetõje. Neki van olyan tanítványa, aki már GyulaiZoltán halála után született. Röviden: a tanítványokközött négy-öt generációt lehet megkülönböztetni.

Gyulairól nem lehet mondani, hogy sem utódja,sem boldog õse. Gyulai elõször a pesti egyetemre irat-kozott be, de hosszantartó tüdõgyulladása miatt hazakellett mennie Erdélybe, és tanulmányait Kolozsváronfolytatta. Itt a kísérleti fizika tanára Tangl Károly [4]akadémikus volt. Elméleti fizikát, abban az idõben ma-tematikai fizikát, a szintén akadémikus Farkas Gyulá-

tól tanult [4]. Gyulai egész életében döntõ jelentõségevolt Ortvay Rudolfnak [4] (levelezõ tag 1925-tõl), akitelõször mint fizikai gyakorlatokat vezetõ tanársegédetismert meg. A hétéves szibériai hadifogság után Ort-vaytól kezdett el újra fizikát tanulni. Többek közöttOrtvay ajánlotta Gyulainak Göttingent. Itt a nála csakhárom évvel idõsebb Robert Wichard Pohl professzorfogadta maga mellé ösztöndíjasként. Pohl a modernszilárdtest-fizika megalapozója. Maga a szilárdtest-fizika elnevezés is Pohl lakásának teraszán, poharaz-gatás közben született. Addig tréfásan „piszkos fizi-ka”-ként volt az élcelõdések tárgya.

Gyulai Zoltán emlékének megõrzésére az elsõ lépéstaz ELFT tette 1969-ben a Gyulai-díj megalapításával. Adíjat eddig 34-en kapták meg. (A díjhoz járó emlékpla-ketten Gyulai születési éve hibásan szerepel, 1888 a he-lyes 1887 helyett.) A BME Könyvtára 1987-ben „GyulaiZoltán” emlékkiállítást rendezett. Az MTESZ ugyanab-ban az évben emléktáblát helyezett el az egyik szegedigimnázium falán. Gyulai így ír elsõ szegedi éveirõl:„Lakni az intézetben laktam, mert az intézet üres volt.Egy szegedi gimnáziumban helyezték el a fizikai ésvegytani intézeteket.” Gyulai szülõfalujában, az erdélyiPipén az MTA, az Unitárius Egyház és a Babes-BolyaiEgyetem 1998-ban helyezett el emléktáblát. Gyulai Zol-tán mellszobrát a Mûegyetem kertjében 2012-ben avat-ták fel. Az egyetem rektora, egykori építõmérnök-karidiák, ünnepi beszédében nemcsak a kísérletekkel gaz-dagon demonstrált elõadásokra, hanem Gyulai „ugye-bár” szavajárására is visszaemlékezett.

Nemcsak tanítványaiban él tovább a tanár, hanemtanítványai tanítványaiban is, ezért halhatatlan õ.

Irodalom

1. Hartmann E.: Magyar ötletek és hazai megvalósításuk. MagyarTudomány (2003/12) 1559–1565.

2. http://real.mtak.hu/74/1/35044_ZJ1.pdf3. Hartmann E.: Tarján Imre a magyar kristályfizikában. Fizikai

Szemle 62/7–8 (2012) 230–233.4. A Magyar Tudományos Akadémia Tagjai 1825–2002. (fõszerk.:

Glatz F.), MTA Társadalomkutató Központ, Budapest 2003.

BÚCSÚ: BONIFERT DOMONKOSNÉ BOTTYÁN KATALINA Szegedi Tudományegyetem Juhász Gyula Pedagó-gusképzõ Kar Általános és Környezetfizikai Tanszékemegrendülten tudatja, hogy 2018. május 26-án el-hunyt Bonifert Domonkosné Bottyán Katalin, a tan-szék nyugalmazott fõiskolai docense, intézményünk1989–1994 közötti fõigazgató-helyettese. Személyé-ben nemcsak egy nagyszerû kollégát, kiváló oktatót,nagyra becsült pedagógust veszítettünk el, hanemtávozott közülünk a tanszék szakmódszertani csoport-ját több évtizeden át vezetõ oktatója, a hallgatók sze-

retett Kati nénije. Szárnyai alatt fizikatanár-szakoshallgatók százai ismerkedtek meg a fizika tanításánakalapvetõ módszertani fogalmaival, az oktatás ezernyititkával, kaptak útmutatót a tanítás hétköznapi tenni-valói megszervezéséhez. A Tanárnõ diákjai nemcsakolyan szakmai és módszertani ismeretek birtokábanléptek ki intézményünk kapuján, amely már másnaphiteles és katedraképes teljesítményt garantált, hanemazt is megtanulták, hogy becsülettel és hûséggel kellhelytállni a tanári pályán.

HÍREK – ESEMÉNYEK 291

Page 78: Fizikai Szemle 68/7–8 – 2018. július–augusztusgoliat.eik.bme.hu/~f.simon/publications/Media/FizSzem-20180708.pdf · okokból standard modellnek hívunk, a világot anyagi és

Docendo discimus. A „tanítva tanulunk” elv vezé-relte a munkájában, s ezt is sikeresen adta tovább tanít-ványainak, nemcsak szóban, írásban is. Számos tudo-mányos közlemény fûzõdik nevéhez. 1992–2014 közötta Fizika Tanítása folyóirat fõszerkesz-tõje és rendszeres szerzõje is volt.

Sok tankönyv, segédkönyv õrzinevét, mint szerzõét vagy szerkesz-tõét. A jogelõd Fizika Tanszéken1975-ben – a Tanárnõ aktív részvéte-lével – egy kutatócsoport alakult azzala céllal, hogy az alapfokú oktatáshoztankönyveket és egyéb oktatási se-gédanyagokat készítsenek. Az igenerõs és elismert módszertani csoportkutatómunkája 2013-ig azzal a céllalfolyt, hogy a 10–14 éves korosztályszámára a NAT-nak, illetve a kerettan-tervi elõírásoknak megfelelõ, a tanu-lók életkori sajátosságait messzeme-nõen figyelembe vevõ, szakmailag ésmódszertanilag magas színvonalú tankönyvek íródja-nak. A tantervi reformok újabb és újabb tankönyvcsa-lád megjelenését, majd átdolgozását tették szükséges-sé. Az átdolgozás mellett – az eredményesebb fizika-tanítás érdekében – feladatgyûjtemények, tudásszint-mérõ feladatlapok is készültek. A tankönyvek és okta-tási segédanyagok, amelyek mind a mai napig hasz-

nálatosak a magyarországi általános iskolákban, aMozaik Kiadó gondozásában jelentek meg.

A Juhász Gyula Tanárképzõ Fõiskola az 1990-esévektõl a hagyományos fõiskolai képzései mellé új

képzési formákat vezetett be. 1998-ban a Fõiskolai Tanács létrehozta aSzakképzési, Továbbképzési és Táv-oktatási Intézetét, amely a kor kihívá-sainak megfelelõ, színvonalas szak-képzési, továbbképzési munkát vég-zett/végez. Az intézet igazgatói fel-adatát – nyugdíjba vonulásáig – 2007-ig, a Tanárnõ látta el.

Férjével, Bonifert Domonkossalegyütt széleskörûen ismertek voltak,mint az általános iskolai matematika-és fizikaoktatás zászlóvivõi – határain-kon belül és túl is. Végtelenül sokattett a fizika és a matematika népszerû-sítése és a tehetségek gondozása ér-dekében. Évtizedeken keresztül szer-

vezte és segítette a 10–14 éves tanulók helyi és me-gyei szintû fizika- és matematikaversenyeit, ez utóbbita Bonifert Domonkos Alapítványon keresztül mégnyugdíjas éveiben is nagy örömmel tette.

A Tanárnõ emlékét kegyelettel megõrizzük.Az Általános és Környezetfizikai Tanszék

munkatársai

HUMBOLDT-DÍJBAN RÉSZESÜLT LEGEZA ÖRS2018 június 28-án az Alexander von Humboldt Alapít-vány Humboldt Research Award díjjal tüntette ki Le-geza Örsöt, a Magyar Tudományos Akadémia WignerFizikai Kutatóközpont tudományos tanácsadóját.

Legeza Örs, a tenzorhálózat-algoritmusok és a kvan-tuminformáció-elmélet ötvözésének nemzetközileg el-ismert úttörõje számos alapvetõ, új eredményt ért el afizika és kémia területén. Kutatásai olyan, a kvantum-mechanika alaptörvényeire épülõ új matematikai algo-ritmusok kifejlesztésére fókuszálnak, amelyek lehetõvéteszik a korábbiaknál jóval komplexebb kvantumrend-szerek numerikus szimulációs vizsgálatát, illetve visel-kedésük elõrejelzését, vagy akár tulajdonságaik terve-zését. Az általa fejlesztett számítógépes programokat avilág számos kutatóintézetében és kutatóegyeteménnagy sikerrel alkalmazzák, például anyagi tulajdonsá-gok szimulációira szilárd testekben, molekuláris kvan-tumkémiában, magfizikában, illetve az információtech-nológia kvantumos szimulációjában. Számításaikkal ha-tékonyan szimulálják az olyan kísérletileg is megépít-hetõ kvantumos rendszereket, az ultra-hideg atomokat,amelyektõl a szakma például a kvantumszámítógépek,vagy éppen a magas hõmérsékletû szupravezetõk kifej-lesztését reméli. Az MTA Wigner FK-ban 2012-benhozta létre az Erõsen korrelált rendszerek „Lendület”

kutatócsoportját, amely számos nemzetközi együttmû-ködésben vesz részt Európában és más kontinensen.Csoportja több olyan eredményt ért el, amelyek a jelen-legi konvencionális módszerekkel nem lettek volnalehetségesek. Többek között mágneses anyagok éskorrelált elektronrendszerek viselkedését vizsgálvakülönféle egzotikus kvantumfázisok létét mutatták ki,illetve olyan átmenetifém-klaszterek molekulapályáiközött fennálló összefonódottsági képeket határoztakmeg, amelyek fontos szerepet játszhatnak biokémiaireakciók során. Másfél éve egy amerikai együttmûkö-dés keretében megkezdték algoritmusaik integrálását aPacific Northwest National Laboratory-ban fejlesztettNWChem professzionális programcsomagba, ami – vár-hatóan – a jövõben a kvantumtechnológiai elméleti ku-tatások meghatározó szoftveres alkalmazása lesz.

A Humboldt-díjat, e kivételes kitüntetést olyan, pá-lyájuk csúcsán álló, nemzetközileg megbecsült, Német-országon kívül dolgozó kutatóknak adományozzák,akiktõl még jelentõs eredmények várhatók. Oket egyévig vendégül látják Németországban, hogy az általukválasztott területen együtt dolgozzanak német kollé-gáikkal. Legeza Örs Ulrich Schollwöck professzor ven-dége lesz a Ludwig-Maximilians-Universität München(Lehrstuhl für Theoretische Nanophysik) osztályán.

292 FIZIKAI SZEMLE 2018 / 7–8

Page 79: Fizikai Szemle 68/7–8 – 2018. július–augusztusgoliat.eik.bme.hu/~f.simon/publications/Media/FizSzem-20180708.pdf · okokból standard modellnek hívunk, a világot anyagi és

Természettudományos fesztivált és kiállítást rendez az Eötvös Loránd

Fizikai Társulat, az Informatika-Számítástechnika Tanárok Egyesülete,

a Bolyai János Matematikai Társulat, a Magyar Kémikusok Egyesülete

és a Magyar Biológiatanárok Országos Egyesülete 2018. október 5–7-ig

Szegeden, a Szent-Györgyi Albert Agórában. THE EUROPEAN NETWORK OF SCIENCE TEACHERS

Szervezõk:

ÚJ UTAKON A DIGITÁLIS GENERÁCIÓHOZ

SZEGED, 2018.OKTÓBER 5–7.

MAGYAR SCIENCE ON STAGE FESZTIVÁL

Page 80: Fizikai Szemle 68/7–8 – 2018. július–augusztusgoliat.eik.bme.hu/~f.simon/publications/Media/FizSzem-20180708.pdf · okokból standard modellnek hívunk, a világot anyagi és

myDAQ-pályázat a 2018–2019. tanévre

Az ELFT és az NI Hungary Kft. az idén is meghirdeti pályázatát fizikatanároknak.

http://sukjaro.eu/ELFT-NI-palyazat

2018. szeptember 16., 24:00 óra.

2019. január 25., 24:00 óra.

először induló

tapasztaltabb,

általános iskolásokkal

A pályázat célja, hogy tanórákon, szakkörökön minél több iskolai kísérlet legyen

elvégezhető a myDAQ eszköz használatával is.

A pályázati kiírás, a pályázat témája, a pályázati határidők és a jelentkezés feltételei

megtalálhatók a weblapon.

Az idei pályázatban a korábbi évekhez képest változás, hogy a 10 tárgyjutalmat a

következőképpen osztják meg a szervezők:

• 5 tárgyjutalmat az öt legjobb pályázó kapja,

• a fennmaradó 5 tárgyjutalmat pedig a már legalább második éve

induló pályázók.

A szervezők fenntartják a jogot, hogy egy 11. helyezettet is díjazzanak, aki a legjobb,

induló pályázó lesz (amennyiben lesz ilyen induló).

A pályázati regisztráció határideje:

Az elkészült pályamunkák leadási határideje:

www.ni.com

9 7 7 0 0 1 5 3 2 5 0 0 9 70081

ISSN 0015325 - 7